Paediatrics Flashcards

1
Q

A 14-year-old girl comes to see you as she has not had her periods yet. You note
that her breasts are stage II and her nipples are set lateral to the mid-clavicular
line. She has no pubic hair. Her weight is on the 50th centile but height is on
the 9th centile. Her parents are both of average height. What is the most likely
diagnosis?

A. Turner’s syndrome
B. Polycystic ovary syndrome
C. Anorexia
D. Constitutional delay
E. Underlying undiagnosed chronic illness

A

A. Turner’s syndrome

Turner’s syndrome (A) is karyotypically 45XO. There are the classic signs of wide spaced niples, short stature, lack of secondary charecteristics, amenorreha and inferlity. Another classic feature not mentioned here is a webbed neck.

PCOS (B) is unlikely as this girl hasn’t started puberty at all. PCOS tends to lead to secondary amenorehoa or oligomenorehoa.

Her weight is normal, so this isn’t anorexia (C)

Constitutional delay (C) would be a simple isolatted delay in seletal growth, it would not have these syndromic features.

There is nothing here to sugest an underlying chronic illness (E), there is also no evidence of failure to thrive.

How well did you know this?
1
Not at all
2
3
4
5
Perfectly
2
Q

A 2-year-old child is referred to you by the GP because he has not started walking.
His mother says that he can stand but cries to be picked up or sits down shortly.
His older sister was walking by 14 months. You note that he is talking well with
short two to three word phrases. He is able to build a tower of six blocks. What is
your management plan?

A. Advise mum to work harder at giving him independence and follow up
in 4 months.
B. Request blood tests including a creatinine kinase
C. Refer for physiotherapy
D. Reassure and discharge as his development is normal
E. Refer to orthopaedics

A

B. Request blood tests including a creatinine kinase

This child has abdormal developmentisolated to gross motor skills, this is not normal (D), nor is discharge and review an appropriate step (A).

The concern here is that there is a muscular dystrophy, the history of being able to stand but fatiguing quickly is classical. Male gender also makes this possible as these are X-linked recessive conditions. The best screening test here is a CK (B), an abnormal CK would lead to a muscle biopsy to confirm a diagnosis of MD.

Hip dysplasia would be another reason for failure to walk, the first step in that case would be an utrasound, not an orthopaedic referal (E).

Without nowing the cuase of the problem, a referal to physio would be incorrect (C).

How well did you know this?
1
Not at all
2
3
4
5
Perfectly
3
Q

You see a boy in outpatients whose parents are concerned he is not talking yet.
You do a developmental assessment and find he is walking well and able to build
a tower of three blocks. He will scribble but does not copy your circle. He is able
to identify his nose, mouth, eyes and ears as well as point to mummy and daddy.
You do not hear him say anything but his parents say he will say a few single
words at home such as mummy, daddy, cup and cat. He is a happy, alert child.
Parents report him to be starting to feed himself with a spoon and they have just
started potty training but he is still in nappies. What is the child’s most likely age?

A. 12 months
B. 15 months
C. 18 months
D. 2 years
E. 2.5 years

A

C. 18 months

This child achieves the 18 month goals in all four developmental domains.

learn the attached table!

http://medimaps.co.uk/wp-content/uploads/2013/12/Paediatrics_developmental_milestones_Ver_0.2.png

How well did you know this?
1
Not at all
2
3
4
5
Perfectly
4
Q

A one-and-a-half-year-old Caucasian child is referred to paediatrics for failure
to thrive. On examination he is a clean, well-dressed child who is quite quiet
and withdrawn. He is pale and looks thin with wasted buttocks. His examination
is otherwise unremarkable. What is the most likely cause of this child’s growth
failure? His growth chart shows good growth along the 50th centile until 6 months followed by weight down to the 9th, height down to 25th and head circumference now starting to falter at 1.5 years.

A. Coeliac disease
B. Neglect
C. Constitutional delay
D. Normal child
E. Beta thalassaemia

A

A. Coeliac disease

The clue in this case is that the child was normal, until 6 months, when he was presumably weaned onto gluten containing foods. It was only at this stage he began to fail to thrive. The pale description is likely anaemia due to malnutrition, wasted buttocks is a clasic coeliac sign (A).

Neglect (B) is an important consideration in any case of failure to thrive, but it is unlikely as he appears well cared for. His withdrawn nature, in this case, is likey due to chronic illness.

Constitutional delay (C) is an isolated finding of delayed skeletal growth, it is usualy seen at puberty.The failure to gain weight and the other symptoms go against this.

He has moved across the growth and weight curves, he is not a normal child (D).

Beta thalassaemia (E) is extremely rare in a caucasian child, symptoms would have appeared at 6 months (when the fetal Hb is discontinued). He would not have gotten to 1.5 years of age without transfusions and a diagnosis.

How well did you know this?
1
Not at all
2
3
4
5
Perfectly
5
Q

A 12-year-old boy presents to his GP with left-sided unilateral breast development
stage III. He is very upset as he is being bullied at school. His mother is worried as
her friend’s sister has just been diagnosed with breast cancer and wants to know
if he could have breast cancer? What is the management?

A. Refer for a breast ultrasound
B. Test sex hormone levels
C. Test alpha fetoprotein
D. Reassure and explain this is a normal part of puberty; it will resolve but
the other breast may enlarge transiently as well
E. Do a fine needle aspirate on his left breast

A

D. Reassure and explain this is a normal part of puberty; it will resolve but
the other breast may enlarge transiently as well

it is common for adolescent males to transiently undergo breast development. THe correct action here is to reassure and counsel that there may be transient development of the other breast (D).

Investigations (A, B, C, and E) would only be warrented if this did not resolve.

The order of male pubertal development; begins with testicular volume increasing above 4ml, penis enlargement, pubic hair growth, and finally the growth spurt.

How well did you know this?
1
Not at all
2
3
4
5
Perfectly
6
Q

You see a baby for the first baby check at 6 weeks. Mum reports no problems
and he is feeding well. On examination you are unable to palpate the testicles
on ether side and do not feel any lumps in the groin area. He has a normal penis
with no hypospadias and the anus is patent. He is otherwise a normal baby on
examination. What is the most important diagnosis to rule out?

A. Klienfelter’s syndrome
B. Congenital adrenal hyperplasia
C. Undescended testicles
D. Virilized female infant
E. Testicular cancer

A

B. Congenital adrenal hyperplasia

CAH (B) is most commonly casued by 21-hydroxylase deficiency. the presentation is with infants of ambiguous genitalia, or bilateral undescended testes. Because they are at risk of a salt-losing crisis around 1-3 weeks of age it is crucial to investigate this possibility. You would measure U&Es and carry out chromosomal analysis, as well as a pelvic ultrasound to look for the testes. .

This could be a virilised female infant (D), but there would be an underlying reason for this (CAH for example).

Undescended testes (C) is an important diagnosis, as if they persist outside of the scrotom past the age of 2 then there is an increased risk of developing testicular cancer (E). As this is a newborn and not a 2 year old we cannot make this diagnosis.

Klienfelter’s (A) syndrome is karyotypically 47XXY and includes; tall stature, delayed puberty, and mild learning difficultiesbut no link to undescended testicles.

How well did you know this?
1
Not at all
2
3
4
5
Perfectly
7
Q

You see an 8-year-old boy in accident and emergency who fell off his bike 3 days
ago and scraped his left calf. The cuts are now angry, red and painful. You note he
is a big boy and plot his growth: his weight is on the 99th centile and height is on
the 75th centile. You note mild gynaecomastia and stretch marks on his abdomen
which are normal skin colour. His past medical history is unremarkable except for
mild asthma. What is the most likely cause of his large size?

A. Cushing’s syndrome secondary to a pituitary adenoma
B. Cushing’s syndrome secondary to becotide inhaler use
C. Obesity
D. His size is within the normal range and is a variant of normal
E. Liver failure

A

C. Obesity

If this were Cushing’s (A or B) you would expect his growth to be impaired, which is not the case here. In addition the specific mentioning that the striae are normal skin colour goes against Cushings, which is classically purple.

It is also important to be aware that inhaled steroids (B) do not provide enough systemic steroid to cause Cushings syndrome. The child would have to be on high dose oral steroids.

The fact that he is on the 99th centile for weight immediately tells you that this is outside the normal variant range (D), this is confirmed by the mismatch between the centiles of height and weight.

There is nothing in the history sugestive of a severe condition such as liver disease. If the child had liver failure (E) he would be extrememely unwell, and probably not riding his bike.

The gynaecomastia is more likely to be fat deposition due to obesity (C), this child should be encouraged to continue exercise and to eat more healthily. There is not normally a reason to put a child on calorie controlled diets as normal growth should facilitate weight loss.

How well did you know this?
1
Not at all
2
3
4
5
Perfectly
8
Q

A 16-year-old boy is brought to the GP by his parents. They are concerned he is the shortest boy in his class. He is otherwise well. His height and weight are on the 9th centile. His father plots on the 75th centile and his mother on the 50th centile for adult height. On examination, his testicular volume is 8 mL, he has some fine pubic and axillary hair. The rest of the physical examination is normal. On further questioning you elicit from his father that he was a late bloomer and did not reach his full height until he was at university. What is the most likely cause of the boy’s short stature?

A. The 9th centile is a normal height and weight so there is nothing wrong
with him
B. Growth hormone deficiency
D. Underlying chronic illness should be sought
E. Anorexia

A

C. Constitutional delay of growth and puberty

This pattern of late blooming (classically in the male line) is sugestive of constitutional delay (C). A constitutional delay presents as a delay in skeletal growth with no other pathological features.

The boy is progressing through puberty in the normal way, just delayed. The order being increased testicular volume (above 4ml), penis growth, then pubic hair, with the growth spurt being the final stage. This makes a GH deficincy (B) or an underlying chronic illness (D) unlikely.

His height and weight are on the same centile, so this makes anorexia (E) unlikely as you would expect the weight to be much lower than the height.

Given his parent’s height you would expect him to be taller, so this is not normal (A). as an estimate you would expect a child to be the mean of the parental height, plus 7cm in boys, minus 7cm in girls, with a leeway of two standard deviations.

How well did you know this?
1
Not at all
2
3
4
5
Perfectly
9
Q

A mother comes to see you with her 2-year-old daughter, Stacey, out of frustration
that her daughter is so ill behaved. She does not know how to make her listen and is worried that she is going to get hurt. Yesterday she ran ahead and did not stop when her mother called to her. She ran into the street and was hit by a cyclist, but fortunately he was OK and Stacey had only had a few cuts and scrapes and seems alright! On questioning you hear other stories of a naughty child. She is active and eats well, feeding herself a lot now, but her mother does say she gets frequent coughs and colds. Her mother says that Stacey only says about 5–10 words and only she can understand what Stacey says. What is the best next management?

A. Ask the health visitor to visit mum for parenting advice and support
B. Order blood tests for full blood count to check for leukaemia as she has
recurrent coughs and colds
C. Give Stacey a tetanus shot to cover her after her fall the day before

D. Refer for a hearing test
E. Tell Stacey that she needs to listen to her mother and not have any
more accidents

A

D. Refer for a hearing test

There appears to be a speech delay here. The child is feeding herself and running, which is normal for a 2 year old but her speech is closer to that of an 18 month old. The other feature of note is that this vignette suggests that what speech she is producing is abnormal if only the mother (who has grown used to it) can understand.

All of this would raise concern of a hearing problem (D), commonly due to glue ear associated with reccurent coughs and colds.

A child this age would be expected to get around 8 coughs and colds a year, on average, so any referal for a blood cancer (B) would be premature.

Her routine vaccinations would cover for tetanus, so further vaccination would be unecessary (C) unless there was evidence she had missed a scheduled vaccine. Also this answer completely misses the point of the consultation!

(A) and (E), similarly fail to adress the underlying concerning signs.

How well did you know this?
1
Not at all
2
3
4
5
Perfectly
10
Q

An older mother books in to see you after attending the health visitor for a weight
check at 2 months for her first child. She and her husband have had a hard time
coming to terms with their daughter’s diagnosis of Down’s syndrome. She is
relieved that the appointment with the cardiologists went well and the heart is
normal. However they have a lot of trouble getting her to take the whole bottle,
she was slow to regain her birth weight and looking at the plotted weight yesterday she is not growing along her birth centile and the mother is worried she is not doing a good enough job. She is not vomiting except for small possets after feeds, is passing urine and opening her bowels. The red book growth chart shows the weight to be falling off centiles. What is the most appropriate management?

A. Contact the cardiologists in light of the poor feeding and slow weight
gain for a second opinion as baby’s with Down’s syndrome are at high
risk of heart problems and they may have missed it
B. Refer to the dietician for nutritional support
C. Replace the growth chart in their red book with a Down’s syndrome
growth chart, reassure mum by re-plotting her growth and explain she
is normal but arrange to review again
D. Tell the mother to try a different milk and come back in 2 weeks
E. Advise the mother to change to a faster flow teat for their bottles so
that she takes her feed faster

A

C. Replace the growth chart in their red book with a Down’s syndrome
growth chart, reassure mum by re-plotting her growth and explain she
is normal but arrange to review again

It is important to remember that children with Down’s syndrome have different rates of growth and so the stanard growth charts are not appropriate (C). The same goes for other conditions, such as Cystic Fibrosis.

Referal back to cardiology (A) would unduely worry the mother and is rather insulting to the specialists to imply they missed something obvious.

Options (B), (D) and (E) al assume there is a feeding problem of some kind, which in all likelehood is not the case.

How well did you know this?
1
Not at all
2
3
4
5
Perfectly
11
Q

Which child should be moved to the resuscitation area for urgent management in
accident and emergency?

A. A miserable 2 year old with a fever and vomiting temperature of 38.5°C, heart
rate of 150, respiratory rate 42, capillary refill time 2–3 seconds who is alert and
clinging on to his father and has just been given paracetamol and started on a
fluid challenge with oral rehydration salts 5 minutes ago by the triage nurse
B. A quiet 4 year old brought in with an asthma attack who is sitting
upright with a respiratory rate of 50, heart rate of 162, capillary refill
time of 3 seconds, subcostal recessions and poor air entry on chest
auscultation following a salbutamol nebulizer
C. An 8 year old, known diabetic, brought in vomiting with her glucose
reader saying HI. She is able to tell you her history and has a heart rate
of 120, respiratory rate of 25, capillary refill time of <2 seconds
D. An alert 3 year old who has had a cough and cold for the past 3 days
which is keeping him up at night and mum noticed a rash on his neck
and face which did not disappear when she pressed a glass tumbler
against it. His temperature is 37.8°C, heart rate is 110, respiratory rate is
30, capillary refill time is <2 seconds
E. A 15 year old, known to social services for a family history of domestic
abuse, brought in to accident and emergency by her best friend after
she admitted to taking 20 paracetamol tablets 4 hours ago. She is alert
but does not make eye contact, her heart rate is 98, respiratory rate is
20, capillary refill <2 seconds

A

B. A quiet 4 year old brought in with an asthma attack who is sitting
upright with a respiratory rate of 50, heart rate of 162, capillary refill
time of 3 seconds, subcostal recessions and poor air entry on chest
auscultation following a salbutamol nebulizer

The child in (B) is seriously unwell and the sickest child here. She needs to be moved to resus and have a dedicated doctor and nurse managing her. She has deranged observations and signs of circulatory compromise, with the cap refil time being >3 secs, and there has been no responce to the nebuliser.

Child (A) is unwell but has just been given treatment, the pudent course of action is to se if his/her observations improve after the anti-pyretic and fluid challenge.

Child (C) is potentially heading towards a DKA, but at this point in time is alert with no circulatory compromise. She should be closely monitored as there is a risk that she could rapidly deteriorate. Needs a blood gas first and foremost.

Child (D) has a non blanching rash, but is actually pretty well. There are no signs to suggest that he is septic, so this is unlikely to be meningococcal sepsis. He should none the less, be isolated in a side room and be reviewed by a senior clinician.

Child (E) Has the potential to become quite sick, but is stable currently. She needs blood levels of paracetamol measured and would benefit from a private space (a side room) to have a psych consult when medically cleared.

How well did you know this?
1
Not at all
2
3
4
5
Perfectly
12
Q

A 4-year-old child has been losing weight recently and has been vomiting for the
past 24 hours, unable to eat anything. His mother has brought him into accident
and emergency out of concern as he seems confused. The triage nurse has taken
him to the resuscitation room and asked for your help. On examination he is
drowsy, has a heart rate of 150, respiratory rate of 60 and a central capillary refill
of 5 seconds. He has subcostal recessions and good air entry bilaterally with no
added sounds. He moans when you examine his abdomen but there are no masses. You put in a canula and take bloods. The venous blood gas shows:

pH 7.12
PCO2 2.3 kPa
PO2 6.7 kPa
HCO3
–15.3 mmol/L
BE –8.6
Glucose 32.4 mmol/L

What is the most likely diagnosis and what is the first management step?

A. Diabetic ketoacidosis, start an insulin infusion
B. Diabetic ketoacidosis, give a fluid bolus
C. Pneumonia, start IV co-amoxiclav
D. Ruptured appendix, give a fluid bolus and book the emergency
operating theatre
E. Gastroenteritis with severe dehydration, give a fluid bolus

A

B. Diabetic ketoacidosis, give a fluid bolus

It may sound obvious, but remember that diabetic ketoacidosis requires thre components for a diagnosis. There must be evidence of diabetes, as in the glucose reading in this case. There must be acidosis, pH 7.12 in this case. Finally, there must be ketones in the urine, this isn’t mentioned here, but the diagnosis of DKA is most likely. We se other classic signs of DKA, not to be confused with other pathology, the tachypnoeia of metabolic acidosis and the abdominal pain of Ketoacidosis.

The first management if DKA is careful fluid resuscitation (B) and insulin only afteran hour of resuscitation (A). It is importatnt to go slowly with the resuscitation as children are very prone to cerebral oedema, and there is a risk with rapid salt changes of brainstem demyelination. Complete correction should take place over 2 days.

Of ther other options, there is not a respiratory acidosis, so this is not pneumonia (C). With an abnormal glucose any GI pathology is unlikely (D and E).

How well did you know this?
1
Not at all
2
3
4
5
Perfectly
13
Q

An 8 year old known asthmatic is brought into accident and emergency by
ambulance as a ‘blue call’. He has been unwell with an upper respiratory tract
infection for the past 2 days. For the past 24 hours his parents have given him
10 puffs of salbutamol every 4 hours, his last dose being 90 minutes ago. The
ambulance staff have given him a nebulizer but he remains agitated with a
heart rate of 155, respiratory rate of 44 and sub/intercostal recessions and on
auscultation there is little air movement heard bilaterally. Saturations in air are 85
per cent. He is started on ‘back to back’ nebulizers with high flow oxygen. How
severe is his asthma exacerbation and what other bedside test would support this?

A. Moderate, venous blood pH 4.4, gas PCO2 = 3.1 kPa
B. Severe, peak flow <33 per cent expected
C. Severe, venous blood pH 4.4, gas PCO2 = 3.1 kPa
D. Life-threatening, peak flow <33 per cent expected
E. Life-threatening, venous blood pH 4.4, gas PCO2 = 3.1 kPa

A

D. Life-threatening, peak flow <33 per cent expected

The signs of a life-threatening attack here are: Agitation and cyanosis indicating hypoxia, and a silent cheast indicating severely compromised airways. This life threatening atack would be confirmed by a peak flow of <33% of expected (D)

This is more serious attack than suggested by (B)

Blood gases are not used to assess asthma in children, if it were used then a severe or life threatening asthma attack would be shown as a rising CO2 as the lungs would not be effectively exchanging gas. In these examples (A, C, E) there is a hypocapnia indicating hyperventilation.

How well did you know this?
1
Not at all
2
3
4
5
Perfectly
14
Q

The accident and emergency triage nurse asks you to look at a 3-year-old child
with a short history of waking up this morning unwell with a cough and fever. She
looks unwell, heart rate is 165, respiratory rate 56, saturations of 96 per cent in
air, temperature of 39.3°C and central capillary refill of 4 seconds. She has a mild
headache but no photophobia or neck stiffness and you notice a faint macular rash on her torso and wonder if one spot is non-blanching. You ask the triage nurse to move her to the resuscitation area and call your senior to review her. Fifteen minutes later your senior arrives and the spot you saw on the abdomen is now non-blanching and there is another spot on her knee. What are the three most important things to give her immediately?

A. High flow oxygen, IV fluid bolus, IV ceftriaxone
B. IV fluid bolus, IV ceftriaxone, IV methylprednisolone
C. High flow oxygen, IV ceftriaxone, IV fresh frozen plasma
D. IV fluid bolus, IV ceftriaxone, IV fresh frozen plasma
E. High flow oxygen, IV ceftriaxone, IV methylprednisolone

A

A. High flow oxygen, IV fluid bolus, IV ceftriaxone

This child has a classic presentation of meningococcal sepsis and should be treated accordingly.

The prescence of a non-blanching rash indicates that she has disseminated intravascular coagulation and will likely need fresh frozen plama (C and D), but blod tests are needed first, so it is not the first step.

High flow oxygen should always be given to sick children as a first action, it is part of the ‘A’ of the ‘ABCD’ approach. This leaves us with options of answers (A) or (E), of the two, (A) is better as we need to give a fluid bolus (20ml/kg) in a child showing signs of circulatory compromise. The child is also not displaying signs of meningitis at this stage, so the utility of steroids (E) is unclear.

How well did you know this?
1
Not at all
2
3
4
5
Perfectly
15
Q

A 9-year-old boy is brought in by ambulance having been hit by a car while
playing football in the street. You have been assigned to do the primary survey in
resus when the ambulance arrives. The patient is receiving oxygen, crying for his
mummy and holding his right arm, but able to move over from the stretcher to the
bed when asked. Which is the correct examination procedure?

A. The trachea is deviated to the right. On auscultation you hear decreased
air entry on the left. Percussion note is hyper-resonant on the left. He is
tachycardic and his heart sounds are muffled, heard loudest at the right
lower sternal edge. You ask for a left-sided thoracocentesis.

B. You introduce yourself and tell him that you will be gentle but need to
check that he is okay. You see his left wrist is deformed and swollen and
check the fingers which are cool and note the capillary refill is 4 seconds.
He is able to feel you touching him and moans when you examine the
wrist. You call for an x-ray to assess the probable fracture in the wrist.

C. You introduce yourself and tell him that you will be gentle but need to
check that he is okay. You listen for equal air entry and think there is
decreased air entry on the left but there is air entry on the right. He is
tachypnoeic and has a pulse which is tachycardic. His capillary refill is
4 seconds. You expose his abdomen and notice bruising and grazes to
the left side. He moans as you palpate in the left upper quandrant and
has guarding. You ask for an IV canula or intraosseous needle and a
20 mL/kg fluid bolus while organizing an urgent CT chest and abdomen.

D. You introduce yourself and tell him that you will be gentle but need
to check that he is okay. He is tachypnoeic. The trachea is deviated
to the right. On auscultation you hear decreased air entry on the left.
Percussion note is hyper-resonant on the left. He is tachycardic and his
heart sounds are muffled, heard loudest at the right lower sternal edge.
You ask for a left-sided thoracocentesis.

E. You listen for equal air entry and think there is decreased air entry on
the left but there is air entry on the right. He is tachypnoeic and has a
pulse which is tachycardic. His capillary refill is 4 seconds. You expose
his abdomen and notice bruising and grazes to the left side. He moans
as you palpate in the left upper quandrant and has guarding. You ask
for an IV canula or intraosseous needle and a 20 mL/kg fluid bolus
while organizing an urgent CT chest and abdomen

A

D. You introduce yourself and tell him that you will be gentle but need
to check that he is okay. He is tachypnoeic. The trachea is deviated
to the right. On auscultation you hear decreased air entry on the left.
Percussion note is hyper-resonant on the left. He is tachycardic and his
heart sounds are muffled, heard loudest at the right lower sternal edge.
You ask for a left-sided thoracocentesis.

This question is addressing your understading of how to perform an examination and the ATLS framework of the A-B-C-D approach. Options (A) and (E) can be excluded straight away because you have failed to introduce yourself, which is always a crucial step in gaining cooperation and trust from the patient.

Option (B) is an examination of disability and misses all the conditions that might be found during the ‘A’, ‘B’, and ‘C’ stages.

Option (C) recognises the tension pneumothorax but does nothing to address it before continuing the primary survey.

Option (E) is secondarily wrong because, as above, it fails to address the tension pneumothorax after it is discovered.

How well did you know this?
1
Not at all
2
3
4
5
Perfectly
16
Q

A 6-year-old boy with a history of anaphylaxis to peanuts is brought in by
ambulance unconscious. He was attending a children’s birthday party. His mother
says there was a bowl full of candy and he may have eaten a Snickers bar but she
is not sure and she did not have his EpiPen with her. His face and lips are swollen and erythematous, he is still breathing but weakly and there is wheeze. His pulse is tachycardic and thready. Which type of shock is this?

A. Hypovolaemic
B. Distributive
C. Septic
D. Cardiac
E. Obstructive

A

B. Distributive

Shock is the hypoperfusion of tissues, to an extent that is insufficient to meet those tissues metabolic needs. In the case of anaphylaxis there is a shift of fluid to the interstitium due to capiliary leakage, this is a distributive shock (B).

If the leaky capiliaries and distributive shock was in the background of an infection, then this would be septic shock (C).

Blood loss or serious dehydration causes hypovolaemic shock (A). You would expect a pale, cool patient with weak pulses.

Cardiac shock (D) is on the background of cardiac insufficiency, you would expect cardiac symptoms of cheast pain. It is rare in childhod outside of congenital abnormalities or Kawasaki’s disease.

Obstructive shock (E) is due to a mechanical force stopping cardiac output, such as a tamponade or a tension pneumothorax.

How well did you know this?
1
Not at all
2
3
4
5
Perfectly
17
Q

A 13 month old is brought in having had a blue floppy episode at home lasting
1 minute. While you are taking a history from the mother, you notice the baby has
gone blue again and seems to be unconscious in her arms. You call for help and
place the baby on the examination table. There is no obvious work of breathing.
The nurses bring the crash trolley and give you a bag valve mask, which they are
connecting to the oxygen. You give two inflation breaths but do not see the chest
rise. You reposition the air way and this time the breaths go in. You feel for a pulse
and there is none. When asked to do CPR the nurse asks for direction on how many breaths and compressions you both need to do.

A. Two inflation breaths per 30 compressions
B. Two inflation breaths per 15 compressions
C. Continuous inflation breaths about 10–12 per minute and compressions
100–120 per minute
D. One inflation breath per five compressions
E. Two inflation breaths per five compressions

A

B. Two inflation breaths per 15 compressions

As per current guidelines there are 2 inflation breaths per 15 cheast compressions (B).

option (A) is the adult guidelines, primarily because infants are more likely to suffer respiratory arrest than cardiac so the breaths are more important.

Answer (C) is the protocol once the child has been intubated.

Answers (D) and (E) are nothing at all.

How well did you know this?
1
Not at all
2
3
4
5
Perfectly
18
Q

A 10-year-old child is brought in by ambulance with seizure activity. His mother
reports it starting 30 minutes ago in his right arm and quickly became generalized
tonic clonic jerking. She gave him his buccal midazolam after the first 5 minutes
and called an ambulance when he did not respond after another 5 minutes.
The ambulance crew gave him rectal diazepam on arrival at 15 minutes into
the seizure. He is receiving high flow oxygen via a face mask and continues to
convulse. The mother tells you that he was weaned from his long-term seizure
medication, phenytoin, 2 weeks ago and that he has had a cold for the past 2 days. What is the next step in management?

A. Gain intravenous or intraosseous access and administer lorazepam
B. Gain intravenous or intraosseous access and administer ceftriaxone
C. Repeat the rectal diazepam
D. Gain intravenous or intraosseous access and start a phenytoin infusion
E. Gain intravenous or intraosseous access and start a phenobarbital
infusion

A

D. Gain intravenous or intraosseous access and start a phenytoin infusion

The guidelines are that a child in status elepticus (seizure longer than 30 mins) should recieve two dose of benzodiazipines from; Buccal midazolam, Rectal Diazepam and IV Lorazepam. Once two of these have been given to no effect, as in this case, the next stage in management should be started. The next stage here is to give IV Phenytoin (D) to arrest the seizure.

Further use of benzodiazipines (A or C) should be avoided due to risk of respiratory depression

Phenobarbital (E) is the second line drug after phenytoin, it would be given if he was already on phenytoin at the time of the seizure, which he is not.

Ceftriaxone (B) may help if the seizures were due to encephalitits, but this is not first line treatment.

How well did you know this?
1
Not at all
2
3
4
5
Perfectly
19
Q

A 3-year-old boy is brought in by ambulance fitting. You are assigned to get
the history from the father. Harry is normally fit and well with no significant
past medical history or allergies. He is up to date with his immunizations and
has been growing and developing normally. His behaviour has been difficult
for the past 2 weeks since the birth of his little sister. Mum has been unwell as
she developed HELLP syndrome and was in hospital for a week following the
delivery. Yesterday, he was quite unwell with a tummy bug, vomiting and had
black diarrhoea. That evening they found a mess he had made in the bathroom
with all of his mum’s things strewn over the floor including her tablets from the
hospital. By that time, Harry was getting better so they did not think anything
of it. Today he has been acting strangely and has been difficult to understand,
he then became lethargic at about 4 pm and started fitting 15 minutes ago. What
is the most likely diagnosis?

A. Paracetamol overdose
B. Aspirin overdose
C. Tricyclic antidepressant overdose
D. Bleach intoxication
E. Iron overdose

A

E. Iron overdose

The clue that the mother is diagnosed with HELLP syndrome tells you that she has had anaemia, and so will have been prescribed iron supplementation. The classic course of iron overdose (E) is a two stage illness where there is an initial gastric irritation, possibly with haematemesis and malena (as seen here). There is then a period of up to 24hrs where there is improvement, followed by a deterioration with liver failure, drowsiness and coma. The liver failiure can produce can produce the hyoglycaemia and seizures seen here.

Significant Paracetamol overdose (A) is not common in small children, as the tablets are difficult to swallow.You would expect teh liver failure after 3-5 days, there would also not be the malena.

Aspirin overdose (B) would be much quicker, with signs of metabolic acidosis, Symptoms include dizzyness, confusion, nausea, vomiting, tinnitus, abdominal pain and seizures.

Tricyclic overdose (C) would present much quicker with tachycardia, anticholinergic symptoms, shock and then seizures.

Bleach intoxification (D) is possible, but in this age group it is rare as it tastes foul. There tends to be localised lesions of the mucous membranes where the bleach made contact.

How well did you know this?
1
Not at all
2
3
4
5
Perfectly
20
Q

A 6-year-old boy with a history of asthma and eczema is brought in to accident
and emergency from a local restaurant. He is on high flow facial oxygen with
significant facial oedema and generalized erythema. On auscultation there is
widespread wheeze for which the ambulance crew gave a salbutamol nebulizer.
What is the next step in management?

A. Insert an IV line and give 10 mg slow intravenous antihistamine
B. Insert an IV line and give 100 mg slow intravenous hydrocortisone
C. Insert an IV line and give 200 μg of 1:10 000 intravenous adrenaline
D. Give intramuscular 1:1000 adrenaline, 250 μg
E. Repeat the salbutamol nebulizer and call for an anaesthetist for
intubation

A

D. Give intramuscular 1:1000 adrenaline, 250 μg

It is important to recognise that the facial oedema and generalised erythema here is indicating an anyphlactic reaction. The next step in the management is to give IM adrenaline (D) to drive the interstitial fluid back into the vasculature to reduce the airway swelling.

The dosage in (C) is for cardiac arrest, so is not appropriate.

(A) here will also help to block the histamine release which is driving the angioedema. (B) will reduce the inflammation and help to prevent any delayed hypersensitivity (type IV) reaction, which is something that needs to be covered.

You would also be putting in an emergency call for an anaesthetist.

BUT the first thing here is to crack on with the adrenaline.

How well did you know this?
1
Not at all
2
3
4
5
Perfectly
21
Q

A newborn baby is born to non-consanguineous parents. She is noted to have
puffy feet on her 1st day check. She weighs 2.0 kg with widely spaced nipples and
absent femoral pulses. You have asked your registrar to review her as you think
she may have Turner’s syndrome. She agrees and asks you to send blood tests for
karyotyping. Which is the chromosomal diagnosis of Turner’s syndrome?

A. 47XXY
B. 45YO
C. 46XY
D. 46XX
E. 45XO

A

E. 45XO

Turner’s syndrome (E) is the loss of the 46th cromosome leading to femal infants with the syndromic features seen here as well as cardiac abnormalites, horseshoe kidney, coarctation of the aorta, short stature,coeliac, delayed puberty and thyroid disorders,

(A) is Klinefelter’s, they are phenotypically male with tall stature, delayed pubery, and gynaecomastia.

(B) I believe this would be incompatible with life

(C) normal male

(D) normal female

How well did you know this?
1
Not at all
2
3
4
5
Perfectly
22
Q

A 15-year-old boy was diagnosed with Down’s syndrome at birth. He is short for
his age, had cardiac surgery as a baby, has treatment for hypothyroidism and now
attends mainstream school with some support. His parents are enquiring now about what complications he faces. Which of these is not a recognized complication of Down’s syndrome?

A. Retinoblastoma
B. Atrioventricular septal defect (AVSD)
C. Type 1 diabetes
D. Leukaemia
E. Alzheimer’s disease

A

A. Retinoblastoma

Associations of Down’s syndrome:

Cardiac: AVSD, Tetralogy of Fallot

Endocrine: Hypothyroid, Addisons, DM type I

Occular: Cataracts, But NOT retinoblastoma (A)

Malignancy: Leukaemia

GI: Duodenal atresia, Hirschprung’s disease

Musckuloskeletal: Atlanto-axial instability

Neuro: Alzheimer’s disease

How well did you know this?
1
Not at all
2
3
4
5
Perfectly
23
Q

A baby is born and you are asked to do the baby check at 6 hours post-natal
age. You go to see the baby and mum states that he has not yet had a feed. You
advise they stay in hospital until the feeding is established. This is the first child of
non-consanguineous parents. On day 4 when you review the baby he has still not
had an adequate intake, has lost over 10 per cent in birth weight and is markedly
hypotonic. Your consultant asks you to request genetic testing for Prader–Will
syndrome. What is the inheritance of Prader–Willi syndrome?

A. X-linked
B. Imprinting
C. Monosomy
D. Microdeletion
E. Trisomy

A

B. Imprinting

This is because for some of your chromosomes to function correctly, you need a paternal or maternal copy to be intact. In Prader-Willi there is the loss of part of the paternal chromosome 15. The loss of the maternal chromosome 15 leads to Angelman’s syndrome

X-linked (A) conditions include; Duchene’s MD, Fragile X syndrome

Monosomy (C) includes Turner’s syndrome

Microdeletions (D) are seen in DiGeorge’s syndrome and William’s syndrome.

Trisomy (E) conditions include: Down’s (21), Patau’s (13) and Edward’s (18) syndromes.

How well did you know this?
1
Not at all
2
3
4
5
Perfectly
24
Q

A 5-day-old baby who is formula fed is on the neonatal unit being treated for sepsis secondary to an Escherichia coli urinary tract infection. He has been on antibiotics for 5 days. He is still unwell and vomiting. The parents are consanguineous and this is their first child. He has had repeat blood and urine cultures taken. Urine reducing substances are positive. What is the most likely underlying diagnosis?

A. Fructose intolerance
B. Galactosaemia
C. Phenylketonuria
D. Lactose intolerance
E. Glycogen storage disease

A

B. Galactosaemia

Galactosaemia (A) is a deficiency in galactose-1-phosphate uridyl transferase. It manifests with the consumption of lactose-containing milks, with vomiting, cataracts and recurrent episodes of E.coli sepsis.

Fructose intolerance (A) can present similarly but the e.coli sepsis is a giveaway that this is Galactosaemia.

PKU (C) presents as a developmental delay and ‘musty’ smelling urine. It doesn’t present as acute sepsis, it is also screened for in the guthrie test.

How well did you know this?
1
Not at all
2
3
4
5
Perfectly
25
Q

A 10-year-old boy is brought to the GP with tall stature. He is taller than his peers at school. His arm span is greater than his height, he has long, thin fingers, scoliosis and pectus excavatum. He is also concerned that he gets short of breath at school during PE lessons. You refer him for an echocardiogram and chest x-ray. You make a clinical diagnosis of Marfan’s syndrome. What is the inheritance of Marfan’s syndrome?

A. X-linked recessive
B. Autosomal recessive
C. Sporadic
D. X-linked dominant
E. Autosomal dominant

A

E. Autosomal dominant

Marfan’s is an autosomal dominant connective tissue disorder affecting the fibrillin gene.

A. X-linked recessive - Duchenne’s and Becker’s muscular dystrophy
B. Autosomal recessive - Homocysteinuria (similar phenotype but with thromboembolic tendancy)
C. Sporadic - Klinefelter’s
D. X-linked dominant - Hypophosphataemic rickets

How well did you know this?
1
Not at all
2
3
4
5
Perfectly
26
Q

A pregnant woman seeks advice from you regarding her condition and its impact
on the pregnancy and risk to the baby. She has phenylketonuria (PKU) and has
been on a phenylalanine-free diet for life. She was told that it was very important
during her pregnancy to be compliant with this diet. She would like to know
how the baby will be tested for the condition as she is aware that is an inherited
condition. What is the initial investigation you will advise?

A. Serum tyrosine levels
B. Genetic screening
C. Serum phenylalanine levels
D. Urine phenylketones
E. Newborn blood spot screening

A

E. Newborn blood spot screening

PKU is a autosomal recessive metbolic condition leading to a defect in the phenylalanie hydroxylase enzyme. This enzyme converts phenylalanie to tyrosine, unrecognised PKU can lead to seizures, musty urine and eventually microcephaly and learning difficulties. PKU is one of the six inherited metabolic disorders already on the UK newborn screening test (E).

Options (A), (C) and (D) would all be useful for monitoring the disease, but are not diagnostic.

(B) is not used for PKU

How well did you know this?
1
Not at all
2
3
4
5
Perfectly
27
Q

You see an 18-year-old boy who is the first child of his African parents and was
born in Kenya before moving to the UK 1 year ago. He has white skin and pink
irises. He was diagnosed with oculocutaneous albinism at birth. He has difficulty
with his sight but has recently developed a skin lesion on his face. His mother has
brought him to his GP as it has recently started to increase in size. On examination
you note is an elevated, 3 cm diameter lump on the left of his nose. It has irregular
edges, is firm and immobile and pigmented in areas. What is the likely diagnosis?

A. Benign naevus
B. Scar from a healing wound
C. Malignant tumour
D. Abscess
E. Wart

A

C. Malignant tumour

His background of albinism means that he has problems synthesising melanin, this makes him more at risk of solar induced genetic damage. Malignancy shouldalways be considered in these patients, especially as he has lived at an equitorial latitude for 17 years. The description of a lump that is increasing in size, with irregular edges, with multiple colours and is solid, are all alarming for malignancy (C). This certainly does not sound like a benign naevus (A)!

There is nothing in this history that suggests trauma (B) or infection (D).

A wart (E) due to HPV infection would be possible, but the location on the face goes against this, and makes malignancy more likely.

How well did you know this?
1
Not at all
2
3
4
5
Perfectly
28
Q

A 20-month-old boy has been referred due to delayed walking. On further
questioning you establish he has no difficulty feeding, had head control at 3
months of age, and sat up by 8 months. He has been crawling for the last 8
months, but he does not pull to stand or walk with support. He has no dysmorphic
features. There is no known family history of muscle problems. His mother has no
myotonia. His mother is very concerned and asks you what is wrong. What is the
most likely diagnosis?

A. Myotonic dystrophy
B. Duchenne’s muscular dystrophy
C. Down’s syndrome
D. Myasthenia gravis
E. Becker’s muscular dystrophy

A

B. Duchenne’s muscular dystrophy

Of these conditions (B) s the most common, with Becker’s (E) being a a similar but more mild presentation. Both are X-linked conditions.

Myotonic Dystrophy (A) is autosomal dominant, so it would also be present in the mother.

Down’s (C) has a variety of syndromic features; Facies (low set ears, epicanthic folds, protruding tounge, flattened nasal bridge), single palmar crease, sandal gap toes.

Myasthenia Gravis (D) can present with delayed walking, but the key finding is fatigability, which isn’t demonstrated here.

How well did you know this?
1
Not at all
2
3
4
5
Perfectly
29
Q

You are asked to see a 3-day-old baby on the post-natal ward. The baby was born
at term and is the first child of consanguineous parents. The baby is drowsy and
vomiting, with no fever, rash or diarrhoea. On examination, the baby is noted to
have ambiguous genitalia. You do some blood tests: white cell count 5 × 109/L,
C-reactive protein 2 mg/L, Na+ 125 mmol/L, K+ 8 mmol/L, glucose 1.7 mmol/L.
17-OH level progesterone is low. You make a diagnosis of congenital adrenal
hyperplasia. What is the best initial management plan?

A. IV hydrocortisone
B. IV dextrose
C. IV dextrose and IV hydrocortisone
D. IV 0.9 per cent saline
E. IV 3 per cent saline and IV hydrocortisone

A

C. IV dextrose and IV hydrocortisone

Congenital adrenal hyperplasia is a deficiency in the enzymes which convert testosterone to cortisol. This accumulates and leads to the virilisation in female infants, and the abnormal male genitalia.

This lack of cortisol leads to a salt-losing crisis, as seen here, with hypoglaceima and vomiting. Replacement of the steroids and the correction of the glucose (C) is the first line management here. Either option in isolation, (A) or (B), is not enough.

Correction of the Hyponatremia needs to be gradual (over 2 days) in order to avoid central pontine demyelination, (E) is far too dangerous. Normal saline (D) will not really do anything for this child.

How well did you know this?
1
Not at all
2
3
4
5
Perfectly
30
Q

A 10-year-old boy is brought to the paediatric outpatient department for a review
of his height. He was found to be on the 0.4th centile and his mid-parental
height is the 98th centile. He also has widely spaced nipples, wide carrying angle,
hypogonadism, pulmonary stenosis and developmental delay. What is the most
likely diagnosis?

A. Angelman’s syndrome
B. Williams’s syndrome
C. Turner’s syndrome
D. Prader–Willi syndrome
E. Noonan’s syndrome

A

E. Noonan’s syndrome

Nonan’s (E) has a similar phenotype to Turner’s (C) but, unlike turners, affects males as well as females.

This case does not present the typical features of the other syndromes given as answer options.

(A) Happy demenour and developmental delay

(B) ‘Elfin’ facial appearance, highly sociable but with developmental delay in other areas, aortic stenosis.

(D) Poor feeding and weight gain in infancy, hyperphagia and obesity in later life.

How well did you know this?
1
Not at all
2
3
4
5
Perfectly
31
Q

A 2-week-old baby was referred to the prolonged jaundice clinic by the
community midwife. The pregnancy was unremarkable, and she was born at term
with no antenatal abnormalities on ultrasound (US) scans or blood serology. She
is now 17 days old and has been jaundiced since day 5 of life and never required
phototherapy. She is breastfed and feeds 3-hourly for 20–25 minutes. She is
afebrile and not lethargic. Her mother reports that the stools are pale and she
has dark coloured urine. The bilirubin is 300 μmol/L, and conjugated bilirubin
100 μmol/L. What is the most important diagnosis to exclude?

A. Breast milk jaundice
B. ABO incompatibility
C. Biliary atresia
D. Neonatal hepatitis
E. Hypothyroidism

A

C. Biliary atresia

Jaundice is an important presenting complaint in neonates. The key to determining the cause is the timing, duration and onset of the jaundice.

Jaundice in the first 24hrs may be due to immune (ABO, or Rhesus factors), or inherited non-immune conditions (Sphereocytosis or G6PD deficiency). The jaundice in these cases is always unconjugated, so option (B) is wrong.

Breast milk jaundice (A) is physiological and appears between 24hrs and 2 weeks, but shouldn’t persist.

Hypothyroidism (E) is important to identify, due to the long term learning difficulties that can arise. It doesn;t present with pale stools, so is unlikely here, and ti should have been detected on the blood spot test.

Neonatal hepatitis (D) can be secondary to congenital infections, or metabolic diseases such as alpa1 anti-trypsin, or cystic fibrosis.

However, the history of dark urine and pale stools s strongly suggestive of biliary atresia (C). This needs to be investigated, as there is a risk of developing cirrhosis.

How well did you know this?
1
Not at all
2
3
4
5
Perfectly
32
Q

A 1-day-old baby is on the post-natal ward. You are asked to review her as she is
febrile and lethargic. On examination she is tachycardic, has a capillary refill time
of 3 seconds centrally and reduced urine output. Her blood culture 24 hours later
grows Gram-positive cocci. Which is the most likely causative organism?

A. Streptococcus pneumoniae
B. Staphylococcus aureus
C. Group B Streptococcus
D. Streptococcus viridans
E. Group A Streptococcus

A

C. Group B Streptococcus

Group B streptococcus (C), GBS, can colonise the reproductive tract, and is a common congenital infection. Symptoms include; sepsis, pneumonia, meningitis, UTI, and septic arthritis.

All the others can cause sepsis but the three most common congenital infections are; GBS, E.Coli, and Listeria monocytogenes.

How well did you know this?
1
Not at all
2
3
4
5
Perfectly
33
Q

A preterm baby is born at 25 + 6 weeks gestation. He is delivered by caesarean section due to maternal pre-eclampsia. He is intubated at birth and given surfactant via the endotracheal tube. He is ventilated and commenced on IV dextrose. After 4 hours of age he has increased work of breathing, with intercostal and subcostal recession and a respiratory rate of 60/min. A chest x-ray shows a ground glass pattern in both lung fields. He has no audible murmur. He is afebrile. You diagnose respiratory distress syndrome. What is the aetiological factor responsible for respiratory distress syndrome?

A. Pneumonitis
B. Lung hypoplasia
C. Surfactant deficiency
D. Immature lung parenchyma
E. Infection with group B Streptococcus

A

C. Surfactant deficiency

The respiratory distress syndrome here is due to immature type II pneumocytes producing insufficient surfactant (C). This baby is vulnerable to this because of it’s prematurity. The description of ‘ground glass’ pattern is charecteristic of respiratory distress syndrome

of the other options presented here, (A) is present in meconium aspiration syndrome due to irritation of teh lungs by the compounds in the meconium.

(B) is due to a develpmental issue with the lung, secondary to renal issues or diaphragmatic hernia.

(D) isn’t an issue as the lungs are structualy immature, but functional

Any infection (E) would present as increase respiratory effort, but would have focal signs on chest radiograph.

How well did you know this?
1
Not at all
2
3
4
5
Perfectly
34
Q

A preterm baby is now 25 + 7 weeks corrected gestation. He is on the neonatal
unit being cared for while his mother recovers on ITU after he was born secondary
to an eclamptic seizure. He has been receiving formula milk as the parents have
not consented to donor breast milk. He has been having bilious aspirates from
his nasogastric tube and today his abdomen in very distended and tense. He has
had one episode of bloody stools. You are going to treat him for nectrotizing
enterocolitis (NEC). What is the best initial management plan?

A. Conservative management, observe and reassess
B. Nil by mouth (NBM), IV antibiotics and emergency exploratory
laparotomy
C. IV fluids, emergency laparotomy and bowel resection
D. IV fluids and IV antibiotics
E. NBM, IV fluids, abdominal X-ray and surgical review

A

E. NBM, IV fluids, abdominal X-ray and surgical review

NEC is a complex disease with a multitude of risk factors, we see here some of the risks; formula feeding, prematurity and (possibly) ischaemia at birth. This child is showing signs of bowel pathology and so the (already high) suspiscion of NEC is supported.

Active management of this condition is called for, option (A) is not the correct management here.

As with all surgial pathology, the patient should be made NBM and given IV fluids. Option (D) doesn’t account for the need of NBM here.

Options (B) and (C) completely bypass the step of acually imaging the patient and getting a surgical opinion prior to undergoing surgery. Without first gaining a surgical review, I’m not sure who will be carring out this surgery… The F1?

How well did you know this?
1
Not at all
2
3
4
5
Perfectly
35
Q

A 3-day-old baby is seen by the midwife for a routine post-natal review. She
notices that he is very floppy and his mother has raised concerns about his poor
feeding. He has a protruding tongue, epicanthic folds, low set ears and sandal gap
toes. She explains to the parents she thinks he may have Down’s syndrome and
refers him to the paediatrician. What is the diagnostic test for Down’s syndrome?

A. Serum alpha fetoprotein, beta human chorionic gonadotrophin,
oestriol, inhibin
B. Gene mutation analysis
C. Clinical diagnosis
D. Karyotype
E. FISH

A

D. Karyotype

Down’s syndrome is due to trisomy 21 so a karyotype test (D) is all that is needed to identify the additional chromosome.

Screening for Down’s during pregnancy uses the combined test, utilising serum markers and an ultrasound scan. The markers used are as in (A).

It is a trisomy anomolity so there is no gene mutation to find (B). That would be used for a condition such as cystic fibrosis.

Although syndromic conditions can be identified on the clinical picture (C) a true diagnosis needs something more objective!

Fluorescence in situ hybridisation (E) is used to identify micro deletions, as in Digeorge’s or Cri du Chat.

How well did you know this?
1
Not at all
2
3
4
5
Perfectly
36
Q

A baby is born by emergency caesarean section due to fetal tachycardia. His delivery was uneventful and you are asked to see him 5 hours later on the post-natal ward. He has just taken his first feed and has been coughing and spluttering since. He had an episode with blue lips transiently and this has now improved; his oxygen saturations are 97 per cent in air and he is apyrexial. On examination, you note other features including vertebral and limb abnormalities, imperforate anus,
pansystolic murmur at the lower left sternal edge and renal anomalies noted on
antenatal scans. What is the most likely cause for his coughing episode?

A. Cleft palate
B. Tracheoesophageal fistula
C. Choanal atresia
D. Incoordinated swallowing reflex
E. Pneumonia

A

B. Tracheoesophageal fistula

All of the answers offered here can result in a coughing baby , but in this case there are a constellation of symptoms consistent with one overiding syndrome; VACTERAL. This stands for Vertebral, anal imperforation, cardiac, tracheo-(o)esophageal fistula, renal and limb abnormalities. With this in mind (B) is the correct answer here.

Cleft palate (A) is often seen in DiGeorge’s or Down’s syndrome.

Choanl atresia (C) is associated with CHARGE (coloboma, heart defects, atresia choanae,retardation of growth and development, genitourinary abnormalities and ear anomolies) syndrome.

An incoordinated swallow (D) would be expected if there was an underlying neurological or neuromuscular pathology. There is nothing in this history to suggest that.

The child is afebrile so a pneumonia (E) is unlikely to be the cause of thier problems.

How well did you know this?
1
Not at all
2
3
4
5
Perfectly
37
Q

A term baby is awaiting his discharge check when you are called to see him at
10 hours of age. His mother reports that he has turned a dusky colour and is not
as alert as he has been. On examination he has central cyanosis, pulse 150 bpm
regular, and both brachial and femoral pulses are palpable. He has normal heart
sounds with no murmur. His oxygen saturations are 65 per cent in air. What is the
most likely underlying diagnosis?

A. Transposition of the great vessels
B. Ventricular septal defect (VSD)
C. Tetralogy of Fallot
D. Aortic stenosis
E. Coartation of the aorta

A

A. Transposition of the great vessels

This baby is presenting with a cyanotic defect, of the answers here only (A) and Tetralogy of Fallot (C) are cyanotic heart conditions. The Tetralogy of Fallot includes; right ventricular outflow obstruction, Ventral-septal defect, overiding aorta and right ventricular hypertrophy

The differentiator here is that Tetralogy presents more commonly around 6 months of age, not immediately post birth. Transposition of the great vessels (A), where the aorta is connected to the right ventricle and the pulmonary artery is connected to teh left ventricle, is more likely to present at birth. The only thing keeping this child alive at this point is a left-right shunt, such as a VSD, ASD, or his patent ductus arteriosus (which will be closing). He needs a prostaglandin infusion to keep it patent and urgent surgical correction.

All the other options here, (B) (D) and (E), would not cause cyanotic disease.

How well did you know this?
1
Not at all
2
3
4
5
Perfectly
38
Q

A 12-hour-old baby on the post-natal ward has just had a seizure lasting
2 minutes. It resolved spontaneously and was generalized in nature. Her mother
had gestational diabetes and poor glucose control in pregnancy. The baby’s birth
weight was 5 kg. There were no abnormalities noted on antenatal US scans or
maternal serology. On examination she has no dysmorphic features and handles
well. What initial blood tests would you do for the baby?

A. Liver function tests
B. Boehringer Mannheim (BM) glucose
C. Full blood count, C-reactive protein
D. Electrolytes
E. Calcium, magnesium

A

B. Boehringer Mannheim (BM) glucose

This is an infant of a diabetic mother, she is displaying signs that the poor control in pregnancy has had an adverse effect, she is macrosomic at 5kg.

Poor control of glucose at the time of conception predisposes the fetus to congenital malformations, and macrosomia and hypoglycaemia post-natally.

This baby has been exposed to a hyperglycaemic enviroment during pregnancy and this has led to an increased baseline insulin level in the child. With the birth this has now suddenly changed and that high insulin is innappropriate, she has become profoundly hypoglycaemic and has started seizing. A BM test (B) is needed to confirm.

Infection can lead to seizures and (C) would be your test, electrolyte disturbances (A, D, E) would also cause seizures, but all of these are less likely than hypoglycaemia given the history of this case. In the event of a normal blood sugar in the infant then, of course, you would proceed to these tests.

How well did you know this?
1
Not at all
2
3
4
5
Perfectly
39
Q

A pregnant woman is admitted to the labour ward for an elective caesarean
section at 38 weeks for her baby who had an antenatal diagnosis of gastroschisis.
The paediatric team are called to attend the delivery. The baby is born in good
condition with no resuscitation required. He is taken to the neonatal unit for
further care. Which of these is a complication of gastroschisis?

A. Dehydration
B. Hyperthermia
C. Necrotizing enterocolitis (NEC)
D. Fluid overload
E. Hypernatraemia

A

A. Dehydration

Gastroschisis is a condition where the bowel herniates throught eh anterior abdominal wall during fetal development. The bowel is not covered in a membrane, as in the similar condion of exomphalos, and so this can lead to fluid, electrolyte and heat losses.

Gastroschisis is rarely associated with other conditions. the baby will require fluid replacement and the strict monitoring of fluid balance, they will also need to be in a heated incubator and have regular electrolyte monitoring.

Dehydration (A) is a major concern in these patients, there is no increased risk of overload (D). As discussed above hypothermia is a risk, not hyperthermia (B). Loss of electrolytes makes hyponatraemia likely, but not hypernataemia (E).

Finally NEC (C) is no more common in this group than others.

How well did you know this?
1
Not at all
2
3
4
5
Perfectly
40
Q

You are called to see a baby who has just been born at 39 weeks’ gestation, as
the midwife thinks he is small and should be admitted to the neonatal unit for his
care. You review the baby. His weight is 1.8 kg, below the 0.4th centile and his
head circumference is 35 cm – 50th centile. He has no dysmorphic features. Which is the most likely cause of this IUGR?

A. Chromosomal anomaly
B. Maternal smoking
C. Congenital infection
D. Maternal alcohol use
E. Placental insufficiency

A

E. Placental insufficiency

Placental insufficiency (E) classically presents as asymetrical IUGR, we see this here with the disparity between the centiles of the infants head and weight. common causes of placental insufficiency include vascular compromise in the placenta on a background of maternal diabetes or pre-eclampsia.

This is because the placenta is able to supply the fetus’ needs up to a point, and in the later stages of the pregnancy can no longer sufficiently perfuse the fetus. This accounts for the fact that the head circumfirence is less affected by the growth restriction.

all of the other conditions here would be expected to result in symetrical growth restriction as the situation would arise in early pregnancy and endure. It is possible to see this picture of asymetric growth restriction due to smoking (B), but it is not the most likely case.

How well did you know this?
1
Not at all
2
3
4
5
Perfectly
41
Q

A mother brings her 4-week-old baby to see you for the third time. He was born at
term by normal vaginal delivery with no complications. You started him on antireflux medicine last week but it has not helped. He is now vomiting his whole feeds and is becoming lethargic and passing less urine and stool. His mother says he is hungry even after he vomits. The practice nurse has weighed him and he has lost 200 g since last week. His mother was breastfeeding him while waiting to be seen and as you go to examine him, the baby has a large milky vomit, which cascades over the clinic floor. What is the most likely diagnosis?

A. Gastroenteritis
B. Volvulus
C. Necrotizing enterocolitis (NEC)
D. Intussusception
E. Pyloric stenosis

A

E. Pyloric stenosis

This is a classic presentation of pyloric stenosis (E). It presents at 1 month with projectile vomiting, more common in males. The cause is hypertrophy of the pyloric muscle at the gastric outflow, this leads to delayed emptying and increasing vomiting. The description of being still hungry is also typical of the condition.

Volvulus (B) and intussesception (D) would both present as a surgical abdomen, with tenderness, distention and generally being an irritable baby.

NEC (C) is generally a disease of the premature infant, with formula feeding another risk (the case her specifies breastfed). Other signs of NEC are that theh infant will have bile stained gastric aspirates and abdominal distention. It generally presents in term infants who have had a history of birth asphyxia or severe growth restriction.

Gastroenteritiis (A) can occur in young infants, the risk here is in parents who don’t sterilise their bottles. The presentation is a shorter history of vomiting and diarrhoea.

How well did you know this?
1
Not at all
2
3
4
5
Perfectly
42
Q

A 15-month-old girl has come to see you with her father. The family are worried
that she has had diarrhoea for more than a month, occasional vomiting and is
losing weight. She used to be a happy interactive baby but now seems lethargic
and miserable most of the time. She has no significant past medical history, the
rest of the family are well and there is no history of travel. Her mother has well controlled type 1 diabetes. The child’s weight at 6 months in the personal child
health record (‘red book’) was on the 50th centile but she is now just below the
9th. What is the most likely diagnosis?

A. Crohn’s disease
B. Ulcerative colitis
C. Coeliac disease
D. Irritable bowel syndrome
E. Giardiasi

A

C. Coeliac disease

The history of autoimmunity in the mother gives you a cule here. The problems here have only started after weaning so that also gives you a clue that this is dietary induced. The answer here is Coelic (C), it is an anutoimmune condition resulting in sensitiity to dietary gluten, leading to atrophy of the small intestinal villi. Children present with abdominal bloating, diarrhoea, failure to thrive and wasting (classically affecting the gluteal muscles).

Inflammatory bowel disease (A) and (B) are rare in younger children, they tend to start in teenagers.

IBS (D) is rare in a very young child as it tends to have a degree of stress involved in it’s onset. It is a diagnosis of exclusion, and the weight loss is less likely in IBS.

Giardiasis (E) would present like this, but it would be unlikely without a travel history to an endemic area. This is made more unlikely by the fact that noone else in the family is unwell.

How well did you know this?
1
Not at all
2
3
4
5
Perfectly
43
Q

A 13-year-boy is brought to see you as he has recently been complaining of
abdominal pain and is increasingly tired. On examination you note some early
clubbing and erythematous palms. His conjunctivae look pale. He has one or
two spider naevi on his chest. His abdomen is soft with mild tenderness in the
epigastrium and right upper quadrant. The liver is palpable at 1 cm and you feel the splenic tip. He has normal bowel sounds and no bruits. On slit lamp examination of his eyes, an amber ring is noted around the cornea. What is the most likely diagnosis?

A. Abdominal tuberculosis
B. Cystic fibrosis
C. Wilson’s disease
D. Acute hepatitis A
E. Glandular fever

A

C. Wilson’s disease

This child is showing signs of chronic liver failure, he has clubbing, palmar erythema and non-significant spider naevi (5+ is significant).

The slit lamp examination has demonstrated Kayser-Fleischer rings, these are copper deposits around the cornea, and are pathognomic for Wilson’s disease (C).

Wilson’s is an autosomal recessive condition caused by a defect in the metabolism of coper, this leads to copper being deposited in the tssues and leads to liver failure with cirrhosis, neurological problems, as well as renal and cardiac problems. There is also an association with heamolytic anaemias due to cper being deposited in teh red cell membranes. .

Abdominal Turberculosis (A) would not cause the rampant liver failure we see in this case.

Cystic Fibrosis (B) would cause the clbbing we see here, but there is no information of pulmonary findings.

Hep A (D) would not cause this chronic liver failure, the splenic enlargement also doesn’t fit, and there is no jaundice here.

Glandular fever (E), or Epstein Barr virus, would not show signs of chronic liver failure.

The overiding factor here are the finding of the Kayser-Fleischer rings, these are only present in Wilson’s Disease.

How well did you know this?
1
Not at all
2
3
4
5
Perfectly
44
Q

An 8-year-old girl is brought to see you, having not opened her bowels in 8 days. She complains of hard painful stools and recurrent abdominal pain for the past 6 months but no vomiting. Her mother thinks that she is avoiding going to the toilet and reports that she has always been a bit irregular opening her bowels, averaging about twice a week. In her past medical history, she passed meconium on day 1 of life and has had no significant medical problems. On examination she is a well-looking, normally grown child. Her abdomen is soft with a palpable indentable mass in the left iliac fossa. The anus is normal, as are her lower limbs. What is the first step in management?

A. Encourage her to increase her fluid intake, dietary fibre and exercise
B. Introduce scheduled toileting with a positive reward scheme such as a
star chart
C. Refer for bowel disimpaction under anaesthesia
D. Start polyethylene glycol with electrolytes such as Movicol
E. Start a stimulant laxative such as senna

A

A. Encourage her to increase her fluid intake, dietary fibre and exercise

This is a typical presentation of constipation, and the indentable mass in the L.Iliac fossa is impacted faeces. The abscence of, growth failure, delayed meconium passage, distended abdomen and vomiting, neuro signs or anal pathology, are all reassuring that this is not serious pathology.

The correct first line management here is (A) as she is impacted at this stage and needs some help to soften the stool. Once this has been achieved we can look at behavioural changes (B) to prevent recurrence of this situation.

If the lifestyle changes do not produce softer stols, then a non stimulant laxitive (D) may be needed to medicaly disimpact the bowel.

Stimulant laxitives (E) on a background of impacted faeces are likelt to casue more abdominal pain, so should be avoided as there is also a risk of bowel perforation and dependance forming.

Surgical disimpaction (C) would be the absolute last step after all conservative and medical options had failed. It would also be necessary to rule out all potential underlying causes of constipation first.

How well did you know this?
1
Not at all
2
3
4
5
Perfectly
45
Q

A 2-week-old baby is brought to accident and emergency by his parents because he has been intermittently inconsolable for the past 12 hours. He does not want to breastfeed and has vomited. The parents think his tummy is upset as he keeps drawing up his legs. He was born at term by normal vaginal delivery with no problems. On examination the abdomen is distended and tense. He is crying and there is a firm swelling in the right groin area. You can hear active bowel sounds. What is the most likely diagnosis?

A. Appendicitis
B. Right inguinal hernia
C. Gastroenteritis
D. NEC
E. Sepsis

A

B. Right inguinal hernia

the examination findings of a groin swelling and a tense abdomen give you an indication of the likely pathology here. The baby is also vomiting, but you hear bowel sounds, this tells you that there is a partial bowel obstruction. Putting the picture together we see that this must be an incarcerated Inguinal hernia (B). He need to be seen by the surgeons and this needs correcting before it becomes a strangulated hernia.

Appendicitis (A) is common in the ages of 11-20, but very unlikely in an infant as the relative size of the appendix inlet compared to the rest of the bowel lumen doesn’t predispose it to obstruction.

Gastroenteritis (C) can occur in infants, usually if the parents are not sterilising the bottles and water for the feeds correctly. The history here is a bit more substantial, and the groin swelling goes against this.

NEC (D) is usually seen in premature babies, or those who have suffered birth hypoxia or severe growth restriction. Abdominal distention and bile stained gastric aspirates are the hallmarks of NEC.

If this was a more ambigious history and lacking in abdominal signs then you may well treat with IV antibiotics for a neonatal sepsis (E).

Of note here; any inguinal hernia mandates a surgical review, c.f. umbilical hernias will self resolve without surgical intervention.

How well did you know this?
1
Not at all
2
3
4
5
Perfectly
46
Q

A 5-year-old girl is brought to accident and emergency with a 24-hour history of
vomiting and diarrhoea and now her eyes and skin have gone very yellow. She has been taking oral rehydration salts and is still passing urine. She is normally healthy and there is no family history of jaundice. On examination her heart rate is 130 and respiratory rate is 26. She is alert, warm and well perfused. The chest is clear, heart sounds are normal and the abdomen is soft with a 2 cm liver edge. What should the management be?

A. Reassure and discharge home, to return if not keeping fluids down
B. Take bloods to test for liver function, hepatitis, and urea and
electrolytes; inform the Health Protection Agency and discharge home
with follow-up to review results
C. Take bloods to test for liver function, hepatitis screen and urea and
electrolytes and admit for IV fluids
D. Take bloods to test for liver function, hepatitis screen and urea and
electrolytes and admit for observation with continued oral rehydration
salts
E. Take bloods to check liver function and urea and electrolytes. If they
are normal, discharge home with reassurance but to return if not
keeping fluids down

A

B. Take bloods to test for liver function, hepatitis, and urea and
electrolytes; inform the Health Protection Agency and discharge home
with follow-up to review results

The most likely cause of this childs jaundice is hepititis A, it is important to know that Hep A is a notifiable disease. There are a lot of them and they can be found here:

https://www.gov.uk/notifiable-diseases-and-causative-organisms-how-to-report

Essentially they are all contagious and serious diseases, the helath professional has a statutory duty to notify the proper authority on suspicion of one of these diseases (you are not to wait for laboratory confirmation).

Knowing this we can straight away exclude all of the answers except (B).

Even without knowing that, the child has a palpable liver and is jaundiced, so investigation is needed, This excludes (A). She is drinking and remaining hydrated, so IV fluids are not needed (C).

(E) doesn’t actually perform a hepatitis screen

(D) admits the patient, which is an infection risk, and doesn’t inform the HPA.

How well did you know this?
1
Not at all
2
3
4
5
Perfectly
47
Q

A 15 year old with well-controlled type 1 diabetes presents with frank
haematemesis. Her blood tests in accident and emergency show: pH 7.37, glucose
18.3 mmol/L, haemoglobin 12.3 g/dL, white cell count 5.3×109/L, neutrophils
2.1×109/L, platelets 165×109/L, Na+ 135 mmol/L, K+ 3.5 mmol/L, urea 5.0 mmol/L,
creatinine 83 μmol/L, alanine transaminase 740 IU/l, bilirubin 96 μmol/L, alkaline
phosphatase 102 IU/l, and albumin 25 g/L. Further investigations once she is
stable on the ward show hepatitis B surface antigen negative, anti-hepatitis C virus
negative, anti-nuclear antibody (ANA) 1:320 and anti-smooth muscle antibodies
are positive. What is the most likely diagnosis?

A. Autoimmune hepatitis with varices
B. Metabolic ketoacidosis
C. Gastroenteritis with a Mallory–Weiss tear
D. Pregnancy with hyperemesis and a Mallory–Weiss tear
E. Systemic lupus erythematosus (SLE)

A

A. Autoimmune hepatitis with varices

It is important here to recognise that type I diabetics can also suffer from other autoimmune conditions. The most commonly occuring with diabetes are Grave’s disease and Coeliac disease, these are screened for on a yealy basis.

Autoimmune Hepatitis (A) is more comon to be found in females and typically presents between 10 and 30, as either a chronic liver failure or an acute hepatitis. The ANA will be positive in around 80% of cases and the Anti-smooth muscle will be positive in 70%.

Ketoacidosis (B) is incorrect as we can see her pH is normal. The raised glucose is a stress responce.

There is no history of diarrhoea, and there are normal amounts of white cells so this makes gastroenteritis (C) unlikely. You would probably still isolate her in a side room to be prudent.

Although there is nothing to suggest pregnancy (D) in the history, it wis always wise to test any woman of childbearing age who has abdominal pathology.

SLE (E) would have a positive ANA, but it would not have a positive anti-smooth muscle. There is nothing else in this history consistent with SLE.

How well did you know this?
1
Not at all
2
3
4
5
Perfectly
48
Q

An 18-month-old child is brought into accident and emergency with a 2-day
history of vomiting, abdominal pain and fever. Which of the following is an
unlikely cause of this clinical picture?

A. Lower lobe pneumonia with pain referred to the abdomen
B. Mesenteric adenitis
C. Diabetic ketoacidosis
D. Pyelonephritis
E. NEC

A

E. NEC

NEC (E) (necrotising enterocolitis) classically affects premature infants, or those with severe birth hypoxia or severe growth restriction. It has an onset in the neonatal period, not in an 18-month old child.

A lower lobe pneumonia (A) may present with abdominal pain and fever, any respiratory tract infection in the young can present with vomiting. The infalamed airways cause a gag reflex, and the increased work of breathing increased the intra-abdominal pressure.

Mesenteric adenitis (B) has this presentation, and is the likely pathology here. It is, however, a diagnosis of exclusion. it is caused by an inflammation of the mesenteric lymph nodes, it can be mistaken for appendicitis.

Diabetes (C) is extremely rare, but not impossible, in a child this young. An illness could cause the condition to manifest for the first time with an episode of ketoacidosis. vomiting would be a typical presenting symptom.

Pyelonephritis (D) is always one to rule out in a child with fever and vomiting.

How well did you know this?
1
Not at all
2
3
4
5
Perfectly
49
Q

A 13 month old is referred up to her local district general accident and emergency
by a GP who is concerned she has intussusception following an 18-hour historyof fever, vomiting and intermittent colicky screaming. A kind radiologist agreed
to do an urgent ultrasound which shows an area of invaginated bowel in the right
side of the colon. What is the most appropriate management?

A. Ask the radiologist to attempt a reduction by rectal air insufflation and
if this fails make nil by mouth (NBM) and transfer to a local paediatric
surgical unit
B. Make NBM and start intravenous fluids while waiting for transfer to a
paediatric surgical unit
C. Move to theatre for an attempt of rectal air insufflation reduction and if
this fails move to surgery in the local hospital as the patient will be too
unstable for transfer
D. Make NBM, start IV fluids and admit for observation
E. Make NBM and start intravenous fluids, and book him onto the
emergency theatre list as he is too unstable for transfer to a local
paediatric surgical unit

A

B. Make NBM and start intravenous fluids while waiting for transfer to a
paediatric surgical unit

Although up to 75% of cases of intussuception can be corrected by insufflation, those that fail carry a risk of perforation. It is for this reasont that it should not be attempted without suitably equiped paediatric surgical facilities available. We see here that you have been specifically told that you are at a district general hospital, so the author wants you to recognise that you will not have the facilities for surgery on a 13-month old infant.

As such the only option here is to transfer to a more specialised hospital (B), taking care to manage the patient with NBM and fluids.

How well did you know this?
1
Not at all
2
3
4
5
Perfectly
50
Q

Which of the following is not a cause of PR bleeding?

A. Constipation with an anal fissure
B. Intussusception
C. Meckel’s diverticulum
D. Bacterial gastroenteritis
E. Abdominal migraine

A

E. Abdominal migraine

Often the first symptom of a migraine in children is with abdominal pain. This would be higher on the list of differentials if there was a family history of migraines.

If there was PR bleeding then it stands to reason that a diagnosis of migraine (E) becomes dificult to support, and bowel pathology should be sought.

The most common cause of PR bleeding in children is constipation with an anal fissure (A). You would expect bright red streaks of blood on the tissue or the surface of the stool.

Interssusception (B) presents with ‘redcurrant jely’ stool as an uncommon late stage symptom. It is due to the distal segment of bowel becoming necrotic and this is a mix of blood and mucus. There is a severe risk of perforation during insufflation at this late stage. It really should have been picked up before this.

A Meckel’s diverticulum (C) is fond in 2% of the population. It is an ileal remnant of the embryological yolk stalk, which can lead to ulceration and perforation, which then leads to severe dark red rectal haemorrhage.

A bacterial gastroenteritis (D) caused by Salmonella or Shigella may produce blood and /or pus mixed with the stool.

How well did you know this?
1
Not at all
2
3
4
5
Perfectly
51
Q

A mother brings her 2 year old to see you. She is very worried that he always
has diarrhoea or loose stools. He eats a normal diet, and no particular foods seem
to upset him but he often still has bits of vegetables or food he has eaten visible
in the stool. She thinks he is losing weight and he is starting to potty train, so
she is concerned this will affect his ability to anticipate needing the toilet. On
examination he is an alert and well-looking child with a normal capillary refill, heart
and respiratory rate. His abdomen is soft with no masses, there is no evidence of
wasting and his weight and height are following the 50th centile. What is the most
appropriate management?

A. Reassure the mother, explaining this is toddler’s diarrhoea and he will
grow out of it
B. Start loperamide as toddler’s diarrhoea is affecting his toilet training
C. Refer for endoscopy and biopsy to rule out coeliac disease
D. Refer for a colonoscopy and biopsy for inflammatory bowel disease
E. Order a blood test for thyroid function to rule out hyperthyroidism

A

A. Reassure the mother, explaining this is toddler’s diarrhoea and he will
grow out of it

Todler’s diarrhoea is a common cause of loose stool in children of pre-school age, it normally requires no treatment (A). The symptoms are linked to immature development of intestinal motility, and will usually resolve around 5 years of age.

There is a case for intervention if the symptoms are socially disruptive, if the can’t start school for example, in that case loperamide (B) would be indicated.

He is only just started to potty train, so this needs time to be implemented.

He is growing well and his weight and height are in line with each other and are in normal ranges, There is also no mention of any wasting, so coeliac disease (C) is unlikely. Of note, the first investigation for celiac is a blood test for anti-endomesial antibodies, not an invasive test like an endoscopy.

There is no sign of growth failure, bloody stools or the passage of mucus, making an inflammatory bowel disease (D) unlikely. The age of presentation for IBD is more commonly the late teens or early 20’s.

Although diarrhoea is a common symptom of hyperthyroid (E), it is a conditon more common in female teenagers or in the neonatal period in infants of Grave’s disease mothers. He has no other features consistent with hyperthyroidism; restlessness, increased appetite, tachycardia, or sweating.

How well did you know this?
1
Not at all
2
3
4
5
Perfectly
52
Q

A 10-year-old boy presents with recurrent mouth ulcers, abdominal pain,
distension and frequent episodes of diarrhoea with mucus. He has been losing
weight. On examination he is slim and plotting his growth shows a fall in weight
from the 50th centile to below the 9th. His abdomen is soft with generalized
discomfort on deep palpation but no masses are present. What is the most likely
diagnosis?

A. Ulcerative colitis
B. Crohn’s disease
C. Coeliac disease
D. Gastroenteritis
E. NEC

A

B. Crohn’s disease

The salient points here that give the diagnosis of Crohn’s disease (B) are:

Mouth ucers -Crohn’s is charecterised by skip lesions that can affect the entire digestive tract, from vermillion border to the anus. Distinguishes this case from Ulcerative Colitis (A)

Abdominal pain and distention - Gives an indication that this is serious pathology.

Frequent diarrhoea with mucus - This is a halmark of inflamatory bowel disease, the prescence of mucus goes against Coeliac (C) and Gastroenteritis (D).

Falling weight crossing the curves - again, gives an indication of serious pathology, and it tells you that this is a chronic illness. It is not Gastroenteritis (D).

In sumary the age of presentation (10 years old) excludes NEC (E), a disease of the newborn, and you would expect Coeliac to be a serious problem soon after weaning. That just leaves the inflamatory bowel disease, often the two are hard to distinguish in a clinical setting. In this case the mouth ulcers are your only discriminator, UC (A) is a disease of the rectum with a proximal spread, whereas Crohn’s is a multi site condition that is also more likely in younger patients.

How well did you know this?
1
Not at all
2
3
4
5
Perfectly
53
Q

A 15-year-old boy comes to see you, complaining of recurrent abdominal and back
passage pain relieved by passage of diarrhoea. He is also complaining of low back
and knee pain and last week there was blood mixed into his stool. He has been
losing weight recently. On examination he is slim and looks pale. His abdomen
is soft but tender in the left iliac fossa with no masses. What is the most likely
diagnosis?

A. Ulcerative colitis
B. Crohn’s disease
C. Coeliac disease
D. Gastroenteritis
E. NEC

A

A. Ulcerative colitis

Ulcerative colitis (A) can affect any age, but it is significantly more prevalent in the teenage years and early 20’s. It is a reccurent and chronic inflammatory disease of the rectum, which spreads proximally. It ‘never’ crosses the illieo-caecal valve

It has extra-intestinal symptoms, some of which can be seen in this case; sero-negative arthritis, spondylitis, pyoderma gangrenosum, erythema nodusum.

Crohn’s (C) is difficult to distiguish, but the history of back and joint pain is your clue here that it is UC.

Coeliac (C) would usually present at a younger age, it is charecterised by malnutrition, wasting and abdominal bloating. In the rare cases of coeliac that there is blood in the stool, it would be digested blood, not frank.

Gastroenteritis (D) is unlikely as this is a very chronic picture of disease, and the joint/spine involment doesn’t fit.

NEC (E) is an illness of the newborn, presenting with abdominal distention and bile stained aspirates.

How well did you know this?
1
Not at all
2
3
4
5
Perfectly
54
Q

Ninety-nine per cent of healthy term infants will pass meconium within the first
24 hours of life and all should do so within 48 hours. Which of the following is
not a cause of delayed meconium passage?

A. Hirschsprung’s disease
B. Cystic fibrosis with a meconium ileus
C. Choanal atresia
D. Imperforate anus
E. Meconium plug syndrome

A

C. Choanal atresia

Choanal atresia (C), despite the name, has nothing to do with the anus. It is a congenital blockage of the nasal airway. This leaves the infant in respiratory distress as they are obligate nose breathers.

Hirschprung’s (A) affects around 1 in 4000 and is the abscence of ganglion cells in the mesenteric and submucosal plexi in the rectum or colon. It leads to a norrowed section of bowel where it is denervated. The lack of movement in this section of bowel causes the delay in meconium passage. It is managed by a surgical resection of the denervated bowel section.

Cystic Fibrosis (B) results in inspissated (thickened) meconium which then has difficulty being passed. It will result in meconium ileus in around 20% of cases of cystic fibrosis.

An imporforate anus (D) is rarer than Hischprung’s and will need surgical intervention.

Meconium plug syndrome (E) is the commonest cause of delayed meconium passage. It results from a transient immaturity of the gut, it is managed with anal stimulation using a glycerine chip.

How well did you know this?
1
Not at all
2
3
4
5
Perfectly
55
Q

A 3-month-old baby is brought to accident and emergency because he has been
vomiting and having diarrhoea for the past month. His mother breastfed him until
he was 8 weeks old and he is now taking formula milk, 4–5 oz every 4 hours. On
examination he is alert but fussy and looks thin. He has eczema on his face, neck
and torso and the mother says this is new. The abdomen is soft, the genitalia are
normal with a significant nappy rash and the anal margin is erythematous. You
plot his growth in his red book and find that he was born on the 50th centile and
was following that but now he is on the 25th centile for weight. What is the most
likely diagnosis?

A. Cow’s milk protein intolerance
B. Lactose intolerance
C. Gastroenteritis
D. Hyper IgE syndrome
E. Wiskott–Aldrich syndrome

A

A. Cow’s milk protein intolerance

The onset of eczema since ending breastfeeding, the nappy rash, and the anal inflammation, all point towards a Cow’s milk protein intolerance (A).

This is an allergic reaction to the protiens within the cow’s milk, and is distict from lactose intolerance (B). This intolerance to cow’s milk protein usually presents with exposure in infancy, although if the intolerance is particularly severe the small amount of protein from a dairy consuming mother that is found in breastmilk is enough to cause problems.

The infant presents with worsening skin inflammation (eczema), Diarrhoea (potentially bloody), vomiting, failure to thrive, colic and irritability.

Lactose intolerance (B) is a deficiency in acose dehydrogenase, the build up of lactose in the gut creats an osmotic draw, leading to diarrhoea and dehydration. The intolerance is usually acquired following an episode of acute gastritis, but it can be congenital. The non-GI symptoms here are not consistent.

The history is too long for gastroenteritis (C) and the other symptoms are not consistent.

Hyper IgE syndrome (D), or Job’s syndrome, is an autosomal dominant immunodeficiency (you would think a parent having the condition would be mentioned) which leads to skin boils and severe eczema, but no gastrointestinal manefestations.

Wiskott-Aldrich syndrome (E) is also associated with eczema, as well as thrombocytopenia, and lymphopenia, but no gastrointestinal symptoms. It is an x-linked recessive disorder.

How well did you know this?
1
Not at all
2
3
4
5
Perfectly
56
Q

A 15-year-old Asian girl with Down’s syndrome came to accident and emergency with a prolonged fever. She has severe learning difficulties and was difficult to assess. Her parents think she is more unsettled than usual and not eating and drinking properly for the last 3 weeks. She is admitted as you cannot confidently find the source of the infection, but she has no cough, rash, vomiting, diarrhoea or meningism. The next day she complains of a headache and starts to vomit. She has a CT scan which is normal and then a lumbar puncture (LP). White cell count (WCC) 150×109/L (20 per cent neutrophils), red blood count 0, protein 2 g/L, glucose 1.2 mmol/L (serum glucose 6.0 mmol/L). What is the most likely cause of this meningitis?

A. Mycobacterium tuberculosis
B. Herpes simplex virus (HSV)
C. Streptococcus pneumoniae
D. Cryptococcus neoformans
E. Neisseria meningitidis

A

A. Mycobacterium tuberculosis

This is a very slow and indolent history of meningitis, it’s cetainly not consistent with the main bacterial causes of meningitis in the UK; S.pneumoniae (C) and N.Meningitidis (E).

Cryptococcus (D) is seen more in the immunosuppressed patients, such as those with untreated HIV infection.

HSV (B) results in encephalitis which is clinically difficult to distinguish from meningitis so should be empirically treated until laboratory confirmation. In this case we are told that this is meningitis, not encephalitis.

How well did you know this?
1
Not at all
2
3
4
5
Perfectly
57
Q

You are on elective in Uganda and spending the day on the paediatric ward. You
are told that it is the rainy season and malaria is now becoming increasingly
problematic. Almost all the children on the ward are suffering with the effects of
malaria. The first child is a 5-year-old boy with a cyclical fever, abdominal pain
and a 4 cm splenomegaly. He has 2 per cent parasitaemia on blood film. You are
asked how you would treat this child. What is the best initial management step?

A. IM quinine
B. IV fluids and IV quinine
C. IV fluids and prophylactic splenectomy
D. Emergency splenectomy
E. Oral atovaquone

A

B. IV fluids and IV quinine

This child has a moderate parasitaemia at 2%, a level above 5% would be considered severe. He needs IV antimalarial treatment and fluids is prudent here (B).

The splenomegaly here is secondary to acute haemolysis, unles there are clinical signs of a splenic rupture he shouldn’t undergo a splenectomy (C) or (D).

When this child is improving he could be switched ot oral anti-malarials (E)

How well did you know this?
1
Not at all
2
3
4
5
Perfectly
58
Q

A 3-year-old girl presents to accident and emergency with a 6-day history of fever
and she is over 38°C when measured by her mother with a tympanic thermometer.
She has become very miserable for the last few days. She has developed a rash on her trunk, which is blanching, erythematous and confluent. On examination, you
also note bilateral non-purulent conjunctivitis, cervical lymphadenopathy, and a
red tongue with lip cracking. Her extremities are also erythematous but not peeling. WCC 14×109/L, C-reactive protein 200 mg/L, and erythrocyte sedimentation rate 60 mm/hour. Blood culture is pending. What is the diagnosis?

A. Staphylococcal scalded skin
B. Toxic shock syndrome
C. Scarlet fever
D. Kawasaki’s disease
E. Measles

A

D. Kawasaki’s disease

Kawasaki’s (D) is an inflammatory disease of unknown origin, this girl has all of the features. All of the other conditions do share some symptoms with Kawasaki’s though. The serious complication that can arise is that there can be coronary artery aneurysms. The distinct feature in this case in the bilateral non-purulent conjunctivitis and the almost pathognomic ‘very misearble child’.

Of the other options here; Staph scalded skin (A) causes fever with peeling skin.

Toxic shock (B) presents as a red macular rash, fever and normally diarrhoea. There would be expected to be some route of intoxication with bacterial toxins (prolonged tampon use being the usual route), as this girl is 3 she lacks the risk factors.

Scarlet fever (C) has a ‘sandpaper’ rash and erythematous mucous membranes.

Measles (E) is now extremely rare, but thanks to anti-vaccinators is making a comeback, as such it should be kept in mind. It causes a febrile child with an erythemotous macular rash.

How well did you know this?
1
Not at all
2
3
4
5
Perfectly
59
Q

A 3-year-old boy presents with a right swollen eyelid. He has had a cold for the
last week but his eyelid started swelling yesterday. He has had no injury or broken
skin around the eye. On examination, his right eye is swollen and red, there is no
discharge, he is now unable to open his right eye and he has proptosis. You are
concerned about the complications of this infection. Within the last hour he has
become more drowsy and started to vomit. His observations are all normal. What
is the concerning complication in this case?

A. Visual loss
B. Abscess
C. Septicaemia
D. Orbital cellulitis
E. Meningitis

A

E. Meningitis

All of the answers here can be complications of orbital celulitis, the alarming feature here is the reduced level of conscious and the vomiting, both of which point to neurological pathology.

If there was focal neurology then it would make an abcess (B) more likely, this level of global neurological impairment is more charecteristic of meningitis (E). The route of infection here is likely to be the cellulitis spreading to the CSF surrounding the optic nerve.

The other possibility here is that septicaemia (C) has lead to shock and this hypoperfusion is causing the symptoms we see, but there isn’t anything to go on here to sugest that specifically.

(A), (B) and (D) all need investigating with a CT head and an opthalmology review.

How well did you know this?
1
Not at all
2
3
4
5
Perfectly
60
Q

A 14-year-old girl presented to the GP with an enlarged lymph node in her neck.
She first noticed it 3 weeks ago and it is increasing in size. She has also had a
dry cough, fevers, night sweats and weight loss. She has had a poor appetite over
the last 2 weeks, which her mother blames for her weight loss. There is no history
of foreign travel or tuberculosis (TB) contacts. A chest x-ray shows a mediastinal
mass. What is the most likely diagnosis?

A. Lymphoma
B. Pneumonia
C. TB
D. Lung tumour
E. Leukaemia

A

A. Lymphoma

Here we have a girl with a prolonged history of fever, this raises the possibility of a non-infectious cause.

We have been told that there are no TB contacts and the CXR shows a mediastinal mass, which is not typical of TB (C) or a pneumonia (B).

A primary Lung tumour (D) is very rare in children, so this is unlikely.

With the lymphadenopathy lymphoma and Leukaemia (E) are potenitals, a lymph node biopsy and a blood film and bone marrow aspirate is neded. Given that she has a mediastinal mass causing a dry cough and the ‘B’ symptoms Lymphoma is more likely.

How well did you know this?
1
Not at all
2
3
4
5
Perfectly
61
Q

A 6-year-old girl presents to accident and emergency with a fever. She has no
history of cough, cold, vomiting, diarrhoea, rash, headache or joint pain. On
examination, she is tachycardic at 150 bpm and there are two petechial spots on
her right ankle. Her capillary refill time is 4 seconds and she has cold feet. All
her other observations are normal. What is the most appropriate course of action?

A. Inform the consultant about child protection concerns
B. IV ceftriaxone
C. IV fluid bolus and IV ceftriaxone
D. Admit to the ward for observation
E. Discharge home and advise to return if the rash spreads

A

C. IV fluid bolus and IV ceftriaxone

This girl has subtle, but distinctive signs of meningococcal sepsis. The prescence of petechial spots, with a fever, and clinical signs of shock means that antibiotics must be comenced straight away (B and C). The tachycardia, and slow capillary refil time indicate early shock, so a fluid bolus is needed (C).

Any bruising should make you think of non accidental injury, but in this case there is a rather obvious pathology at work so (A) is incorrect.

She needs urgent treatment so (D) is incorrect, and (E) is just plain negligent.

How well did you know this?
1
Not at all
2
3
4
5
Perfectly
62
Q

A 3-year-old boy was brought to accident and emergency with his mother. She says he has been limping for a day now and refusing to walk for the last 2 hours. He has had a fever to 39°C which can be brought down with paracetamol. He has had no vomiting, diarrhoea, rash, cough, coryza or injury. He lives with his mother and is her only child. She is currently unemployed and has a background of depression. On examination of the right leg he has a swollen thigh and cries inconsolably when it is touched. It is red and tender. He refuses to allow movement of the hip either passive or active. The left leg is unremarkable on examination. What is the most likely diagnosis?

A. Perthes’ disease
B. Septic arthritis
C. Fractured femur due to accidental injury
D. Juvenile idiopathic arthritis
E. Fractured femur due to non-accidental injury

A

B. Septic arthritis

This child is presenting with fever and a limp, he also has a reduced range of movement and signs of acute inflammation over the hip joint. All of this makes a septic arthritis (B) affecting the right hip the most likely diagnosis here.

Perthe’s disease (A) usually affect older children, those older than 5, it results from an avascular necrosis of the femoral head. The pathology of which is unclear. You wouldn’t expect the child to be febrile with localised inflammation.

Without a reported history of trauma we can exclude accidental injury (C), non-accidental injury (E) must always remain in mind. This is particularly true given the social background of deprivation and depression, but it isn’t the most likely diagnosis here.

Juvenile idiopathic arthritis (D) is diagnised after 6 weeks of joint pain and swelling that is unexplained after other diagnoses have been excluded.

How well did you know this?
1
Not at all
2
3
4
5
Perfectly
63
Q

A pregnant woman attends her booking appointment at the antenatal clinic and
has her routine blood tests done. She is now 13 weeks pregnant with her first child
and you have a positive result for cytomegalovirus (CMV) IgM. You need to discuss
the implications of CMV infection on her unborn child. Which of the following are
not features of congenital CMV infection?

A. Deafness
B. Intrauterine growth retardation
C. Hydrocephalus
D. Thrombocytopenia
E. Congenital cardiac defects

A

E. Congenital cardiac defects

All of the features here are possible sequale of congenital CMV infection, apart from congenital cardiac defects (E), which are seen in other conditions such as congenital rubella.

Mothers are screened for CMV during pregnancy as standard, along with CMV they are also screened for Hep B, rubella, Syphilis, and HIV.

This is different to the newborn screening test, the TORCH screening which looks for;

toxoplasmosis, rubella, cytomegalovirus, herpes simplex, and HIV

How well did you know this?
1
Not at all
2
3
4
5
Perfectly
64
Q

A 5-year-old girl was admitted to the ward after she presented to her local accident and emergency with diarrhoea. She was passing 7–8 loose, watery stools per day for the last 4 days and had been vomiting for 1 day prior to this. There was blood in the stools and this had worried her mother. You ask about foreign travel and her mother reveals they had been in India until 2 weeks ago, staying with family and drinking tap water. She had no vaccines prior to travelling. On examination, she now has abdominal pain, swinging pyrexias, right upper quadrant tenderness but no rebound or guarding. You notice a pale pink (rose) spot on her trunk. What is the most likely infecting organism?

A. Rotavirus
B. Shigella spp.
C. Vibrio cholerae
D. Salmonella typhi
E. Escherichia coli 0157

A

D. Salmonella typhi

Of the organisms on this list, Rotavirus (A) is by far the most common cause of gastroenteritis in the world, it accounts for over 50% of cases. In this case, however, there are some unusual features which would make you suspect another cause, the distictive rose spot and the blood in the stols.

How well did you know this?
1
Not at all
2
3
4
5
Perfectly
65
Q

A 10-month-old baby is brought to accident and emergency by ambulance having
had a seizure. His mother reports that he went floppy suddenly and then his right
arm and leg started shaking and he was not crying. It lasted less than 5 minutes
and he was sleepy afterwards. He has had a fever and runny nose for the last
2 days and is off his food. Why is this not a febrile seizure?

A. He is too young
B. He has had a focal seizure
C. He has recently had a viral illness
D. The seizure lasted too long
E. The fever was not high enough

A

B. He has had a focal seizure

Febrile seizures are common in children aged 6 months to 6 years, this makes (A) an incorrect answer. It is believed that they occur due a rapid rise in body temprature at the start of a febrile illness. This is particularly the case for viral illnesses., so (C) is spurious.

There is no agreed range of tempratures that put children at risk for a febrile seizure, so (E) is oncorrect.

Any seizure lasting less than 5 minutes that is self resolving is classified as a simle seizure, so (D) is not correct.

The answer to this question is that the focal nature (B) of the seizure excludes the possibility of this being a febrile seizure. Focal neurology points to focal pathology, this child needs a head CT and antibiotic cover for a potential bacterial meningitis or abcess, as well as antivirals for herpes simplex virus (potentially this is caused by meningoencephalitis).

How well did you know this?
1
Not at all
2
3
4
5
Perfectly
66
Q

A 4-year-old boy has been brought into accident and emergency with breathing
problems. He is assessed by the paediatric team and found to have inspiratory and expiratory stridor, audible wheeze, lip and tongue swelling, and an urticarial rash on his trunk and abdomen. His heart rate is 167 bpm and his respiratory rate is 40, BP 90/45 mmHg. What is the single most important management step?

A. Do not examine his throat as this may distress him
B. Give a normal saline fluid bolus
C. Give IV adrenaline 0.1 mg/kg of 1:10 000
D. Give IM adrenaline 0.01 mg/kg of 1:1000
E. Mobilize the paediatric anaesthetist as his airway is compromised

A

D. Give IM adrenaline 0.01 mg/kg of 1:1000

It’s vital here that you recognise this as an anyphlactic reaction, there are cardiac and respiratory involvement with signs of systemic shock (tachycardic and hypotensive).

A fluid bolus (B) is needed here due to the distributive shock, but it doesn’t solve the root cause of the shock, so is not the most important step.

In order to stop the capilliary leakage and the swelling you need to give adrenaline, in anaphylaxis the adrenaline needs to be given IM not IV (C), IV adrenaline is used in the cardiac arrest protocol.

A paediatric anaesthetist (E) will be needed to assess the airway, and that should be called for but not before giving the adrenaline as it will slow further airway compromise.

It is important to be aware that there is a biphasic reaction, and the child may have another reaction in 6-12 hours despite no further contact with the allergen, long after the adrenaline has worn off. This is why steroid and antihistamine cover is the normal procedure.

(A) is a consideration in bacterial induced airway obstruction, seen in epiglotitis or bacterial tracheitis.

How well did you know this?
1
Not at all
2
3
4
5
Perfectly
67
Q

A 12-year-old girl with a history of discitis in her lumbar spine was admitted
following investigation at her tertiary centre. She was started on IV benzylpenicillin
and clindamycin. She received 24 hours of medication and a rash appeared on her
trunk and arms. There were discrete red lesions which outlined a central target
lesion. They were non-blanching and itchy. What is the most likely diagnosis?

A. Erythema migrans
B. Erythema toxicum
C. Erythema marginatum
D. Erythema nodosum
E. Erythema multiforme

A

E. Erythema multiforme

A target rash is described as Erythema multiforme (E), it has many causes, a drug reaction (as in this case penicillin), infection (atypical pneumonia) or it can be idiopathic.

Erythema Migrans (A) - is the characteristic rash of lyme disease

Erythema Toxicum (B) - is a benign rash that is seen in newborns (first 3 weeks of life) that is usually a pustular or vesicular rash surrounded with an erythemotous area.

Erythema Marginatum (C) - is seen in rheumatic fever along with carditis, arthralgia, subcutaneous nodules and the pathognomic Sydenham’s chorea.

Erythema Nodusum (D) - can be seen as a drug reaction (particularly sulphonamides), infections such as strep species, autoimmune, and malignancy. The most famous cause of this is, however, inflammatory bowel disease.

68
Q

A 3-year-old boy is admitted to the children’s ward. He has been isolated in a
cubicle as he is at risk of infections. He is awaiting a bone marrow transplant and
has a brother with the same condition. His mother tells you they both have SCID.
What are the likely immune function test results in SCID?

A. Normal B cells, normal T cells, normal immunoglobulins
B. Low B cells, low T cells, low immunoglobulins
C. Normal B cells, normal T cells, high immunoglobulin M subsets
D. Low B cells, normal T cells, low immunoglobulins
E. Normal B cells, low T cells, normal immunoglobulins

A

B. Low B cells, low T cells, low immunoglobulins

Severe combined immunodeficiency (SCID) is a group of disorders, as the name sugessts, that result in both T and B-cell dysfunction. Knowing this we see that only (B) can be correct here.

(A) is a normal blood result

(C) appears to be a form of hyper IgM syndrome, this is caused by a defect in the CD40 ligand and causes defective class switching as well as problems with T-cell function and this leads to reccurent infections.

(D) suggests Brunton’s agammaglobulinaemia, where the lack of B cells lead to low immunoglbins and a predispostion to respiratiory and CNS infections.

(E) may be due to underdevelopment or abscence of the thymus, as this only affects the T-cells. This can be linked to DiGeorge’s syndrome

69
Q

A 3-week-old baby attends accident and emergency with bloody diarrhoea. Mum
says he has been having diarrhoea for the past 2 days since she started using
formula milk. He was previously breastfed and mum was not having any dairy
products due to lactose intolerance. He also has eczema on his cheeks and a strong family history or asthma and eczema. Mum is concerned that he may be allergic to milk too. What is the most likely diagnosis?

A. Lactose intolerance
B. Gastroenteritis
C. Cow’s milk protein intolerance
D. Fructose intolerance
E. Galactosaemia

A

C. Cow’s milk protein intolerance

The natural history of this baby’s issues tells you that there is an issue with the formula milk. The prescence of eczema on his cheeks tells you that this is an atopic presentation, and the bloody diarrhoea points at a Cow’s milk protein intolerance (C) as opossed to Lactose intolerance (A).

Cow’s milk protein intlerance presents with diarrhoa on the introduction to cow’s milk formula or breast milk if the allergy is severe and the mother is eating dairy.

Congenital lactose intolerance is very rare as it would involve complete abscence of lactase at the intestinal brush border, it usually occurs secondary to infection or after 2 years of age when levels start to fall naturally in most people.

Fructose intolerance (D) is very rare, it presents with hypoglycaemia, failure to thrive and vomiting. There will also be hepatomegaly, jaundice, renal complications and severe metabolic acidosis.

Gastroenteritis (B0 should always be a consideration, but without signs of fever or any vomiting, it is unlikely.

Galactosaemia (E) is a deficiency in glactose-1-phosphate uridyl transferase deficiency. It can present as metabolic aberations, sepsis, vomiting and collapse.

70
Q

A 2-year-old child is brought to cardiology clinic due to a heart murmur heard
by the GP after an examination when she was recently unwell. She was born at
40 weeks by normal vaginal delivery but was noted to have a cleft palate at birth.
She was kept in hospital for establishment of feeding but during this time she had
a seizure, noted later to be because her calcium was low. You hear a harsh, grade
3/6 pansystolic murmur, loudest at the left lower sternal edge, consistent with
a ventral septal defect (VSD) as seen on echocardiogram. With this history and
current examination finding, you wish to exclude DiGeorge’s syndrome. What is
the best diagnostic test?

A. Karyotype
B. FISH (fluorescence in situ hybridization)
C. ELISA (enzyme-linked immunosorbent assay)
D. Geneticist review and diagnosis
E. Identification of specific mutation

A

B. FISH (fluorescence in situ hybridization)

DiGeorge’s syndrome is a microdeletion at chromosome 22q11.2. We see in the description here some of the clasical signs, there are also aortic arch abnormalities, thymic hypoplasia, and a syndromic facial appearence.

In order to best identify microdeletons, FISH (B) is the technique of choice.

Abnormalities with chromosome numbers or translocations (such as Down’s or Turner’s syndromes) are best seen with karyotype studies (A).

ELISA (C) is used to look for the prescence of specific proteins. Such as antibody screening to check for an infection or immunity.

Genetic review (D) may be helpful for future family planning, but doesn’t help with diagnosis of this child.

Identifying a specific mutation (E) is a rather loose term but would be part of a karyotyping test or a PCR test.

71
Q

A 13-year-old girl is on the ward having a bone marrow transplant for acute
leukaemia. She is noted to be profoundly anaemic with haemoglobin 5.9 g/dL and
she is due to receive a transfusion of one unit of red blood cells. You are called
to see her 5 minutes after starting the transfusion. She has come out in a rash,
is looking frightened, with a heart rate of 120 bpm and respiratory rate of 30. As
you arrive, you can see she has swollen lips and tongue and her blood pressure is
measured as 90/45 mmHg. What is the best initial management step?

A. Repeat a full set of observations as it is likely to be anxiety resulting
in the abnormal heart rate, respiratory rate and blood pressure. If still
abnormal, stop transfusion
B. Stop the transfusion and return the unit of blood to blood bank
C. Stop the transfusion, take down the giving set, give IM adrenaline
immediately
D. Give IM adrenaline, stop the transfusion, take down the giving set
E. Stop the transfusion, give IM adrenaline immediately and restart if the
reaction settles

A

C. Stop the transfusion, take down the giving set, give IM adrenaline
immediately

It is good to be familiar with the Handbook of Transfusion Medicine, even wothout knowing that specifically this is clearly an anaphylactic reaction to the transfusion. This maes (A) a shocking answer!

The first thing with any allergic reaction is to remove the antigenic substance, in this case, the blood after that IM adrenaline should be given. This means that (D) is the wrong order, and (B) doesn’t treat the patient.

If the patient has responded with anaphylactic shock to this unit of blood then restarting it (E) is absolutely wrong.

72
Q

A couple are referred to a geneticist as they are planning on having their first child.
There is a history of Wiskott–Aldrich syndrome on the woman’s side. The woman’s
father and great grandfather have the condition (eczema, thrombocytopenia,
recurrent infection) but she is unaffected. There is no history of the condition in
the man’s family. What is the risk of having the condition if the child is a boy or
a girl respectively?

A. Boy: 1/4; Girl: 1/4
B. Boy: 1/2; Girl: 0
C. Boy: 0; Girl: 0
D. Boy: 0; Girl: 1/2
E. Boy: 1; Girl: 0

A

B. Boy: 1/2; Girl: 0

Wiskott-Aldrich syndrome is an X-linked condition where a mutation in the WASp gene causes problems with the cell cytoskeleton. It causes the symptoms reported in this vignette.

Even if you had never heard of this condition you can work out that it is X-linked. With an obscure condition we can discount it being an autosomal recessive, it has affected 3 generations out of 5 of this womans family, unless there is some serious consanguinity this is statistically improbable. If it were autosomal dominant it couldn’t have skipped generations as in this case, it would be a continous line of sucession that either persisted or became extinct.

Logically it has to be X-linked or X-dominant. If it were X-dominant then the daughter of an affected male would have the condition, the mother in this case is unaffected, so this must be X-linked.

Knowing this, we know that the mother is a carrier, she has a 50% chance of passing the afflicted X-chromosome on to any boys or girls she has. However the girls can only be carriers not affected individuals. This gives us our answer of (B).

(A) is the probability for an autosomal recessive trait where both parents are carriers.

(C) is where it is autosomal recessive with one parent as a carrier and the other is not. Or it is an autosomal dominant and the mother hasn’t inherited the defective gene.

(D) only makes sense if the father is a carrier of an x-dominant trait

(E) would be if the mother was affected by an x-linked condition.

73
Q

A 4-year-old boy with severe ezcema is brought to accident and emergency by
his mother. His skin has been worse recently since the weather has become colder. He is scratching a lot more and now is very miserable and has a temperature of 38.6°C today. On examination of his skin he has multiple areas of erythematous, excoriated lesions on his elbow and knee flexures as well as his trunk and back. In addition they are hot, tender and slightly swollen with areas of broken skin. There are also some yellow fluid-filled vesicles on some of these lesions. You send some blood tests and commence him on IV flucloxacillin and aciclovir. Which are the two most likely organisms that can complicate eczema?

A. Gram-positive cocci and herpes simplex virus
B. Gram-negative cocci and herpes simplex virus
C. Gram-positive cocci and varicella zoster
D. Gram-negative bacilli and herpes zoster
E. Gram-positive bacilli and herpes simplex

A

A. Gram-positive cocci and herpes simplex virus

The two important complications of eczema are cellulitis, or bacterial superinfection, and eczema herpeticum (caused by herpes simplex virus). Knowing this we can exclude (C) and (D) which both reference different names for the causative organism of chicken pox/shingles.

In order to choose from the remaining options we need to know that the causative organism for cellulitis are gram-positive cocci of the genus Staphylococcus and Streptococcus. Therefore our answer is (A).

74
Q

A 3-year-old boy has been admitted to hospital with a right-sided pneumonia
and pleural effusion. The pleural fluid grew Gram-positive cocci. He is on
IV ceftriaxone, oral azithromycin and has a chest drain in situ. On further
questioning of Richard’s mother, you establish that he has had multiple chest
infections since he was born (in the UK). He has been admitted three times before
and also had a sinus wash out following an episode of sinusitis. He has no cardiac
anomalies or dysmorphism. His mother also tells you about his older brother, who
sadly died of meningitis aged 6 years old. He too had ‘more than his fair share of
infections’. The two brothers had different fathers but his mother is HIV negative.
What is the most likely underlying immunodeficiency in this family?

A. DiGeorge’s syndrome
B. Complement deficiency
C. X-linked agammaglobulinaemia
D. Severe combined immunodeficiency disorder (SCID)
E. HIV

A

C. X-linked agammaglobulinaemia

The specific mention of the lack of cardiac anomolies or dysmorphic features tells you this isn’t Digeorges (A). The mother is HIV negative so it’s unlikely that her 3-year old caught the virus (E) by a route other than vertical transmission, especially when you consider the history with the brother.

SCID (D) would present earlier in life, and would likely have been fatal before now if it was undiagnosed.

Complement deficiency (B) typically predisposes those affected to meningococcal sepsis.

If you consider that this condition has affected two brothers from different fathers then it makes an X-linked condition likely. In X-linked agammaglobulinaemia there is a lack of B-cells and thus, antibodies. This leaves those affected prone to getting bacterial infections and in particular, respiratory infections.

75
Q

You are in immunology clinic and the first patient is a 2-year-old boy who has
a complement deficiency. You know this involves a cascade of proteins involved
with innate immunity but are unsure about the manifestations in children. The
professor of immunology asks you which organism is this child at risk of being
infected with. He gives you a clue by telling you the child has a late complement
deficiency, meaning C5-C9. What is the most likely causative organism that infects
these children?

A. Streptococcus pneumoniae
B. Neisseria meningitidis
C. Haemophilus influenzae
D. Mycobacterium tuberculosis
E. Pneumocystis jiroveci

A

B. Neisseria meningitidis

Deficienciesin the complement pathway can predispose to a variety of conditions. A deficiency of C3 results in reccurent pyogenic infections. This is because C3 is cleaved into C3a and C3b which are responsible for chemotaxis/mast cell degranulation and opsonisation , respectively. Without these mechanisms full blown bacteriemia is common.

In this case the deficiency is later, at C5-C9, this is well known to predispose to infection with N.Meningitidis (B).

Another complement deficiency, involving C1, C2, and C4 is associated with autoimmune conditions such as SLE.

Of the other answers here; encapsulated bacteria such as (A) and (C) particularly affect those with without a spleen or hypospleenism.

(D) and (E) are associated with increased rates of infection in the immuno compromised, such as HIV, steroids, or immunosuppresive drugs.

76
Q

A 5-year-old boy was diagnosed with asthma aged 3 years. He presented to
accident and emergency with shortness of breath, increased work of breathing and a 1-week history of coryzal illness and fever. On examination he is tachypnoeic 60/ min, tachycardia 160 bpm and has minimal air entry bilaterally. He has intercostal recession, tracheal tug and is too breathless to complete a sentence. Oxygen saturation is 90 per cent in air. What is the initial management of this boy?

A. Immediate intubation and ventilation
B. High flow oxygen through non-rebreather mask
C. IV salbutamol and magnesium sulphate infusion
D. Back to back salbutamol and Atrovent though oxygen driven nebuliser
E. Trial of continuous positive airway pressure support

A

D. Back to back salbutamol and Atrovent though oxygen driven nebuliser

This child is having a life-threatening asthma attack, but we still need to follow the stepwise British Thoracic Society guidelines. The first step in this treatment pathway is back to back Salbutamol nebulisers (D) there is a benifit to these being oxygen driven due to his hypoxia.

Giving oxygen alone (B) is of little benefit as it doesn’t improve the disease process.

The second line treatment is IV salbutamol/aminophylline and magnesium sulphate (D). THis however is only used after back to back salbutamol inhalers have failed.

77
Q

A 3-year-old child presents to the GP with a chronic cough for the last month.
He had previously been fit and well since he suffered a severe pertussis infection
when he was 1 month of age. He has subsequently been fully immunized but
was noted to be on the 0.4th centile for height. What is the most likely cause
for his cough?

A. Cystic fibrosis
B. Recurrent pertussis infection
C. Habit
D. Asthma
E. Bronchiectasis

A

E. Bronchiectasis

A severe respiratory infection in early childhood can present later as Bronchiectasis (E), this is due to dilation and poor mucociliary elevator action. As with any chronic illness this can impact on the growth of the child, as we see here.

Of the other options herer, Asthma (D) is always a possibility in children with a chronic cough. You would expect it to be a nighttime cough with other atopic signs.

CF (A) would likely have gastrointestinal involvement, and a through history might elucidate this. In additon, as CF is on the newborn screening programe it is unlikely to have been missed in this 3-year old.

Reccurent pertussis (B) is all but impossible, once teated pertussis has an natural immunity (it does wane over time). The child wouldn’t get re-infected at the age of 3.

Habit (C) would be a diagnosis of exclusion.

78
Q

A 15-year-old boy attends his GP with a week of cough productive of yellow
sputum, fever to 39°C and chest pain on the right side of the chest on coughing.
There is no history of foreign travel or unwell contacts. On examination there is
reduced air entry in the right lower zone with crepitations and bronchial breathing.
You diagnose a right-sided chest infection. What is the most likely causative
organism?

A. Staphylococcus aureus
B. Mycobacterium tuberculosis
C. Streptococcus pneumoniae
D. Mycoplasma pneumoniae
E. Chlamydophila pneumoniae (Chlamydia pneumoniae)

A

S.Pneumoniae (C) is the most common cause of pneumonia, and would be most likely in those under 4.

M.Pneumoniae (B) is the more common organism in older children presenting with pneumonia.

TB (B) is not consistent with this short history and lobar focus. You would also expect a haistory of weight loss and fevers and a degree of anorexia.

Both Staph (A) and Chlamydia (E) pneumonia are possible but less likely organisms statistically.

79
Q

A 4-year-old girl has recently moved to the area and is registering with you, her
new GP. She has had a diagnosis of primary ciliary dyskinesia (PCD) made last
week and the parents wish to know more about the complications. Which of the
following is not a complication of PCD?

A. Pancreatic insufficiency
B. Infertility
C. Sinusitis
D. Bronchiectasis
E. Dextrocardia

A

A. Pancreatic insufficiency

In this condition there is a polygenic inheritance that leads to porr or uncoordinated movement off the cillia. This leads to problems with all of the body systems that rely of cilliry action to function correctly.

Of the options here the reproductive tract (B) (Ductuli efferentes in males or fallopian tubes in females) requires coordinated cilia to function. The sinuses (C) need cillia to clear mucus, failure here leads to sinusitis. Finally loss of cillia function in the respiratory tract (D) leads to inability to clear debris and pathogens and this leads to recurrent cheast infections.

Interestingly, during embyonic development cillia are also crucial for determining the site of the internal organs, the loss of function leads to dextrocardia (E) or full situs inversus.

Of the options here, only pancreatic secreations (A) do not require cilliated movement, they instead rely on smooth muscle action.

80
Q

A 26 week, premature baby was born by emergency caesarean section due to
maternal pre-eclampsia. He required ventilation until age 38 weeks corrected
gestation and is still requiring oxygen to maintain his saturations. At 12 months
of age he has poor vision and neurodevelopmental function, requires home oxygen and was admitted for a recent respiratory syncytial virus (RSV) bronchiolitis. What is the underlying diagnosis of his respiratory problems?

A. Respiratory distress syndrome (hyaline membrane disease)
B. Chronic lung disease (bronchopulmonary dysplasia)
C. Cystic fibrosis
D. Diaphragmatic hernia
E. Pulmonary hypoplasia

A

B. Chronic lung disease (bronchopulmonary dysplasia)

the definition of chronic lung disease (B) is that there is a continued requirement for oxygen at 36 weeks corrected gestational age, or 28-days post-partum. In this case we clearly see that this is the case with this infant. This conditionis found in premature babies as a consequence of barrotrauma or other ventilation trauma during the neonatal period, surfactant deficiency or due to oxygen therepy.

Its effects can be mitigated by using the minimum amount of ventilatory support needed in premature infants. These children are particularly vulnerable to RSV and often recieve preventative monoclonal antibodies.

RDS (A) is due to surfactant deficiency and may occur soon after birth. If it persists then it becomes, by definition, chronic lung disease.

This isn’t the presentation of CF (C), you would expect meaconium ileus or neonatal jaundice.

If there was a diaphragmatic hernia (D) or renal issues, then it would result in pulmonary hypoplasia (E) and this would present with severe respiratory problems at birth.

81
Q

John is a 2-year-old boy whose mother has been concerned about a cough for the
last 2 weeks which started out of the blue. He has been previously fit and well
with no respiratory or cardiac problems from birth. There is no family history of
illness. He is thriving and eating as normal, but has a persistent cough, recently
productive of yellow and slight blood stained sputum. You suspect that John may
have a pneumonia and lung collapse secondary to an inhaled foreign body. Which
is the most likely location of this boy’s foreign body?

A. Left lower lobe
B. Right upper lobe
C. Right middle lobe
D. Left upper lobe
E. Right lower lobe

A

C. Right middle lobe

The right main brochus is the most vertical and it’s the shortest, as such it is the site for the majority of inhaled foreign bodies.

Of the three lobes, it is the middle lobe (C) that has the most direct parth in terms of it’s bronchi, so it is the most likely location for this inhaled object.

See the attached image.

82
Q

A couple who are known to both be carriers of cystic fibrosis ask to see you. They
had genetic counselling but declined antenatal diagnostic testing and their baby
has now been born and is ready to be discharged home. The parents are now keen to get the baby tested so that if treatment is required it can be initiated early on. What initial test do you suggest for the baby?

A. Newborn blood spot screening
B. Chest x-ray
C. Faecal elastase
D. Genetic testing
E. Sweat test

A

A. Newborn blood spot screening

During the national newborn blood spot screening (A) programme infants are screened for CF as well as Sickle cell disease, congenital hypothyroidism, Phenylketonuria, and medium chain acyl dehydrogenase deficiency. They are just rolling out four further metabolic diseases that wil soon be nationally tested for; maple syrup urine disease (MSUD), isovaleric acidaemia (IVA), glutaric aciduria type 1 (GA1), and homocystinuria (pyridoxine unresponsive) (HCU)

If the sample taken at the screening test is positive for immune reactive trypsin then it will warrant further genetic testing (D) to identify the mutation causing the cystic fibrosis.

Following this the gold standard test is the seat test (E) to confirm.

Faecal elastase (D) is a marker of pancreatic insufficiency, a consequence o0f CF, but not always present.

A chest x-ray (B) in a newborn is rather pointless.

83
Q

A 5-week-old baby was admitted today to the children’s ward with bronchiolitis.
The nasopharyngeal aspirate identified respiratory syncitial virus. He was
saturating to 96 per cent in air this morning and was feeding two-thirds of
his usual amount of formula milk. You are asked to review him as his work of
breathing is worsening now it is night time. He has nasal flaring, intercostal and
subcostal recession, tachypnoea and crepitations and wheeze heard bilaterally.
What do you expect his capillary blood gas to show?

A. pH 7.16 PCO2 kPa 3.1 PO2 10.0 kPa BE –8 HCO–
3 18 mmol/L
B. pH 7.38 PCO2 kPa 5.5 PO2 12.0 kPa BE +1 HCO–
3 25 mmol/L
C. pH 7.20 PCO2 kPa 8.2 PO2 8.3 kPa BE +2 HCO–
3 26 mmol/L
D. pH 7.40 PCO2 kPa 1.2 PO2 7.5 kPa BE +5 HCO–
3 28 mmol/L
E. pH 7.47 PCO2 kPa 6.3 PO2 11.0 kPa BE +10 HCO–
3 35 mmol/L

A

C. pH 7.20 PCO2 kPa 8.2 PO2 8.3 kPa BE +2 HCO–
3 26 mmol/L

Due to the illness process he is having increased work of breathing, and this is leading to him tiring . This will result in a type 2 respiratory failure, as we see in (C), this is because he is not breathing effectively enough to oxegenate (saturations are falling), and so he couldn’t possibly be blowing off enough CO2.

(A) and (B) don’t demonstrate the hypercapneia we would expect, in fact (A) is a metabolic acidosis, while (B) is normal.

(D) is hyperventialtion without sufficient oxygenation, this may have been his blood gas prior to exhaustion.

(E) is a metabolic alkalosis.

84
Q

Clara is a 14-year-old girl who was diagnosed with muscular dystrophy when she
was younger. She now mobilizes in a wheelchair and other co-morbidities include
a scoliosis and cardiomyopathy. She is being seen for her annual review in clinic.
Which of these would best represent the respiratory complications of muscular
dystrophy?

A. Normal FVC, low FEV1/FVC ratio
B. Flattened diaphragms on chest x-ray
C. Morning dips in peak expiratory flow rate
D. Extrathoracic obstruction on flow-volume loops
E. Reduced FVC, normal FEV1/FVC ratio

A

E. Reduced FVC, normal FEV1/FVC ratio

Muscular Dystrophy causes a restrictive pattern of respiratory pathlogy, this is due to the weak intercostal and diaphragm muscles, combined with scoliosis (which restricts cheast wall expansion). The restrictive spirometry result here is (E).

(A) is an obstructive picture, as seen in asthma, for example.

(C) is a hallmark of asthma

Flattened diaphragms (B) is seen in hyperinflation with air trapping, this is found in bronchiolitis.

Extrathoracic obstruction (D) would e if there was an obstruction of the airway at a level higher than the thoracic inlet. You would expect syptoms of cough or massive lympadenopathy.

85
Q

A 10-month-old baby boy is brought to accident and emergency with inconsolable
crying. His mother says he is a miserable baby and even after feeding he does not
settle. He has recently started to cruise around furniture, but is not yet walking. His
crying has been worse today and both his parents had been awake all night due to his incessant crying. On examining the baby, you note that he is more upset when being handled and is a bit better when lying on his front. You do a chest x-ray which shows three posterior rib fractures; his mother states he fell down some steps yesterday. What is the likely diagnosis and appropriate management strategy?

A. Birth trauma; no intervention necessary as they will heal spontaneously
B. Accidental injury; ensure no pneumothorax present, reassure and
discharge home
C. Accidental injury; ensure no pneumothorax present and admit for
observation
D. Non-accidental injury (NAI); advise the parents you will refer to social
services and discharge home
E. NAI; discuss with social services and paediatric consultant and admit
the child to a place of safety

A

E. NAI; discuss with social services and paediatric consultant and admit
the child to a place of safety

A posterior rib fracture is highly indicitive of a NAI, this is because the point of greatest stress on a rib is the sharpest angle, near the angle of the rib. This means that accidental injury, (B) and (C), are unlikely.

Birth should not lead to rib fractures (A) and they should be pretty healed at 10 months anyway. Birth trauma tends to lead to clavical fractures (shoulder dystocia or forceps use).

This child needs safegaurding, he needs to be admitted to ensure safety (E) and the consultant involved. Discharging the child would be inappropriate (D).

In this case it may well be that the child is inconsolable after feeding due to reflux disease. The implication here is that the parents have injured the child out of frustration/anger.

86
Q

What is the most common congenital heart defect?
A. Coarctation of the aorta
B. Ventricular septal defect (VSD)
C. Atrial septal defect
D. Patent ductus arteriosus
E. Transposition of the great arteries

A

B. Ventricular septal defect (VSD)

in decreasing order of frequency;

B. Ventricular septal defect (VSD) - around 1/3 of all congenital heart defects

D. Patent ductus arteriosus - Just over 10% of congenital heart defects

The remaining three conditions (below) each account for about 5% of congenital heart defects.

A. Coarctation of the aorta

C. Atrial septal defect
E. Transposition of the great arteries

87
Q

Which of the following is not a presenting symptom or sign associated with
congenital heart disease?
A. Respiratory distress with feeds
B. Cyanosis
C. Hepatosplenomegaly
D. Vomiting with feeds
E. Sweating with feeds

A

D. Vomiting with feeds

congenital heart disease may present after the ante-neo-natal period with signs which can be obvious or subtle.

The classic presentation of congenital heart disease are when children become breathless (A) and sweaty (E) when feeding.

This additional effort of feeds (already the most energy intense activity in an infant’s life), means the child will often have restricted growth and may lead to failure to thrive.

The back pressure of right-sided heart failure leads to Hepatosplenomegaly (C), it is unlikely you will see a raised JVP in an infant as the neck is very short.

Cyanois (B) occurs in a right-to-left shunt.

There isn’t a reason why a cardiac defect would cause vomiting (D), this is more in keeping with a reflux disease.

88
Q

Which of the following is not a feature of an innocent murmur?

A. Systolic murmur
B. Diastolic murmur
C. Asymptomatic
D. Heard only at the left sternal edge
E. No heaves or thrill

A

B. Diastolic murmur

Characteristics of an innocent murmur are; a soft blowing systolic murmur (A), located at teh left sternal edge (D), with no radiation, and no diastolic component (B), or parasternal thrill (E) and normal heart sound in an asymptomatic patient (C).

89
Q

You are asked to see a 2-year-old child with difficulty in breathing, a runny
nose and a barking cough. His mother tells you he had a heart defect repaired as
a baby and he still has a murmur. On examination he has noisy breathing with
mild subcostal recession. He is apyrexial with a respiratory rate of 44 breaths per
minute and heart rate of 152 beats per minute; capillary refill is 1–2 seconds.
His throat is red and the tonsils are enlarged with no exudate. On his chest you
see a midline sternotomy scar with a drain scar and a right thoracotomy scar.
On auscultation the lung fields are clear, but he has an ejection systolic murmur
in the left upper sternal edge which radiates to the back. He does not have a
gallop rhythm. There are transmitted upper airway sounds only on the lung fields
and the abdomen is soft with no organomegaly. What is the most appropriate
management?

A. Admit for IV antibiotics
B. Give IV furosemide and admit
C. Admit for observation
D. Send home on oral antibiotics
E. Give oral dexamethasone and observe

A

E. Give oral dexamethasone and observe

This child has undergone corretive cardiac surgery and this has resulted in pulmonary stenosis.

At this point in time the child is presenting acutely with croup, this is a viral illness and so the best treatment in this case (as in other children) is oral dex (E). This will help with the acute airway swelling and potential compromise. If his observations improve then he should return home.

There is no fever, and no exudate on the tonsils to suggest a bacterial tonsilitis, so antibiotics are not indicated (A or D).

Admitting (C) without actual treatment would not be wise.

He is not in heart failure (no gallop rhythm, no hepatomegaly, no basal crackles so a diuretic is not indicated here (B).

90
Q

A 3-day-old baby is brought to accident and emergency with acute respiratory
distress. She is tachypnoeic, tachycardic, cyanosed and her capillary refill is 5
seconds centrally. You note she has a flat nasal bridge, down sloping palpebral
fissures and epicanthic folds. On auscultation there is a loud ejection systolic
murmur at the left sternal edge. What is the most likely diagnosis?

A. Coartation of the aorta
B. VSD
C. Transposition of the great arteries
D. Tetralogy of Fallot
E. Patent ductus arteriosus

A

D. Tetralogy of Fallot

This is a presentation of a cyanotic heart disease, the presentation at three days of age with these signs of circulatory compromise marks this as a duct-dependent congenital heart defect. .

The closing of the ductus arterious means that there is no compensatory flow to account for defects such as ToF (D), pulmonary atresia, tricuspid atresia, transposition of the great arteries (C), and totally anomalous pulmonary venous drainage. As such when the duct fully closes, they will become cyanotic (they will turn blue)>

The ductus arteriosus needs urgent reopening with a prostaglandin infusion.

As such, this case is not a patent ductus arterious (E), and is in fact, presenting acutely because of it closing.

Coartation of the aorta (A) does not cause cyanosis centrally.

a VSD (B) will produce a left to right shunt, so is not cyanotic. The exception is when this is untreated for many years and pulmonary stenosis develops (usually as tenagers) and this leads to an increase in right sided pressure, and then it becomes a right to left shunt, this is known as Eisenmenger’s syndrome.

In the case we have here, there are several features hich point towards a diagnosis of Down’s syndrome. This syndrome is associated with VSD, ASD, and ToF (D), which is in fact the answer in this case.

91
Q

You are doing a baby check on the post-natal ward on a baby who is 23 hours old.
His mother tells you that he is not feeding well. On examination he is unsettled
with a respiratory rate of 76 and a heart rate of 182. You think his hands and feet
look blue and there is a soft systolic murmur heard at the left upper sternal border.
You ask the midwives to check his saturations which are 85 per cent in air and
start some oxygen. You explain to the mother that he needs to be managed on the
neonatal unit. What is the next step in your management?

A. Stop the oxygen as this may drive the closure of the ductus arterioles
B. Give prostaglandin intravenously to open the duct while organizing an
echocardiogram
C. Give antibiotics and prostaglandin intravenously while organizing an
echocardiogram
D. Give indomethacin intravenously to open the duct while organizing an
echocardiogram
E. Give indomethacin and antibiotics intravenously while organizing an
echocardiogram

A

C. Give antibiotics and prostaglandin intravenously while organizing an
echocardiogram

Here we have a blue baby with a murmur, the correct approach here is to treat them as a duct-dependent anomoly until shown otherwise.

The most important thing here is to keep the duct open, this is achieved with the use of IV prostaglandin. We also need to find out what cardiac anatomy we have in this case, so an echo is needed. Finally we have to appreciate that a septic baby could present mottled and with a flow murmur, until the echo there is not a rapid way of ruling ot sepsis, so antibiotic cover is the last part of the management plan (C). This is why option (B) is incorrect, it neglects to cover in case of sepsis.

There is no reason to stop the administration of oxygen (unless the cardiologist specifically tell you to), as such (A) is wrong.

Indomethacin (D and E) is used to close a patent symptomatic duct, so is the wrong choice here.

92
Q

A 7 year old with a 3-day history of upper respiratory tract infection is brought to
accident and emergency by his mother because he suddenly went pale and sweaty and seems to be working hard to breath. The triage nurse calls you to see him urgently because his heart rate is 200 beats per minute. You take him round to the resuscitation area, give him oxygen and connect him to the cardiac monitor. The electrocardiogram (ECG) shows a narrow complex tachycardia with a rate of 180 beats per minute. He remains alert, with a respiratory rate of 40. What is the most appropriate initial diagnosis?

A. Supraventricular tachycardia (SVT)
B. Wolff–Parkinson–White syndrome
C. Ventricular fibrillation
D. Atrial fibrillation
E. Ventricular tachycardia

A

A. Supraventricular tachycardia (SVT)

This child has a heart rate of 180 with narrow complex rhythm, this tells you that the impuls is being conducted along the physiological route, as oppossed to a rhythm being generated within the ventricles.

Knowing this we can exclude a ventricular tachycardia (E) as that would have wide complexes with no identifiable p waves. Equally ventricular fibrillation (C) can be excluded as that would be fast wide, rather chaotic complexes.

AF (F) would have no discernable p waves

All we can say for certain with this child is that he has a supra-ventricular rhythm (A), which could be a result of an accessory pathway, such as WPW (B) or it could be due to AF (D), but the question here asks what the most appropriate initial diagnosis is, and that is the broader term of SVT.

93
Q

What is the first step in management for the child in the previous question?

[alert and presenting with a supra-ventricular tachycardia]

A. Non-synchronized shock
B. Adenosine
C. Adrenaline
D. Vagal manoeuvres
E. Synchronized shock

A

D. Vagal manoeuvres

As the child is alert, the first step should be to perform vagal manoeuvres (D), examples incude; getting the child to perform a valsalva manuveour for 20 seconds (blowing up a syringe), unilateral carotid sinus massage, or inducing a mammalian diving reflex (through immersing the head in ice water). During these manoeuvres the child should be connected to cardiac monitoring to assess the effectiveness of them.

If these vagal manoeuvres fail, then adenosine (B) is the next treatment option.

Only if adenosine fails, would you lookto perform a synchronised shock (E) under sedation. If the child loses conciousness then you wouldn’t wait for the sedation.

Adrenaline (B) would only be used if the child lost cardiac output.

non-synchronised cardioversion (A) would only be used in VF or pulseless VT. THis is because in other situationsit can cause VF if te shock arrives at the same time as the QRS complex

94
Q

A 2-year-old child is referred to hospital by the GP after his third visit that week;
he now has a rash and the GP is worried he has meningitis. He has had a fever
for 5 days up to 39.5°C or above every day and is not eating or drinking well. On
examination, he has a temperature of 38.5°C, heart rate of 150, respiratory rate of
30 and is miserable. He has a blanching macular rash on his torso, swollen hands
and feet, red eyes, red cracked lips, large tonsils with no pus, and a left-sided
2 cm × 3 cm cervical lymph node which is mobile. There is no photophobia or neck
stiffness. His chest is clear with normal heart sounds and his abdomen is soft with
a palpable liver edge. You note his BCG scar is inflamed. What is the most likely
diagnosis?

A. Viral tonsillitis
B. Bacterial tonsillitis
C. Meningitis
D. Hand, foot and mouth disease
E. Kawasaki’s disease

A
95
Q

A 14-year-old refugee from Afghanistan who has lived in the UK for 2 years comes
to see you complaining of increasing fatigue and breathlessness on exertion. On
examination she appears cyanosed and has bilateral basal fine crepitations and a
soft pansystolic murmur with a displaced apex beat. She has never been in hospital and has no surgical scars. You urgently refer her for a cardiology review. What is the most likely diagnosis?

A. Bacterial endocarditis
B. Tetralogy of Fallot
C. VSD producing a left-to-right shunt
D. Eisenmenger’s syndrome
E. Ebstein’s anomaly

A

D. Eisenmenger’s syndrome

This child has likely got a large VSD that has remained uncorrected, this had lead to Right ventricular hypertrophy and failure, she now also has pulmonary hypertension and a right-to-left shunt causing the cyanosis and symptoms of hypoxia. This constellation of symptoms is Eisenmenger’s syndrome (D).

There are no infective symptoms to suggest endocarditis (A).

An undiagnosed ToF (B) would not have survived to the teenage years.

A VSD causing a left to right shunt (C) was the origional pathology in this case during infancy. It would not have caused cyanosis until it reversed.

Ebstien’s anomoly (E) is a combination of an abnormal tricuspid, hypoplastic right ventricle and pulmonary stenosis. The child would not have survived to be a teenager without corrective surgery. .

96
Q

The major criteria for rheumatic fever include all of the following features except?

A. A new murmur
B. Swollen right knee for the past 8 days
C. A geographic shaped rash with central pallor on the abdomen
D. Involuntary movements of the arms
E. Fever

A

E. Fever

Funnily enough, fever (E) is only a minor criteria of rheumatic fever.

Major criteria are; pancarditis, murmur (A) or valvular dysfunction, myocarditis, heart failure, pericarditis - [rub, effussion or tamponade], polyarthritis common in the knees (B), wrists and ankles [may be fleeting and variable, it also has to last greater than a week, so not to be mistaken for a reactive arthritis], sydenhams chorea (D) [starts 2-6 months post infection], Erythema marginatum (C), subcutaneous nodules.

Minor criteria are; Fever (E), arthralgia,family or personal history of rheumatic fever, raised inflammatory markers, and ECG changes {prolonged PR interval].

97
Q

A 5-year-old child was admitted overnight awaiting surgical repair of a broken
right ankle and was noted to have a raised blood pressure consistently above
130/90 mmHg despite adequate analgesia. On examination he has a plaster on his
right foot and appears comfortable at rest. On auscultation there is a soft systolic
murmur heard at the right upper sternal edge. His femoral pulse is difficult to find,
but present bilaterally. When felt with the radial pulse, the impulse in the femoral
pulse occurs slightly later. His abdomen is soft and there are no bruits heard. The
blood pressure done in the right arm is 136/92 mmHg but the left arm gives a
reading of 124/80. What is the most likely diagnosis?

A. Normal blood pressure in the left arm with a spurious result from the right
B. Coarctation of the aorta
C. Renal artery stenosis
D. Phaeochromocytoma
E. White coat hypertension

A

B. Coarctation of the aorta

This child has the clinical signs of lower limb low blood pressue (difficult femoral pulses), high BP in the Left arm and a very high BP in the right arm. Given this picture we know that the blood pressure leaving the left ventricle is elevated but at a location between the Braciocephalic artery and the left subclavian artery there is a drop in blood pressure, there must be a stenosis at that point. This is therefore a case of coarctation of the aorta (B). This is supported by the radio-femoral delay and the systolic murmur at the aortic area (a flow murmur across the narrowed aorta).

The BP in the left arm is not normal, and the other findings bmean that (A) is incorrect.

The lack of bruits in the abdomen and the uneven blood pressures mean that this is not a renal artery stenosis (C).

A phaeo (D) would produce elevated blood pressure evenly across all limbs in an episodic fashion, there would also be other sympathetic effects reported.

White coat hypertension (E) is a diagnosis of exclusion, and with the pathology found here it cannot be the answer.

98
Q

Which of the following is not a feature of cardiac insufficiency?

A. Scattered wheeze on auscultation of the chest
B. Central cyanosis
C. Sacral oedema
D. Tachypnoea with the apex beat palpable in the 7th intercostal space just
lateral to the mid-clavicular line
E. Hepatomegaly

A

B. Central cyanosis

These can all be signs of cardiovascular disease, but in the case of cyanosis (B) it isn’t by itself a sign of heart failure. It can be a sign of lung pathology or a right-to-left shunt, but these are not signs of failure (although they may lead to it).

A scattered wheeze (A) or bi-basal crackles are signs of right heart failure leading to pulmonary oedema.

Dependent oedema (C) is seen in infants who are mostly lying down.

Tachyopnea and a displaced apex beat (D) indicate left ventricular hypertrophy

Hepatomegaly (E) is due to back pressure in the venous system leading to congestion.

99
Q

Forty per cent of children with trisomy 21 have congenital heart defects. Which of
the following is not associated with Down’s syndrome?

A. Tetralogy of Fallot
B. Atrioseptal defect (ASD)
C. VSD
D. Atrioventricular septal defect (AVSD)
E. Transposition of the great arteries

A

E. Transposition of the great arteries

of the cardiac defects in children with Down’s syndrome;

40% are AVSDs (D)

30% are VSDs (C)

10% with ASD (B)

6% with ToF (A)

Transposition of the great arteries (E) is often found together with ASDs and VSDs, but it isn’t associated with Down’s so this is the odd one out here.

100
Q

A 14-year-old girl was seen in accident and emergency following her third
collapse this year and referred to cardiology for review of a low rumbling murmur
heard at the left upper sternal edge. Her ECG in accident and emergency was
normal. Her blood sugar was 5.3 mmol/L. Urea and electrolytes were normal. The
most recent collapse occurred at school while waiting for exam results to be given
out. Previously they occurred while watching a parade all afternoon standing in
a crowded street, and at a party. On all three occasions she felt dizzy beforehand,
was unconscious for less than 10 seconds and fully alert following the episode, but
did feel nauseous. Her echocardiogram today is normal. What is the most likely
diagnosis?

A. Venous hum murmur and vasovagal syncope
B. Innocent murmur and epilepsy
C. Wolff–Parkinson–White syndrome
D. Patent foramen ovale and sick sinus syndrome
E. Neurocardiogenic syndrome

A

A. Venous hum murmur and vasovagal syncope

A low rumbling murmur heard above the nipple line, with a normal ECG, is a venous hum and is so a normal finding in children.

She has a normal cardiovascular work up so this points to vasovagal syncope (A). This is suported by the history of triggers being anxiet or prolonged standing.

There is no suggestion of an epileptic prodrome, a post-ictal phase or any fitting, making epilepsy (B) unlikely.

WPW (C) would have delta waves on the ECG

a PFO (D) would be sen on the echo and sick sinus would appear as a bradycardia on ECG.

Neurocardiogenic syndrome (E) would be diagnosed on a tilt table test, and doesn’t have this kind of prodromal dizzyness.

101
Q

1-month-old baby attends accident and emergency with a 2-day history of fever
to 38.8°C measured at the GP surgery. He has been vomiting, with no diarrhoea,
rash, cough or coryza. A clean catch urine has leukocytes +++ and ketones, no
nitrites, blood or protein. An urgent microscopy shows >200 cells/μL white cells.
What is the most appropriate course of action?

A. Discharge home with 3 days of trimethoprim
B. Admit for a course of IV antibiotics to cover a urinary tract infection (UTI)
C. Admit for a lumbar puncture, blood cultures and chest x-ray, IV
antibiotics
D. Organize an urgent DMSA scan
E. Discharge home with reassurance and advice to return if fever persists

A

C. Admit for a lumbar puncture, blood cultures and chest x-ray, IV
antibiotics

This child has a rather non-specific presentation, the vomiting could be due to infection, surgical abdomen, reflux and a raised ICP. This child is febrile, which makes us think of infective causes but any child under 3 months with fever needs a septic screen.

The septic screen involves; blood culture, urine culture, CSF culture, CXR and inflammatory markers (CRP). This means that option (C) is our answer here.

(E) is a dire option, (A) relies on the urine dip results to be accurate and that this is a UTI and not a different source of infection. Given that urinary leukocytes are not a sensitve measure in isolation, or indeed in infants, this would be a reckless course of action.

If the urine culture grows more than 105 colony forming units per ml of a single organism, then a diagnosis of UTI can be made and option (B) would be appropriate.

Imaging after a UTI is necessary in infants, be this an ultrasound or DMSA (D)(dimercaptosuccinic acid scan, a form of radioisotope scan), to assess for damage to renal morphology. This would, however, need to be done at least 6 weeks after the infection, as the images in the acute phase will be misleading due to inflammation.

102
Q

A 5-year-old boy presents to his GP with a 3-day history of puffy eyes. He
has been unwell with a coryzal illness for the last week. His mother states he
has had no new medications and no hayfever, allergies or asthma. On further
examination he has generalized oedema and scrotal oedema. He is tachycardiac
and has cool peripheries, no skin rashes or erythema. What is the most likely
diagnosis?

A. Periorbital cellulitis
B. Allergic reaction
C. Nephrotic syndrome
D. Nephrotic syndrome with hypovolaemia
E. C1 esterase deficiency

A

D. Nephrotic syndrome with hypovolaemia

Nephrotic syndrome is defined by a triad of proteinuria, hypoalbuminaemia and generalised oedema.

The generalised oedema here goes against a diagnosis of periorbital cellulitis (A), especially given teh lack of infective symptoms.

There is no skin rash, blister or wheals, and no airway/coryzal symptoms to suggest this is an allergic reaction (B)

C1 esterase deficinecy (E) causes hereditary angiodema, as this is an autosomal dominant condion you would expect some family history here. It presents usually as isolated facial and oral swelling. The generalised oedema is a rare presentation, but the systemic shock in this case makes it unlikely.

Once we know that this is Nephrotic syndrome, we only have to see how shocked this child is to know that there is a loss of fluid to the interstitium, leading to hypovolaemia (A)

103
Q

A 12-year-old girl presents to her GP with a UTI. She has no past medical history
of note and is not taking any medication. On testing her routine observations, her
blood pressure was 140/90 mmHg with a manual sphygmomanometer. You are
concerned this may be high for her age. She has no headaches, visual disturbance, vomiting, chest pain, dyspnoea or neurological signs. What is your next course of action?

A. Repeat the blood pressure on three different occasions
B. Discuss the blood pressure reading with a paediatric nephrologist
C. Commence sodium nitroprusside
D. Repeat the blood pressure measurements with an automated machine
E. Discharge home with reassurance

A

A. Repeat the blood pressure on three different occasions
Just as in adults you wouldn’t jump to managing/investigating hypertension without three seperate blood pressure readings (A). There could be many reasons why this is a anomolous BP reading for this child.

A manual reading is always prefered to an automated machine (D)

If the BP is still high on 3 occassions then you would investigate and manage, with advice from the nephrologists (B).

Sodium nitroprusside (C) is used to treat malignant hypertension, which is unlikely here.

Discharge home with no further investigation (E) is a poor call.

The causes of hypertenson in children include: Essential hypertension,renal (artery stenosis, CKD, Wilm’s tumour), Cardiac (Coarctation of the aorta), endocrine (Cushing’s. Phaeo, neuroblastoma) and metabolic (Hyperaldosteronism, Congenital adreanal hyperplasia).

104
Q

A 6-year-old girl presents to hospital with a large right-sided abdominal mass.
It does not cross the midline. On further questioning she has had macroscopic
haematuria and weight loss of 4 kg over the last 4 months. She has reduced
appetite and lethargy. Her blood pressure is 125/73 mmHg, heart rate 120 bpm.
Which of the following is not a complication of this malignancy?

A. Malnutrition
B. Hypertension
C. Renal impairment
D. Urinary catecholamines
E. Metastatic spread

A

D. Urinary catecholamines

This child’s history is consistent with a Wilm’s tumour, this tumur can result in hypertension (B) (due to renal artery stenosis), renal impairment (C), metastases (E) is always a concern with malignancy, and the anorexia common with serious disease can lead to malnutrition (A).

The prescence of catecholamines in the urine (D) is more consistent with a phaeochromocytoma, or a neuroblastoma.

105
Q

James is an 8-year-old boy who recently attended accident and emergency with a
swollen left ankle. He had an x-ray and was discharged home and told there was
no fracture. He has now developed a dark purple rash on his legs, which does not
disappear with a glass pressed on. He was brought back to the department today
vomiting, with abdominal pain. His observations and urine dipstick are all within
normal limits. What is the most likely diagnosis?

A. Diabetic ketoacidosis
B. Viral gastroenteritis
C. Meningococcal sepsis
D. Idiopathic thrombocytopenic purpura
E. Henoch–Schönlein purpura

A

E. Henoch–Schönlein purpura

HSP (E) is based of several diagnostic criteria, these are; Arthralgia, Rash (can be simple urticarial, right through to purpuric), abdo pain, and renal involvement (nephrotic syndrome or hypertension).

It is a common condition that affects 3-10 year olds, usually preceeded by a viral illness.

Although it can casue severe GI and Renal problems, it is usually self-limiting. .

Diabetic ketoacidosis (A) is always going to be a concern in a child with vomiting and abdo pain. but this child has normal observations.

A viral gastroenteritis (B) could precipitate a reacive arthritis, but this would follow the illness not be concurrent.

A purpuric rash will cause many to jump to the idea of meningococcal sepsis (C), but with the normal observations this is very unlikey. If meningococcal sepsis had progressed to having a purpuric rash this extensive the child would likely be in shock.

ITP (D) would have a rash, but the other symptoms here do not fit, there would also be a thrombocytopenia reported!.

106
Q

A 7-year-old boy presented to accident and emergency with diarrhoea and
vomiting for the past week. He had no history of foreign travel and but had been
to a zoo recently on a school trip. He was discharged home, after providing a
stool sample, with rehydration advice as he was less than 5 per cent dehydrated
and tolerating oral fluids. The stool had grown ‘Escherichia coli 0157’ which was
phoned from the microbiology laboratory to the on-call doctor 48 hours later.
What is the most serious complication?

A. Acute kidney injury
B. Haemolytic uraemic syndrome
C. Severe hypernatraemic dehydration
D. Henoch–Schönlein purpura
E. Post-gastroenteritis syndrome

A

B. Haemolytic uraemic syndrome

E.coli o157 is classically linked with visits to farms, petting zoos and slaughterhouses. It produces a particular toxin that causes a massive inflammatory reaction, resulting in a triad of; microangiopathic haemolytic anaemia, acute kidney injury, and thrombocytopenia.

The damage done by haemolytic uraemic syndrome (B) may in itself cause an aki (A) as detiled above, the child may suffer from dehydration and fluid loss leading to (C).

Post-gastroenteritis syndrome (E) may be a case of new food intolerances due to damage to the mucusoal enzyme activity in the gut (lactose intolerance is an example of this).

HSP (D) can follow an acute viral infection, but that is not the pathology here.

107
Q

A 5-year-old girl was brought to hospital at midnight by her mother with 5 per
cent partial thickness burns to her chest and abdomen. Her mother states that she
pulled on the kettle at 2 pm and the boiling water scalded her. On examination she is tachycardic, and drowsy with cool peripheries. Her initial blood tests: sodium 150 mmol/L, potassium 7.8 mmol/L, urea 10.2 mmol/L, creatinine 104 μmol/L, haemoglobin 14 g/dL. What is the most likely aetiological factor to account for these results?

A. Post-renal cause of acute kidney injury
B. Poisoning
C. Renal cause of acute kidney injury
D. Dehydration
E. Pre-renal cause of acute kidney injury

A

E. Pre-renal cause of acute kidney injury

This girl suffered a scald injury some 10 hours ago (alarmbells for NAI should be ringing). Since that time she has suffered severe dehydration due to the compromise of the barier function of the skin. She has now moved beyond dehydration (D) and is showing signs of shock and acute kidney injury. This is best charecterised as a pre-renal failure due to hypovolaeimia (E).

The damage may later lead to acute tubular necrosis, a renal cause (C), but at this time it’s all about the pre-renal.

Causes of post-renal failure (A) include kidney stones and bladder obstruction, physical and neurological.

Poisoning (B) should be considered, especially in this context of potential NAI and the presentation of a drowsy child. But the results here point to an acute kidy injury as the most obvious pathology.

108
Q

A 10-day-old baby boy was brought to accident and emergency with a distended
abdomen. On questioning, he was born at term with no antenatal concerns. Until 2
days ago he had been feeding well and not vomiting, he had been wetting nappies, but mother has not witnessed a good urinary stream. On examining the child, you find a mass, dull to percussion, arising out of the pelvis, and he has had no wet nappies for the last day. You suspect he may have posterior urethral valves. Which one test will help to diagnose this underlying condition?

A. DMSA scan
B. Renal biopsy
C. Computed tomography (CT) abdomen
D. Micturating cystourethrogram
E. Renal ultrasound

A

D. Micturating cystourethrogram

In this case there is a presentation of a large palpable bladder cmbined with 24 hours of anuria. THis picture is highly suggestive of a bladder outflow obstruction.

Given this information and the poor urinary flow, the protagonist of our tale has hypothesised that this child has posterior urethral valves.

To confirm this diagnosis you need to conduct a dynamic imaging study of the bladder and urethra, the test of choice is a micturating cystourethrogram (D).

A DMSA scan (A) and renal biopsy (B) would be useful for looking at the kidney

A CT scan (C) would be unlikely to adequately show the urethral valves, it also lacks the dynamic flow visualisation.

A renal ultrasound (E) would be a reasonable imaging technique in this scenario, but it is inferior to the cystourethrogram.

109
Q

A 12-year-old boy who was born with multicystic dysplastic kidneys. He had a
renal transplant when he was 7 years old due to chronic kidney disease stage V
after having peritoneal dialysis for 1 year. Which of the following would you not
expect him to be taking?

A. Septrin
B. Tacrolimus
C. Diclofenac
D. Growth hormone
E. Erythropoietin

A

C. Diclofenac

Post transplantation this child is going to need to be on several drugs, broadly speaking these will be immunosuppresive drugs to prevent rejection of the transplanted organ, Tacrolimus (B) here, and antibiotics to act as prophylaxis, Septrin (A) here.

With a renal transplant there are some special considerations, the child may have impaired growth,and need GH (D) supplementation, and as the kidney is the site of EPO (E) production this may be deficient and need an endogenous source.

The odd one out here is Diclofenac (C, it is a NSAID, and like all NSAIDs, it is nephrotoxic and can cause harm to a patient with any renal impairment.

110
Q

An 11-year-old girl presents to the out of hours GP while on holiday in England
with abdominal pain. She tells you she has polycystic kidney disease which was
diagnosed early in life. She has bilateral palpable kidneys and hepatosplenomegaly, with visible distended veins on the abdomen and ascites. Abdominal ultrasound shows liver fibrosis. What is the inheritance of this condition?

A. Autosomal dominant
B. X-linked
C. Sporadic mutation
D. Autosomal recessive
E. Microdeletion

A

D. Autosomal recessive

There are two types of polycystic kidney disease, there is an autosomal dominant disease (A) which presents in older children or adults, it involves cysts composed of grossly dilated nephrons which compress the normal renal architecture.

This girl’s diagnosis was early in life, which is more consistent with the autosomal recessive (D) type. This involves bilateral renal involvement, with masses, respiratory distress due to pulmonary hypoplasia, and congenital hepatic fibrosis and pulmonary hypertension. The pathology involves tubular dilation of the distal collecting system, it is progressive, with renal function deteriorating over several years until replacement therapy is needed.

111
Q

An 11-year-old girl was brought to accident and emergency in December with pain
in her left leg. She is known to have sickle cell disease and her baseline haemoglobin is 7.0 g/dL. She has been admitted in the past with painful leg and chest crises. She has a cough and coryza. Today her blood results show: haemoglobin 6.8, white cell count (WCC) 12 × 109/L, platelets 209 × 109/L, C-reactive protein (CRP) 20 mg/L. What is not part of the appropriate initial management?

A. IV fluids
B. 15 L oxygen through a non-rebreather mask
C. Exchange transfusion
D. IV antibiotics
E. Oramorph

A

C. Exchange transfusion

This girl is having a painful crisis as the small blood vessels supplying the bones in her leg have become occluded. This may well be due to the cold (it is december in this scenario), dehydration, hypoxia or illness.

The management of a painful crisis is to provide pain relief, such as oramorph (E), and then correct any precipitating causes of the painful crisis (A, B, and D) as well as warming the patient.

If there is a marked drop in Hb then a top-up transfusion may be indicated.

An exchange transfusion (C) is only indicated for priapism, chest crisis, and stroke.

112
Q

A 4-year-old boy is brought to accident and emergency with a limp for 1 day. He
was unhappy to weight bear on his right leg. He had been with his grandparents
all day and his mother brought him to hospital when she returned from work that
evening. He was afebrile with a heart rate of 110 bpm but had had a cold last week.
Mum reports no history of trauma. What is the most important diagnosis to exclude?

A. Behavioural
B. Acute leukaemia
C. Reactive arthritis
D. Soft tissue injury
E. Septic arthritis

A
113
Q

A 14-year-old girl was diagnosed with idiopathic thrombocytopenic purpura (ITP)
last week after she attended the children’s assessment unit with recurrent epistaxis. She had a platelet count of 16 × 109/L last week and now re-presents to accident and emergency with further episodes of epistaxis, haematemesis and petechiae. She had a heart rate of 110 bpm and her blood pressure is 100/70 mmHg. What is the next best management step?

A. Give a platelet transfusion and red cell transfusion
B. Arrange an urgent upper gastrointestinal endoscopy
C. Give IV immunoglobulin and steriods
D. Admit and monitor the haemodynamic status and administer a fluid bolus
E. Discharge home with advice to return if the symptoms continue for
more than 24 hours

A

D. Admit and monitor the haemodynamic status and administer a fluid bolus

This girl has typical signs of ITP, her low platelet count and mucosal bleeding. It can occur a few weeks after a viral illness, as in this case., and it vrought about by an autoimmune destruction of platelets. It occurs most frequesntly in children aged 2 to 10, and is a self limiting condition, but supportive management is somethimes needed.

In this case the girl is demonstrably hypovolemic and cannot simply be discharged (E). As is the normal practice with a haemodynamically unstable patient the first step is to give a fluid bolus and monitor for it’s effect (D).

Donor platelets (A) would just be destroyed in the same auto-immune process.

An endoscopy (B) would be indicated if we suspected an upper GI bleed, but given the history we believe the reason for the bleeding is the low platelets, there is also the increased risk of further bleedng due to the procedure.

IV Ig and steroids (C) would be indicated if the child continued to bleed , but before this step you would want a bone marrow aspirate to esclude a malignant cause.

114
Q

A 14-year-old girl went to her GP with a sore throat and cervical lymphadenopathy.
She had a blood test done and you are called later that day with results. Haemoglobin 6.0 g/dL, WCC 230 × 109/L, neutrophils 0.9 × 109/L, platelets 77 × 109/L; blood film showed blasts and Auer rods. What is the most important management priority for this child in the first 24 hours from diagnosis?

A. Overwhelming sepsis
B. Febrile neutropenia
C. Heart failure
D. Uncontrollable bleeding
E. Tumour lysis syndrome

A

E. Tumour lysis syndrome

The prescence of Auer rodsin the blood film tells you that this girl has Acute myeloid Leukaemia. As a result of the condition she has a very high WCC and a reduction in the other cell lines.

This anaemia can result in heart failure (C), the low platelets can cause bleeding (D), and the neutropenia can lead to infection (B) and sepsis(A), the most important consideration is tumour lysis syndrome (E).

As treatment for AML begins there will be a rapid destruction of cells, this will result in the release of electrolytes, urea and lactate. Prior to commencing treatment there needs to be prophylactic hyperhydration, an anti-urate drug (allopurinol or rasburicase) and electrolyte monitoring.

115
Q

A 2-year-old boy is admitted to the paediatric ward with a swollen, painful left knee. He has been afebrile and has a history of minor trauma to his knee earlier today. His mother is a haemophilia carrier and his father is not affected. You are keen to rule out haemophilia in this child. Which two clotting factors should you test for?

A. Factor VII and IX
B. Factor VII and VIII
C. Factor V and VI
D. Factor VIII and IX
E. Factor X and XI

A

D. Factor VIII and IX

Haemophillia A is due to deficiency in factor VIII, Haemophillia B is due to a deficiency in factor IX.

These conditions are -linked and they can be of varying severity depending on how much function remains.

treatment revolves around replacemnt clotting factors, desmopressin to stimulate clotting factor production, special consideration of clotting prior to any surgery, and physiotherepy for arthritic joints.

116
Q

A 20-year-old man presents to the infectious diseases department with a large
7 cm × 8 cm swollen painful lump in the left anterior triangle of his neck. He has
night sweats, 10 kg weight loss and a dry cough for the last month. He was treated
with surgery and radiotherapy for a high grade astrocytoma when he was 8 years
old. Which of the following is not a recognized complication of his childhood
condition and its treatment?

A. Finger clubbing
B. Haematological malignancy
C. Educational difficulties
D. Short stature
E. Infertility

A

A. Finger clubbing

This question is all about the long term effects of radiotherapy and chemotherapy, the big concern is seconday malignancy due to high levels of radiation, eg Leukaemia and Lymphoma (B).

The long periods of missed school or impaired ability to concentrate, hearing or visual loss can all contribute to educational difficulties (C).

The radiotherapy can damage the spine or reduce the ability of the pituatary to produce growth hormone, leading to short stature (D).

Both radio and chemotherapy can lead to gonadal dysfunction (E).

Clubbing (A) is not caused by radiotherapy or chemotherapy, it is a consequence of many lung, cardiac and GI pathologies.

117
Q

A 12-year-old girl has been seeing her GP for the last year with heavy periods and had suffered with bleeding gums when she was younger. She is otherwise well and lives with her adoptive parents who now have parental responsibility. Her coagulation tests reveal normal prothrombin time (PT) and activated partial thromboplastin time (APTT), low factor VIII, low von Willibrand factor (vWF), abnormal platelet aggregation and increased bleeding time. What is the likely inheritance of her condition?

A. Autosomal dominant
B. Autosomal recessive
C. X-linked
D. Robertsonian translocation
E. Sporadic mutation

A

A. Autosomal dominant

We see here that this girl has von Willebrand’s disease (vWD) and has the typical presentation of unusual bleeding. The blood tests show that her vWF is low but not absent, this tells us that she has vWD type 1 (a partial deficiency of vWF), which is inherited in an autosomal dominant (A) fashion.

Type 2 is defective vWF, and is also autosomal dominant

Type 3 is a total abscence of vWF and is autosomal recessive

X-lnked (C) would be the haemophillias

A robertsonian translocation (D) is the fusion of two chromosomes at the centromere with teh loss of the short arms. This can lead to Down’s syndrome or loss of the pregnancy.

Sporadic mutations (E) are not linked with vWD, but are associated with the majority of cases of Achondroplasia

118
Q

A 4-year-old girl has just returned from holiday in France where she visited a petting farm. She has had diarrhoea for 2 days, and her mother noticed fresh red blood mixed with the stools. She has also been vomiting. On admission to hospital her blood tests showed: Hb 5 g/dL, WCC 15 × 109/L, platelets 55 × 109/L, urea 19 mmol/L, creatinine 110 μmol/L. Her stool culture is pending. What is the most likely diagnosis?

A. Platelet disorder
B. Inflammatory bowel disease (IBD)
C. Severe dehydration
D. Henoch–Schönlein purpura (HSP)
E. Haemolytic uraemic syndrome (HUS)

A

E. Haemolytic uraemic syndrome (HUS)

The history here of contant with farm animals is quite a classic EMQ lead in for heamolytic uraemic syndrome (E), caused by strains of E.coli (o157) and Shigella gastroenteritis, where an enterotoxin is produced that creates a massive inflammatory response.

Treatment is usually supportive, but renal replacement therapy may be needed.

A platelet disorder (A) could explain the bleeding we see here, but doesn’t explain the renal impairment.

IBD (B) is unlikely in a child this young, and would present with abdo pain, weight loss, bloody diarrhoea and anaemia. You would also expect a more chronic picture than the two day history in this case.

Severe dehydration (C) may lead to the renal impairment we see here (pre-renal AKI) but it doesn’t explain the haemotological results here.

HSP (D) is a array of symptoms, including; arthralgia, a purpuric rash, abdominal pain, and renal compromise. There may be bloody diarrhoea due to vasculitis in the GI tract and renal failure, but this is not a hallmark of HSP. Given that we have no other symptoms of HSP, it seems unlikely.

119
Q

An 18-month-old boy presented to the GP with a history of eating soil. He had
been in the garden this afternoon as his mother put the washing out. She found
him eating the soil and took him straight inside. On examination, he is well and
alert but has pale conjunctivae. He is not tachycardic or tachypnoeic. His diet
consists of predominantly of breast milk. What is the most likely result of his
haemoglobin and haematinics?

A. Hb 10 g/dL, MCV 80 fl, ferritin normal, iron normal, vitamin B12 and folate normal
B. Hb 6.5 g/dL, MCV 100 fl, ferritin normal, iron normal, vitamin B12 and folate low
C. Hb 5.5 g/dL, MCV 55 fl, ferritin low, iron low, vitamin B12 and folate normal
D. Hb 7 g/dL, MCV 70 fl, ferritin normal, iron normal, vitamin B12 and
folate normal
E. Hb 6.8 g/dL, MCV 65 fl, ferritin normal, iron low, vitamin B12 and folate normal

A

C. Hb 5.5 g/dL, MCV 55 fl, ferritin low, iron low, vitamin B12 and folate normal

This child is going to be profoundly iron deficient, hence the pica (eating of non food items ) we see here. The pale conjunctiva support this supposition.

Given the clinical picture of a severe iron deficiency anaemia you would expect a microcytic anaemia with low iron stores (ferritin) as well as low circulating iron, this is consistent with option (C).

This situation has likely arisen due to prolonged breastfeeding and delayed weaning.

of the other results here;

(A) is a normal result

(B) is a macrocytic anaemia, likely due to the folate and B12 deficiency

(D) is a microcytic anaemia with normal iron, suggesting something like a thalassemia or sideroblastic anaemia.

(E) is a macrocytic anaemia with normal folate and B12, suggestive of hypothyroid, liver disease or a myeloproliferative disorder

120
Q

A 9-month-old boy presented to his GP with lethargy and a prominent forehead. He is pale on examination and has yellow sclerae. He is the first child of his non-consanguineous parents. His haemoglobin is 6.5 g/dL, WCC 5.0 × 109/L, platelets 300 × 109/L. His blood film shows evidence of haemolysis, no spherocytes, no sickle cells and a good reticulocyte count. Direct antiglobulin test (DAT) is negative. What is the most likely diagnosis?

A. Beta thalassaemia
B. Sickle cell disease
C. ABO incompatibility
D. Hereditary spherocytosis
E. G6PD deficiency

A

A. Beta thalassaemia

The question stem tells you that there are no spherocytes or sickle cells, tis effectively excludes (D) and (B).

This child is too old to be presenting with an ABO compatibility issue (C), there would also likely be a positie result from the DAT test.

So we are left with to options, of these, G6PD deficiency (E) would present witht eh jaundice and anaemia seen here, but it is an episodic comdition precipitated by an illness or a dieteray precipitant. The fact that this child has extra-medullary haemopoesis (frontal cranial bossing), we can infere that the process here is severe and chronic. This means that beta thalassaemia (A) is the most likely diagnosis here.

Beta thalassaemia is an autosomal recessive haemolytic anaemia that presents after 6 months of age as the body starts to produce haemoglobin A (made of 2 beta and 2 alpha chains), which in beta thalassemia cannot occur.

121
Q

A 3-year-old is brought into accident and emergency on a Monday morning
because she has developed several bruises on her buttocks, left leg and right arm.
She is seen with her nanny who reports finding the bruises when she was getting
her dressed this morning. Recently the girl has not been herself. She has had
several colds over the past 2 months and has been more lethargic lately. The nanny is worried she is losing weight. On examination she appears withdrawn, pale and has a bruise on the left buttock which is 5 cm × 8 cm. She has three other bruises on her left leg and right arm which are of varying colours. She also has some fine petechiae on her neck and cheeks. She has a runny nose and a cough but the chest is clear. What is the most likely diagnosis?

A. Non-accidental injury
B. Leukaemia
C. Idiopathic thombocytopenia
D. Henoch–Schönlein purpura
E. Accidental injury

A

B. Leukaemia

This presentation of widespread bruising and petechiae should make you worry about a low platelet count. Of the options that we have here only two of them (B and C) would cause a thrombocytopenia.

This child has ‘B’ symptoms such as lethergy and weight loss, which points us towards Leukamia (B).

HSP (D) does not cause a thrombocytopenia but could produce the lower limb bruising, there would also be abdominal pain, haematuria and arthralgia, which there is not here.

ITP (C) is also ususally a post-viral condition, it wouldn’t usually present concurrently with the period of illness.

NAI (A) would be a concern if the blood results were normal.

Accidental injury (E) is unlikely with brusing of different ages that is sited on the buttocks, the petechiae on the neck and face are also incongruent with this mechanism.

122
Q

On the day 1 baby check a mother is very concerned about a rash on her baby’s face. Over the right eye, forehead and temple there is a pink-red, flat area of erythema. He is opening the eye, and his eye movements seem intact. The child’s observations and rest of the examination are normal. What should you tell the mother?

A. This is a strawberry naevus and it may get bigger before it goes away
by about 5 years of age
B. This is a port wine stain and the baby needs an MRI scan to check for
intracranial involvement
C. This is a capillary haemangioma (stork mark) and is normal; it will fade
over the first year or so of life
D. This is orbital cellulitis and he needs intravenous antibiotics
E. This is erythema toxicum which is a normal baby rash and will go away
within the first few weeks

A

B. This is a port wine stain and the baby needs an MRI scan to check for
intracranial involvement

A port wine stain (naevus flammeus) (B) is a flat erythemotous area caused by a vascular malformation in the dermis. It is present from birth and will persist for life.

In this particular case it is present in a trigeminal distribution which means there is a risk of Sturge-Weber syndrome, where there is intracranial involvement that could lead to seizures and developmental problems.

A strawberry naevus (A) (cavernous haemangioma) is not normal present from birth, they will grow larger before they outpace thier blood supply and shrink down and disappear, usually before the age of 5 years. They can interfere with neighbouring structures, but crucially they are not a flat lesion.

A capillary haemangioma (C) is a pink macule found on the nape of the neck, eyelids, or central forehead. They are caused by distended dermal capillaries, and mostly fade over the first year. They are not found on teh lateral face.

This is a well child with normal observations, Cellulitis (E) seems unlikely in that respect.

Erythema toxicum (E) would be pustular, it would come and go all over the body for the first few weeks of life. The morpholoy of the rash is not correct for that condition.

123
Q

A mother brings her 6-month-old, formula-fed baby to see the GP complaining
that the olive oil she is using is not helping his persistent cradle cap and worsening rash on his face and arms. On examination he has extensive cradle cap and eczematous changes on his cheeks, neck, chest and arms. The neck skin creases are red and oozing with yellow crusts. He is miserable and feels warm to touch. What is the most appropriate management?

A. Advise using emollients and a soap substitute
B. Start emollients with a topical antibiotic
C. Refer to hospital for intravenous antibiotics
D. Recommend a trial of switching to soya based formula as he may be
cow’s milk protein allergic
E. Start topical steroids on the inflamed areas, and intensive emollient
treatment

A

C. Refer to hospital for intravenous antibiotics

This child has some very badly infected eczema, the likely causative organism here Staphylococcus (yellow crusting), but there is a possibility this is eczema herpeticum.

In this scenario he is systemically unwell with a likely fever and he needs IV antibiotics, and assesment in hospital (C).

If he was not unwell with the skin infection then oral antibiotics and emolients would be a good management plan, but using topical antibiotics (B) will just create an enviroment to select for resistant strans.

Exploring contributory factors such as cow’s milk allergy (D) would be wise, but this is not a step for the immediate management here.

Emollients and soap substitutes (A) will also form part of the ongoing management, but it is not our first step.

Steroid creams (E) should not be used on infected skin as these have an immunosuppresive effect.

124
Q

A 4-year-old is brought into accident and emergency by very anxious parents.
She has had a bad cough which makes her vomit and a fever for 2 days. She has
now developed a rash on her face which does not pass the ‘glass test’, in that the
spots are still visible when a glass is pressed against the skin. On examination she
is alert and comfortable at rest, with fine petechiae on her cheeks and neck which
are non-blanching. She has red, enlarged tonsils without pus and the chest is clear. What is the most likely cause of her rash?

A. Meningococcal sepsis
B. Idiopathic thrombocytopenia
C. Henoch–Schönlein purpura
D. Non-accidental injury
E. Capillary rupture secondary to raised pressure in the superior vena cava
distribution

A

E. Capillary rupture secondary to raised pressure in the superior vena cava
distribution

The parents have done the ‘glass test’ and are now concerned this means meningococcal sepsis (A), given that this girl is well, with no signs of shock or meningism this is unlikely. It would be prudent to admit her for observation though.

The distribution of the petechiae are in the area of the SVC drainage, this supports the diagnosis of capillary rupture (E), secondary to vigorous coughing and vomiting. This type of capilliry rupture is also seen in babies who have been shaken, or stragulation injuries.

The coughing here could very likely be the cause of the vomiting, but as it could be a sign of meningococcal sepsis, then that supports that this child should be kept in for monitoring.

ITP (B) is a complication that can arise about a week after a throat infection, it is unlikely to appear at the same time as the illness, as in this case.

As mentioned above, this pattern of injury could be due to a strangulation injury (D), but there is a more likely explaination here.

HSP (C) is a vasculitis of the small vessels and will cause a purpuric macular-papular rash on the buttocks and extensor limb surfaces. This doesn’t fit with this case, there would also be a renal compromise , arthralgia and abdo pain.

125
Q

A 2 year old was seen in accident and emergency by the senior house officer with
a short history of fever, malaise and now vomiting. She had a blanching rash on
her arms and abdomen. She looked unwell but had no clear focus for her fever.
She was tachypnoeic but her chest was clear. A urine sample was requested which showed a trace of leukocytes and two plus of ketones. Forty-five minutes later the paediatric registrar came to review the child who appears lethargic with a capillary refill centrally of 6 seconds and the rash on her abdomen is now non-blanching. What is the most likely diagnosis?

A. Urinary tract infection (UTI)
B. Idiopathic thrombocytopenia
C. Meningococcal sepsis
D. Human herpes virus 6 infection
E. Diabetic ketoacidosis

A

C. Meningococcal sepsis

The slow capillary refil time combined with the expanding rash that is becoming purpuric are alarming signs for DIC a component of meningococcal sepsis (C).

ITP (B) is a condition ususaly seen about a week following an infection in well children. It doesn’t present in a background of a profoundly unwell child.

The leukocytes are non-specific for a UTI (A) especially without nitrites in the urine.

HHV6 (D) would cause a febrile illness with a rash, but it would not be purpuric and the child would not be shocked.

DKA (E) is not indicated here as there is no evidence on the urine dip of hyperglycaemia.

126
Q

A 2 year old is brought in by ambulance after pulling a pot of boiling water off
the stove down on top of himself. He has significant burns to the whole of his face,
chest and right arm. Estimate the percentage body surface area affected.

A. 20 per cent
B. 30 per cent
C. 40 per cent
D. 50 per cent
E. 60 per cent

A

With paediatric burns, it is useful to remember the rule of 9’s

Each arm is 9%, the chest is 2x9% (18%) and the back us another 18%

The head is another 18%

The remaining 28% is the legs (doesn’t really fit with the rule, sorry).

Using this rule in this case; 9% for the face, 18% for the chest, and 9% for the right arm = 36% which is closest to (C)

A burn of 50% (D) or 60% (E) would be very extensive.

127
Q

Which of the following is not a cause of erythema nodosum?

A. Oral contraception
B. Tuberculosis infection
C. Hepatitis B infection
D. Streptococcal infection
E. Sarcoidosis

A

C. Hepatitis B infection

Erythema nodusum is an inflammatory condition affecting the subcutaneous adipocytes, usually appearing as painful nodules on both shins.

causes of erythema nodusuminclude; the oral contraceptive pill (A), TB (B), Streptococcal infections (D), sarcoidosis (E), and inflammatory bowel disease.

Hepatitis B (C) causes non-specific rashes and jaundice.

128
Q

A 3 year old is brought to see the GP with multiple pearly raised papules with central

umbilications. They have been there for more than a month on his torso and upper
legs. His mother is worried he has warts. What is the most likely diagnosis?

A. Molluscum contagiosum
B. Congenital warts
C. Scabies
D. Melanocytic naevi
E. Guttate psoriasis

A

A. Molluscum contagiosum

Tis child has a the common presentation of Molluscum contagiosm (A), the description of a pearly papule with an umbilicated center is absolutely classic. They are cause by teh pox virus and are self limiting and will resolve within a year.

If they are particularly stubbon they may need cryotherapy, they can also become secondarily infected with a bacterial infection which needs treating, but that isn’t the case here.

Congenital warts (B) is a term for genital or anal warts, not consistent with the distribution here. They can take up to 2 years to apear after the vertical transmission, this may lead to icorrect suspicion of child sexual abuse.

Scabies (C) is a burrowing parasite that usually affects the groin, axilla and classically the hands, and finger web spaces.

A melanocytic naevi (D) would be hyperpigmented and would not have a central punctum.

Guttate psoriasis (E) can follow a URTI and produces scaly pink patches on the torso and arms.

129
Q

On a newborn baby check of an Asian, 36-hour-old baby you note a large bruise
coloured area on the buttocks and lower back which seems non-tender. The mother does not know how it got there. He is handling well and the rest of the baby check is unremarkable. What is the most likely explanation?

A. Non-accidental injury
B. Mongolian blue spot
C. Neonatal sepsis with disseminated intravascular coagulation
D. Idiopathic thrombocytopenic purpura
E. von Willebrand’s disease

A

B. Mongolian blue spot

A mongolian blue spot (B) is a congenital birthmark, usually found on the lower back and buttocks and it will fade as the child ages.

The fact that the child isn’t bothered by it goes against this being an injury (A), The child would be much more unwell if this was DIC on a background of sepsis (C).

ITP (D) follows an infection, so cannot occur in newborns.

vWD (E) would present wih mucosal bleeding.

130
Q

A 5-day-old baby is brought to see the GP because she has had a rash for the past
3 days which started on her chest, is spreading to her face and getting worse. On
examination she handles well and is alert. There is an erythematous rash on her
face, torso and right arm with little pustules. What is the most likely diagnosis

A. Infected eczema
B. Neonatal sepsis
C. Neonatal acne
D. Molluscum contagiosum
E. Erythema toxicum

A

E. Erythema toxicum

The erythemotous rash with pustules in the context of a well newborn is a classic presentation of erythema toxicum (E). This is a benign condition that comes and goes during the first few weeks of life.

An infection (B) or (A) is unlikely in a well baby.

Neonatal acne (C) would be combined to the face.

Molluscum contagiosum (D) would be pearly papules with an umbilicated center. There would not be erythema or pustules.

131
Q

A 10-year-old girl was diagnosed with diabetes 1 year ago. She has been compliant
with her insulin regimen and her HBA1c is 6 per cent. She is attending her annual
diabetic review and has been asking about why she has diabetes. What is the
aetiology of type 1 diabetes?

A. Inflammation of the pancreas causing exocrine and endocrine
dysfunction
B. Impaired glucose tolerance
C. Secretory dysfunction of the pancreatic duct
D. Autoimmune destruction of pancreatic islet cells
E. Peripheral insulin resistance

A

D. Autoimmune destruction of pancreatic islet cells

The other processes here describe the pathology of other diseases;

A. - chronic pancreatitis
B. Corticosteroid use (Cushing’s)
C. Cystic Fibrosis
E. Along with reducedproduction of insulin - Diabetes Mellitus II

132
Q

A 2-year-old girl was brought by her mother to accident and emergency after
4 days of vomiting and abdominal pain. She had brought her in 2 days ago
after developing a cold and was discharged home and diagnosed with a ‘tummy
bug’. On examination, she was drowsy, had dry mucous membranes, deep heavy
breathing, cool peripheries and tachycardia. Her mother reports a 1-month history
of weight loss, excessive drinking and passing large volumes of urine prior to
this episode. Her urine dipstick has ketones and glucose. Her blood gas shows the
following: pH 7.10, PCO2 kPa 2.1, PO2 kPa 10.0, BE −12, HCO3
– mmol/L 18. What is
the most likely diagnosis?

A. Severe dehydration secondary to gastroenteritis
B. Sepsis secondary to gastroenteritis
C. Diabetic ketoacidosis (DKA)
D. Chronic kidney disease
E. Hyperosmolar hyperglycaemic non-ketotic state

A

C. Diabetic ketoacidosis (DKA)

We have a classic presentation of diabetic ketoacidosis in this case. She has developed polyuria, polydipsia and has weight loss. A viral illness has then precipitated this ketoacidotic state (C). The presentation of vomiting and abdominal pain is a common one. It needs to have three criteria to be diagnosed, that we see here, there needs to be hyperglycemia, acidosis, and the prescence of urinary ketones.

Gastroenteritis (A and B) could cause all these symptoms, except the glycosuria.

Chronic kidney disease (C) could explain the polyuria but is not a good fit in terms of an acute episode and the other symptoms of diabetes.

HONK (E) is a complication of diabetes type I, it is rare in childhod and does not produce ketonuria.

133
Q

A 3-week-old baby is brought to the ‘prolonged jaundice clinic’. His mother
reports he has poor feeding, is not gaining weight appropriately and is more sleepy compared to her previous child. He opens his bowel once a day and is being mix breast and bottle fed. He is floppy, jaundiced, has a large, protruding tongue and a hoarse cry. He had a newborn blood spot screening test done at birth which was normal and he has no dysmorphic features. What is the most likely diagnosis of this child?

A. Beckwith–Wiedemann syndrome
B. Congenital hypothyroidism
C. Down’s syndrome
D. Normal baby
E. Prader–Willi syndrome

A

B. Congenital hypothyroidism

There are many features of hypothyroidsm (B) in this child: hypotonia, jaundice, reduced frequency of bowel movement, evidence of goitre (horse cry and large tounge). The newborn blood spot screening looks for high levels of TSH in response to an underactive thyroid, if the baby has a secondary hypothyroid then it will not show on the test.

The lack of dismorphic features goes against Down’s syndrome (C).

Beckwith-Wiedeman syndrome (A) presents with macroglossia, but it has associated features such as macrosomia, hypoglycaemia, hemi-hypertrophy, ans abnormal ear development.

Prader-Willi (E) would also lead to hypotonia and poor feeding, but the picture here fits better with hypothyroidism.

Jaundice beyond 2 weeks is pathological, this is not a normal baby (D).

134
Q

A 10-year-old girl with Graves’ disease attends her GP with worsening of her
symptoms. She was well controlled on carbimazole and has had relatively few
symptoms for the past 6 months. She now has sweats, weight loss, diarrhoea and
tremors which are affecting her school performance. What is the next management
step?

A. Review in 3 months
B. Radioisotope therapy
C. Stop carbimazole and start propranolol
D. Optimize carbimazole dose and add propanolol
E. Referral for thyroidectomy

A

D. Optimize carbimazole dose and add propanolol

the raised levels of thyroxine are responsible for the symptoms of hyperthroidism, Carbimazole acts to supress the thyroxine production. This girls resurgent symptoms suggest that her hyperthyroidism has gotten more sever and her dosage of carbimazole needs adjusting, adding in propranolol for symptomatic relief would also be advised (D).

Propranolol on its own (C) would not supress the excess thyroxine production.

Until we have explored other management approaches, radioisotope therapy (B) and surgery (E) are not yet indicated.

This patient has symptoms, so choosing no treatment (A) is not an appropriate option.

135
Q

A 4-year-old boy was diagnosed with nephrotic syndrome 6 months ago and
has required a long course of oral corticosteroids to maintain remission of the
condition. He has developed truncal obesity and you are concerned he may be
developing Cushing’s syndrome. Which of the following is not a complication of
Cushing’s syndrome?

A. Osteoporosis
B. Short stature
C. Gastric irritation
D. Hypertension
E. Hypoglycaemia

A

E. Hypoglycaemia

The side effects form an iatrogenic Cushing’s syndrome are numerous and concerning. All of the otions listed here, except (E), are side effects, and there are many more.

One of the side effects of corticosteroid excess is impaired glucose tolerance, leading to hyperglycaemia, not hypo as (E) states.

136
Q

A 13-year-old girl has presented to her GP with her mother with concerns that
she is the shortest in her class at school. She has always been ‘on the small side’
according to her mother, despite eating well. When you examine her you find she
is hypertensive but has no cardiac murmur. Respiratory and abdominal systems are normal. She has no signs of pubertal development and you notice she has widely spaced nipples and a low hair line. You are considering the diagnosis of Turner’s syndrome. What is the most appropriate diagnostic investigation?

A. Mid-parental height
B. Echocardiogram
C. Four limb blood pressures
D. Karyotype
E. Fluorescence in situ hybridization (FISH)

A

D. Karyotype

Turner’s syndrome is due to a 45XO genotype so the only option here that is diagnostic is a karyotype (D)

FISH (E) is for diagnosing microdeletions such as; Digeorge’s syndrome, cri du chat syndrome, and William’s syndrome. The echo (B) would be useful for congenital cardiac or aortic anomolies. The four limb blood pressure (C) would be useful for coarctation of the aorta. Finally the mid-parental height is useful for establishing if short stature is normal or pathological (eg growth hormone deficiency)

137
Q

You are asked to examine a tall 15-year-old boy. His height is above the 98th
centile for his age and he has other concerns about the development of breast
tissue. He was told this was normal as he develops through puberty but his father
states he has no facial or underarm hair. Jake allows a brief examination of his
genitalia and you note he has a small penis and testicular volume. He has no
arachnodactyly or visual problems. What is the most likely diagnosis?

A. Delayed onset of puberty
B. Klinefelter’s syndrome
C. Precocious puberty
D. Marfan’s syndrome
E. Normal variation

A

B. Klinefelter’s syndrome

This boy is showing signs of abnormal pubertal development, in that he has no develpment of secondary charecteristics of genital growth at 15 years of age. This si not a normal variation (E) as 15 years of age is the point where it is not normal for boys to have not started puberty. This is also not a case of delayed pubery (A) as we would expect short stature. The fact that he hasn’t started puberty means that this isn’t precocious puberty (C).

The two conditions here you would expect to lead to tall stature are Klinefelter’s (B) and Marfan’s (D). With Marfan’s you wuld expect an array of other signs; high arched palate, arm span greater than height, , hypermobility, aortic arch abnormalities, mitral valve prolapse, chest wall deformity, myopia and lens dislocation, and arachnodactly. This is not the case here.

The signs we have here (gynaecomastia with delayed puberty, tall stature, and hypogonadism are all associated with Klinefelter’s, which is a karyotype of 47XXY.

138
Q

A 16-year-old boy attends your GP clinic for the first time with his father. He
has recently moved to the area. His father is concerned that he is shorter than
his peers at school and he frequently complains about being bullied. On further
questioning there is no evidence of chronic illness or familial illness and he eats
a balanced diet. His weight is on the 25th centile and his height is on the 10th
centile. On examination he has no evidence of facial, axillary or pubic hair, his
testes are both descended and are <4 mL volume. What is the most likely cause of
his delayed puberty?

A. Anorexia nervosa
B. Hypothalamo-pituitary dysfunction
C. Kallmann’ syndrome
D. Cryptorchidism
E. Constitutional delay

A

E. Constitutional delay

Cryptorchidism (D) is the condition of undescended testes, which is not the case here.

Anorexia nervosa (A) is possible, particularly in light of the bullying, but this child’s weight is on a greater centile to his height. This suggests that his BMI would not be in the range for anorexia, but should be checked.

Kallman’s syndrome (C) is charecterised by low levels of luteinizing hormone-releasing hormone and anosmia, which is not mentioned here. It is very rare, and it doesn;t include gynaecomastia.

Hypothalamo-pituitary dysfunction (B) should be investigated, but is not the likeliest scenario here.

The commonest cause of delayed puberty, and the likely case here, is constitutional delay (E). It is a diagnosis of exclusion and is charecterised by isolated short stature (delayed skeletal growth) and causes delayed puberty.

139
Q

A 6-year-old girl has presented to her GP with a rapid increase in growth. Her
mother is also concerned that she seems to have developed pubic and axillary
hair and breast development prior to this but thought it would go away. She has
no history of trauma and has reported problems with her vision. Her levels of
gonadotrophin-releasing hormone (GnRH), follicle-stimulating hormone (FSH),
luteinizing hormone (LH) and oestrogen are high. You are concerned that she may
have a pituitary tumour. What is the likely visual field defect?

A. Monocular blindness
B. Central scotoma
C. Homonymous hemianopia
D. Bitemporal hemianopia
E. Myopia

A

D. Bitemporal hemianopia

A pituitary tumour grows upwards, as it sits on top of a bony plate (sella turcica), when it gets large enough it begins to press on the optic chiasm.

The optic chiasm carries the fibres of the optic tract arising from the nasal halves of the retina (which register images in the temporal fields). As a growing pituitary stretches and damages these fibres you are left with a bitemporal visual field defect (D).

Of the oher options here;

Monocular blindness (A) is due to a defect before the optic nerve (eg trauma, or cataracts), or a unilateral optic nerve pathology (retinoblastoma).

Central Scotoma (B) can result from multiple sclerosis or optic nerve gliomas

Homonymous hemianopia (C) is a lesion affeting the optic radiation posterior to the lateral geniculate body, this could be a stroke or cerebral palsy,

Myopia (E) is short-sightedness, which has a link to Marfan’s syndrome.

140
Q

A 9-year-old girl presents to accident and emergency with fever, vomiting and
dysuria. She is wearing a steroid bracelet and has a steroid card stating she is
on daily prednisolone for severe asthma and eczema and is therefore at risk of
adrenal suppression. She is tachycardic at 140 bpm and you are concerned that
her blood pressure is low. Her capillary glucose is 3.0 mmol/L. What is the single
most important investigation?

A. Cortisol
B. Full blood count
C. Renal function tests
D. Urine culture
E. Blood culture

A

C. Renal function tests

This girl has a uinary tract infection, as seen in her fever and dysuria. Her body would naturally increase her circulating cortisol in response to illness but due to the exongenous steroid use her hypothhalamic-pituitary-adrenal is downregulated and cannot mount a compensatory response.

The upshot of this is that she is now steroid depleted, an Addisonian crisis, this is the cause of her hypotension, vomiting and hypoglycaemia. She is likely to develop hyperkalaemia and hyponatraemia and needs a renal function test (C) to assess this.

Knowing her cortisol levels (A) won’t actually change her management, which is to give hydrocortisone and IV dextrose, with antibiotics to tret the suspected UTI.

Performing cultures (E and D) and doing a full blood count (B) would help to identify the source of the infection, but would not really help to treat the immediate issue here.

141
Q

A 3 year old is brought to accident and emergency by his parents because he
has not been walking for the past day and refuses to stand. He is normally fit
and healthy but he did have antibiotics for tonsillitis 2 weeks ago. They do not
think he has had any injuries but he attends daycare and something could have
happened there. He is up to date with his immunizations and his parents have
no concerns with his development. On examination he looks well, is apyrexial,
with a heart rate of 120 and respiratory rate of 26 with no bruising. His knees are
normal on examination and the hips have a full range of movement except he
cries on external rotation of the right hip. There are no deformities seen on x-ray
of the hips and knees. After some paracetamol he manages to stand and take a
few antalgic steps with encouragement, limping on the right leg. What is the most
likely diagnosis?

A. Reactive arthritis
B. Non-accidental injury
C. Growing pains
D. Osteomyelitis
E. Septic arthritis

A

A. Reactive arthritis

The picture of a mono-arthritis following an URTI/Tonsilitis is typical of a reactive arthritis (A).

He is well and apyrexial which makes any infective cause (D) or (E) unlikely. YOu would still do a full blood count ans inflammatory markers for him.

Growing pains (C) would not affect the child’s ability to weight bear, and would be more a feature at night.

NAI (B) should always be a consideration, but there is no evidence of injury here and the xray is normal.

142
Q

A 6-year-old girl is taken to see her GP because she is complaining of knee and
elbow pains frequently. Her mother thinks it is worst after her ballet classes and
when she gets home from school. She denies stiffness or pain in the mornings. Her mother has been administrating paracetamol several times a week and is worried that this is too much to be giving a child. On examination, the child looks well and has full range of movement of her joints with evidence of hyperextension. There are no swollen joints or effusions present and she is non-tender on examination. What is the most likely diagnosis?

A. Repetitive strain injury
B. Marfan’s syndrome
C. Hypermobile joints
D. Osteoarthritis
E. Juvenile idiopathic arthritis (JIA)

A

C. Hypermobile joints

There is evidence here on examination of hypermobile joints (C), this is the most likely explaination of the joint pain she experiences after activities. Physiotherapy might be useful here to strengthen the muscles surrounding the joint.

Marfan’s (B) does include hypermobile joints, but there is no ention of any of the other signs associated such as; arm span being greater than height, high arched palate, arachnodactyly, myopia, lens dislocation, or chest wall deformities.

There is no history here of repetitive strain injury (A), and this shouldn’t be developed as a result of a children’s ballet class.

There is no evidence of inflammatory joint changes to support either (D) or (E).

143
Q

A 14-year-old slightly overweight boy is brought into accident and emergency
from a football match where he slipped and fell but was unable to get back up due
to pain in his right leg, which is now looking shortened and externally rotated.
X-rays show the right femur to be disconnected from the femoral head almost
completely at the level of the epiphysis. What is the most appropriate management?

A. Analgesia, nil by mouth until emergency internal fixation can be
performed
B. Antibiotics and nil by mouth while waiting for an open reduction
operation
C. Analgesia and bed rest with traction until healed
D. Analgesia and a hip spica cast
E. Reassure and mobilize with physiotherapy as tolerated

A

A. Analgesia, nil by mouth until emergency internal fixation can be
performed

A slipped upper femoral epiphysis, is more common in teenage boys who are obese, and can often occur following a minor injury. THis needs to be managed by internal fixation (A), the usual approach is to use a pin.

An open approach (B) is unusual as it has a greater rate of avascular necrosis of the femoral head. This option also doesn’t call for any analgesia.

The risk of avascular necrosis with teh bed rest and traction (C) is very high, and so it is not a reccomended approach.

A hip spica cast (D) is the treatment for developmental hip dysplasia

Mobilisation (E) will not help with the avulsed femoral epiphysis, which needs to be pinned back in place to prevent avascular necrosis.

144
Q

What is a greenstick fracture?

A. The classic pattern of vertebral column fractures associated with abuse
by being hit with a cane or ‘green stick’
B. A fracture of the distal radius and ulna with dorsal displacement
associated with a fall on the outstretched hand
C. A fracture of the distal radius and ulna with ventral displacement
D. A fracture of the long bones in young children where only one cortex is
broken and the other is buckled
E. A fracture of the long bones in young children where the cortex is
buckled on one side of the bone with no cortex separation on the
opposite side

A

D. A fracture of the long bones in young children where only one cortex is
broken and the other is buckled

Because te bones of children are more flexible a force applied to the bone may result in the bending of the bone rather than breaking it. This can lead to a buckling of the cortex, if there is just a buckle on one side this is known as a Torus fracture (E). If there is a fracture of one cortex with just a buckle on the other cortex (D) this is known as a greenstick fracture.

The other options here describe various injury patterns.

(B) is a Colles fracture, commonly seen in older patients. In children this type of fall more usually results in a fracture throught he growth plate, known as a Salter-Harris fracture.

(C) is a Smith’s fracture, the opposite of a Colles.

(A) doesn’t describe any recognised pattern of injury.

145
Q

A 2-year-old boy is brought to accident and emergency for the sixth time and
is found to have a right-sided non-displaced transverse fracture of his tibia. His
parents state that he was running in the living room and tripped landing on a
toy truck. He has broken his other leg twice, several fingers and his right arm
previously. He appears healthy, is well dressed and his growth is normal. His
mother is very upset, she is 5 months pregnant with their second child and her
anomaly scan yesterday suggested the baby has a broken leg. What is the most
likely explanation for these fractures?

A. Osteogenesis imperfecta
B. Domestic violence and child abuse
C. Osteopetrosis
D. Achondroplasia
E. Clumsy child

A

A. Osteogenesis imperfecta

Osteogenesis imperfecta (A) is a collegen disorder which is autosomal dominant but with variable penetrance. The most severe cases develop fractures in utero. Given that information it is the most like diagnisis here.

Falling over onto a toy truck shouldn’t fracture a tibia, and the number of previous fractures mean that this is not a case of a clumsy child (E) and something more serious is occuring. Non-accidental injury (B) is always a considertion where the reported mechanism of injury doesn’t correlate with the injury sustained. But the child appears healthy and well loked after, and the prescence of the pathological in utero fracture in the sibling points towards a genetic issue.

Osteopetrosis (C) is an inherited disorder with dense brittle bones, failure to thrive, reccurent infections, hypocalcaemia and thrombocytopenia. Aside from the pathological fractures, nothing else is consistent with that diagnosis.

Achondroplasia (D) is a form of dwarfism, no association with fractures.

146
Q

A 2-month-old baby is brought in by the babysitter because he has been crying
since she arrived to look after him and his right leg looks swollen. He is the only
child living in the household. She does not think he is moving it and is worried it
is injured. On examination he is miserable, his heart rate is 160, respiratory rate
of 56, and capillary refill is less than 2 seconds. He has a swollen right thigh. He
cries more when that leg is examined. You note a yellow bruise on his left thigh
and two purple bruises on either arm. X-rays show a fracture of the right femur but
the arms appear intact. A chest x-ray shows three healing posterior rib fractures.
You are highly suspicious of non-accidental injury. What is the most appropriate
management?

A. Give analgesia and plaster the leg fracture. Ask the babysitter to bring
him back with the parents because he needs to be admitted
B. Give analgesia. Call the duty social worker on-call to get permission to
discharge him once his leg has been plastered
C. Give analgesia and plaster the leg fracture. Contact the parents and
inform them that he needs to be admitted. Ask them to come to the
hospital and inform social services once they have arrived and been
updated
D. Give analgesia and plaster the leg fracture. Call the police to bring the
parents to hospital
E. Give analgesia and plaster the leg fracture. Call the police to arrest the
babysitter for child abuse

A

C. Give analgesia and plaster the leg fracture. Contact the parents and
inform them that he needs to be admitted. Ask them to come to the
hospital and inform social services once they have arrived and been
updated

Given the suspicion of non-accidental injury here you cannot discharge the child until the injuries have been investigted and explained. This is a child protection issue and is of paramount importance. This means that options (A) and (B) are innapropriate.

The police are not a taxi service, and do not ferry people from home to hospital (D)!

Your role in this scenario as a medical professional is to treat teh child and document the injuries, collect evidence, and inform the police if you believe this is a non-accidental injury. The first step is finding out more information by taking a history (C), it certainly isn’t calling the police and telling them to arrest someone(E), even if you had the authority to do so (only a court can compell a police constable to arrest).

147
Q

On a newborn baby screening examination, you see a baby girl born by elective
caesarean section for breech presentation. This is her mother’s first child. The
examination is normal except for a clunk felt on Barlow’s test and a relocation click
on Ortolani’s manoeuvre on the right side. What is the next step in management?

A. Refer to orthopaedics
B. Arrange an ultrasound for 6 weeks of age
C. Refer to physiotherapy
D. Ask a midwife to put on a plaster hip spica
E. Explain to the parents a watch and wait management is most
appropriate as most self-resolve

A

B. Arrange an ultrasound for 6 weeks of age

This child has a few risk factors for developmental hip dysplasia; female sex, first child, and breech presentation. Other factors for hip dysplasia not here include; Clubed foot, positive family Hx, oligohydramnios and macrosomia.

During the Barlow’s examination the hip has displaced and during the Ortolani it has relocated with a ‘click’. This is a positve test

The most apropriate management is to refer for an ultrasound at 6 weeks (B), it is a sensitive test and waiting until 6-weeks improves the specificity as many cases will have self-resolved.

A positive ultrasound would warrent a referal to orthopaedics (A), where the first line therapy would b a Pavlik harness put on by specialist physiotherapists (C)

Midwives are not trained to administer a plaster (D), this is also a specific plaster used post surgery when first line correction has failed.

Due to the risks in later life of walking with a limp, never walking or avascular necrosis of the femoral head, an expectant management (E) is inappropriate.

148
Q

Which of the following is not a correct match?

A. Systemic JIA – acute illness with daily fevers, malaise, failure to thrive,
rash, muscle and joint aches for greater than 6 weeks associated with
raised inflammatory markers

B. Extended oligoarthritis – an arthritis originally affecting one or two
joints for the first 6 weeks and over time has spread to multiple joints
C. Psoriatic arthritis – presents with interphalangeal joint swelling, scaly
skin rash, nail pitting and dactylitis
D. Polyarticular arthritis – more common in boys, affecting multiple small
joints for more than 6 weeks
E. Enthesitis-related arthritis – associated with HLA-B27 tissue type, and
presents in older boys with large joint arthritis, swollen tender tendons,
sacro-iliitis and bamboo spine on x-ray. It is associated with anterior
uveitis which if left untreated may cause blindness

A

D. Polyarticular arthritis – more common in boys, affecting multiple small
joints for more than 6 weeks

polyarticular arthritis is more common in girls and presents with symmetrical arthritis of the wrists, hands, ankles and knees.

149
Q

A 4 year old is brought to accident and emergency acutely unwell and refusing
to walk for the past 2 days. Her parents are not aware of any recent injuries. On
examination, she is pyrexial (T = 39.2°C), capillary refill 3 seconds centrally, heart
rate 150 beats per minute, respiratory rate 40 breaths per minute. Her right thigh
is swollen and slightly erythematous but too tender to examine fully. An x-ray of
the hip and femur shows soft tissue swelling surrounding the proximal femur but
the bones look normal. An urgent MRI shows a periosteal reaction in the proximal
femur with extensive inflammation in the surrounding soft tissues. What is the
most likely diagnosis?

A. Osteomyelitis
B. Non-accidental injury
C. Cellulitis
D. Reactive arthritis
E. Juvenile idiopathic arthritis

A

A. Osteomyelitis

Here we have a child with a painful joint on a background of being acutely unwell, febrile and the early signs of circulatory shock. Given this picture this is a concern for an osteomyelitis (A) or septic arthritis. THe MRI shows a periosteal involvement, which gives us the answer of osteomyelitits (A).

The X-ray showed no trauma to point at a NAI (B)

There is involvement up to the bone on the MRI, excluding a diagnosis of cellulitis (C).

A reactive arthritis (D) would present a week or two after an infection, this child is acutely unwell.

The history is not long enough for JIA (E), which is defined as greater than 6 weeks and no identified infection

150
Q

A 5-year-old is referred to paediatrics due to concerns initially raised by his
school teacher that he is weak and clumsy. On examination he has wasting of his
quadriceps and walks in a waddling gait. His blood creatine kinase is 1600 mmol/L
(normal is 24–190). What is the most likely diagnosis?

A. Muscular dystrophy
B. Neglect with failure to thrive
C. Malnutrition with failure to thrive
D. Acute myositis
E. Spinal muscular atrophy

A

A. Muscular dystrophy

The presentation of a ‘weak’ or ‘lazy’ child, with a waddling gait and with an examination finding of wasting of the quadriceps muscles, leading to calf muscle pseudohypertrophy (were the lower leg looks hypertrophic by comparison), is classic for Muscular Dystrophy, such as Duchene or Beck’s. These are X-linked conditions.

You would expect a positive Gower’s sign, where the child ‘walks’ his hands up the legs in order to stand up from the floor. The profoundly raised creatine kinase demonstrates muscle cell destruction and is diagnostic here.

The other options here are not plausable.

Neglect (B) or malnutrition (C) are impossible to identify wthout any detail on his weight and height or thier changes over time. These would not explain the raised CK either.

There is no mention of pain which would be a prominent feature of acute myositis (D)

Finally, spinal muscular atrophy (E) would present in infancy with a flopy baby and they would never walk.

151
Q

A 3 year old is brought into accident and emergency by ambulance following a
generalized tonic clonic seizure that lasted 2 minutes. She did not require any
treatment to stop the seizure but on arrival the ambulance crew measured her
temperature as 39.2°C and gave paracetamol. She is now apyrexial with a heart
rate of 140, respiratory rate of 30 and capillary refill less than 2 seconds. On
examination she has red enlarged tonsils with no pus, no neck stiffness or rash.
What is the most appropriate management?

A. Oral penicillin
B. Lumbar puncture and IV ceftriaxone
C. Explain that this was a febrile convulsion and discharge home
D. Start phenytoin
E. Discharge home with rescue buccal midazolam for future seizures

A

C. Explain that this was a febrile convulsion and discharge home

This is a typical presentation of a febrile seisure which are linked to rapid rises in temprature during the early stages of a febrile illness. They tend to affect children under 5, they are defined by being simple seizures (lasting less than 5 minutes and not requiring medication to arrest the seizure).

The parents need reassurance (C) that this is a febrile seizure, they are not usually associated with epilepsy but may reccur in future febrile illness. The child should be managed with regular anti-pyretics and lightly dressing the child when feverish.

As tis appears to be a viral tonsilitis (non-exudative) then antibiotics (A, or B) are not indicated, (B) is also incorrect as there is no indication that this is meningoccoccal septiciaemia.

Midazolam (E) is used in pre-hospital settings for seizures lasting longer than 5 minutes and phenytoin (D) is used to stop status ellepticus.

152
Q

A 7 year old is referred to neurology due to frequent episodes of day-dreaming
at school where she is unresponsive. She is falling behind in her work because of
this. An electroencephalograph (EEG) shows three spike waves per second activity
in all leads. What is the most likely diagnosis?

A. Temporal lobe epilepsy
B. Absence epilepsy
C. Day-dreaming
D. Benign Rolandic epilepsy
E. Narcolepsy

A

B. Absence epilepsy

Spike waves are a pathological finding on a EEG, and the description here of 3 spike waves per second in all leads is descriptive of an absence seizure (B). The description of the girl falling behin in her work and being unresponsive is also typical of this condition.

Temporal lobe seizures (A) would appear as spikes on the EEG located just in the temporal area. They present clinically as a seizure with a warning aura/sensation that is focal with an unresponsive patient.

someone who is day dreaming (C) would respond to being called.

Benign Rolandic epilepsy (D) affects young children and presents as early morning partial seizures.

Narcolepsy (E) would be causing her to fall asleep spontaneously at inappropriate times.

153
Q

A mother brings her 2-year-old daughter to the GP on a Monday morning. Over
the weekend she became very upset on being told ‘no’. She was screaming and
then held her breath, went blue and fainted. She woke up quickly and seemed
okay afterwards. However, it has just happened again this morning when she
found some scissors and her mother took them away. On this occasion she had
a brief generalized convulsion lasting about 10 seconds. What is the most likely
explanation?

A. Breath holding attacks
B. Reflex anoxic seizures
C. Absence epilepsy
D. Wolff–Parkinson–White syndrome
E. Vasovagal syncope

A

A. Breath holding attacks

Breath hold attacks (A) are not uncommon in toddlers and result during tantrus, THey will grow out of it and there is no treatment needed. Reassurance is needed for the parent that these brief seizures are not harmful.

Reflex anoxic seizures (B) occur following an episode where a child gets hurt or frightend. The child will go pale and faint.

Absence epilepsy (C) will apear as short (<5 mins) episodes where the child appears vacant and is unresponsive. The child may appear confused following the seizure, as they realise they have missed something.

WPW (D) is a spontaneous re-entrant tachycardia or a SVT that leads to dizzyness, shortness of breath and sometimes causes fainting. There would be a delta wave on the ECG.

A vasovagal (E0 is a simple faint, and is linked with standing for long periods or warm enviroments.

154
Q

A mother with known placenta praevia with heavy vaginal bleeding was rushed
into the labour ward and delivered by emergency caesarean section at 35 weeks’
gestation. Pre-delivery the fetus was bradycardic and after birth APGARs were
three at 1 minute, five at 5 minutes and nine at 10 minutes. Thirty-six hours later
on the special care baby unit the baby is irritable and requiring nasogastric tube
feeds as he is not sucking well. The tone in his upper limbs is reduced and an EEG
showed seizure activity which has been controlled by intravenous phenobarbitone. His cranial ultrasound is normal. His blood sugar monitoring is between 3.5 and 5 mmol/L, C-reactive protein (CRP) was less than 5 mg/L and is 7 mg/L today. He is apyrexial. What is the most likely diagnosis?

A. Intraventricular haemorrhage
B. Group B streptococcal meningitis
C. Hypoglycaemia
D. Mild hypoxic ischaemic encephalopathy (HIE)
E. Moderate HIE

A

E. Moderate HIE

This child has experienced a hypoxic insult perinatally during his mother’s haemorrhage and is showing the following signs of HIE; poor feeding, altered tone and seizure activity on EEG.

A mild case of HIE (D) would have iritibility, startle responses, poor feeding and hyperventilation.

Moderate HIE (E) also shows altered tone or reduced movement, and seizure activity.

THese ischemic changes are difficult to detect in the early stages on ultrasound, but an intraventricular haemorrhage (A) would not have been missed.

Group B strep (B) is a serious cause of neonatal seizures and is life threatening, so when suspected it needs expidient antibiotic treatment. This infant is at descreased risk as this infection is contracted during delivery from the birth canal, whereas this infant was born via caesarean. Also his CRP has returned normal on two ocassions.

This child’s blood glucose eliminates hypoglycaemia (C) as a cause of the patients condition

155
Q

A 15-year-old girl comes to accident and emergency complaining of sudden right
arm weakness and double vision. Last week she was incontinent of urine twice. She is normally fit and well. On examination she has a left-sided 6th nerve palsy and four out of five power in her right arm. The examination is otherwise unremarkable. An MRI head shows multiple hyperintense, inflammatory, white matter lesions. What is the most likely diagnosis?

A. Brain metastasis
B. Multiple sclerosis
C. Tuberous sclerosis
D. Tuberculous meningitis
E. Neurofibromatosis

A

B. Multiple sclerosis

multiple lesions of the white matter, separated in space and time, is essentially gold standard diagnosis of multiple sclerosis (B).

The other options here can cause multiple lesions in the brain but none fit the pattern we see here.

Brain mets (A) and TB (D) would cause space occupying lesions with surrounding oedema.

In Tuberous Sclerosis (D) the MRI typically shows subependymal calcifications and hypointense white matter lesions or tubers.

Neurofibromatosis (E) would cause gliomas or acoustic neuromas which would appear as space occupying lesions.

156
Q

A 15-year-old girl is brought into accident and emergency from school having
disclosed to a friend that she took 10 paracetamol tablets last night. Her blood level of paracetamol is below the treatment line, her liver function tests and clotting are normal. Her father died of a brain tumour 3 years ago and her mother is being treated for reactive depression. The girl tells you that she has been feeling low lately, particularly because she does not think she will do well in her up-coming exams. She regrets taking the tablets and does not think she will do it again. What is the most appropriate management?

A. Admit for monitoring of liver function and Child and Adolescent Mental
Health Services (CAMHS) assessment
B. Refer to CAMHS as an outpatient and discharge as not currently
suicidal
C. Refer to Social Services
D. Start antidepressant – fluoxetine
E. Start IV Parvolex

A
A. Admit for monitoring of liver function and Child and Adolescent Mental
Health Services (CAMHS) assessment

This girl needs to be assessed properly for risk, the team to do that is CAMHS. As she presented after the window of intervention she needs to have her LFTs monitored also (A).

Discharging her as all the other options imply is the wrong choice as these feelings may resurface later and lead to a repat of this self harm, she needs a proper assesment.

There isn’t really a reason here to involve the social services (C)

It’s really not an A&E call to start an antidepressant (D)

Parvolex (E) reverses the toxicity of paracetemol overdose, but is ineffective after 15 hours.

157
Q

Which of the following is not a feature of a UMN lesion?

A. Slow-relaxing Achilles tendon reflex
B. Brisk reflexes
C. Increased tone
D. Decreased power
E. Up-going plantar reflex

A

A. Slow-relaxing Achilles tendon reflex

Slow relaxing reflexes are a sign of illnesses such as hypothyroidism or aLMN lesion such as Guillain-Barre syndrome.

The rest are all features of an UMN lesion.

158
Q

Which for the following is not a feature of raised ICP?

A. Headache
B. Morning vomiting
C. Sun setting eyes
D. Bulging anterior fontanelle
E. Papilloedema

A

D. Bulging anterior fontanelle

Sly question really, a buldging fontanelle indicates an increased amount of fluid in the skull, but that increased volume has been compensated for by the buldging fontanelle, so there is no increase in ICP. Essentially the prescence of the fontanelles removes one of the assumptions of the Munroe-Kellie hypothesis.

159
Q

A 10-year-old girl with sickle cell disease presents to her GP on Monday morning
complaining of weakness in her right leg. She says she collapsed on Saturday
afternoon and has not felt right since. What is the most likely diagnosis?

A. Sickle cell painful crisis
B. Parvovirus B19 infection
C. Aplastic crisis
D. Cerebral infarction
E. Osteomyelitis of the right femur

A

D. Cerebral infarction

The presentation of unilateral weakness and collapse in a sickle cell patient should raise the alarm for a stroke (D).

She has not complained of any pain in the leg so this makes osteomylitis (E) and a painful crisis (A) unlikely.

Sickle cell patients who contract B19 (B) are at a risk of developing an aplastic crisis (C) which would lead to infections, bleeding and severe anaemia, but none of that tallys with the clinical picture here.

160
Q

A 6-year-old boy is registering with a new GP, having just moved to the area.
He is in a wheelchair but is able to mobilize with a fast scissoring gait over short
distances. He has increased tone in his legs and has scars from previous tendon
release surgeries. His upper limbs are normal. His mother says that his school
performance is good and he is writing well. She thinks he was going to have a
Statement of Special Educational Needs assessment before they moved. As the GP, what is the most appropriate next step in management?

A. Reassure his mother that as he is doing well at school he does not need
a statement
B. Refer to a community paediatrician
C. Refer to the physiotherapists and occupational therapists
D. Liaise with his new school teacher to make sure the school is able to
support his physical needs
E. Refer to an educational psychologist

A

B. Refer to a community paediatrician

This child has diplegic cerebal palsy and is going to need a coordinated care package, it certainly wouldn’t be appropriate to fob him off (A).

He will likely need all the support outlined in (C), (D) and (E) as well as involvement of orthopaedics andpotentially social services for grants and support for adaptation of the home.

Due to the complex nature of the case, a community paediatrician (B) is best placed to coordiante this package of support.

161
Q

A 6-year-old boy is taken to see the GP by his mother because he has been getting severe abdominal pains, sometimes with vomiting and yesterday with a headache as well. He has no diarrhoea or constipation. His growth and examination are normal. He has no significant past medical history. In his family history, his maternal grandfather recently died of gastric cancer and mum’s migraines have been worse since his death. She is worried her son is getting gastric cancer too. What is the most likely diagnosis?

A. Crohn’s disease
B. Brain tumour
C. Somatization disorder
D. Gastric cancer
E. Coeliac disease

A

C. Somatization disorder

The poor kid is having a terrible time and this is manifesting as non specific abdominal pain and headache. This is a somatization disorder (C) which encompasses; abdominal migraine, functional, IBS, non-organic pain, and reccurent abdominal pain. This is suported by teh distinct psychological triggers here.

Crohn’s (A) would normally present in an older child (but not always) and would have a presentation of diarrhoea +- blood, failure to thrive, and there are often oral and cutaneous manifestations. there is often a family history.

A brain tumour (B) would have neurological signs and would have a more headache predominant history.

Gastric cancer (D) is a extremely unlikely diagnosis in a 6 year old. But it would have features such as por appetite and weight loss also.

Coeliac (E) presents classically with abdominal distention, diarrhoea and failure to thrive.

162
Q

Which of the following is not the correct side effect of anti-epileptic medicine?

A. Sodium valproate – aplastic anaemia
B. Carbamazepine – visual disturbance
C. Lamotrigine – rash
D. Vigabatrin – behavioural disturbance
E. Levetiracetam – anorexia

A

A. Sodium valproate – aplastic anaemia

Side effects of valporate include; increased appetite, weight gain, hair loss, and liver failure.

163
Q

A 13-year-old Somali girl presents to accident and emergency with a 1-month
history of headaches, weight loss and night sweats. Her father is concerned that
she seems confused and is more unwell with her headache despite paracetamol.
She was born in the UK and has had all her immunizations. She travelled to
Somalia 6 months ago. The rest of the family is well although dad has a cough.
On examination she is thin and looks unwell but is neurologically intact with
no abnormal findings on clinical examination. Which diagnosis needs to be
ruled out first?

A. Brain tumour
B. Tuberculous meningitis
C. Pulmonary tuberculosis

D. Migraines
E. HIV infection

A

B. Tuberculous meningitis

Weight loss, night sweats, Somalia, father with a cough… TB should be very high on your list of differentials. Given the presentation of a headache over a month duration, the confusion and the above, TB meningitis (B) needs to be investigated as a priority.

The lack of cough, sputum and haemoptosis or any respiratory symptoms makes pulmonary TB less likely. Regardless the pulmonary manifestations of TB (C) are much less immediately life threatening.

A brain tumour (A) is unlikely given that she is neurologically intact, and it doesn;t explain the other symptoms here.

A migraine (D) wouldn’t cause night sweats and weight loss.

HIV (E) is a common co-infection with TB, there are cerebral manifestations of HIV, but these tend to be late stage and dementia type.

This patient needs an LP (assuming no signs of raised ICP) and a chest xray in case of pulmonary involvement.

164
Q

A 4-month-old baby being investigated for infantile spasms is noted to have an
ash leaf macule on his back under Wood’s light. His EEG shows hypsarrythmia.
The report of his MRI brain states there are subependymal nodules. What is the
diagnosis?

A. Neurofibromatosis type I
B. Neurofibromatosis type II
C. West’s syndrome
D. Tuberous sclerosis
E. Tay–Sachs disease

A

D. Tuberous sclerosis

Ash leaf macules are a skin manifestation of tuberous sclerosis (D) and these are typical MRI findings.

Neurofibromatosis (A)(B) does cause seizures, but they are much rarer and do not have findings on MRI. The classic cutaneous finding in these conditions is of ‘cafe au lait’ spots.

West’s syndrome (C) is a syndrome of infantile spasms and developmental regression, it can be caused by tuberous sclerosis. It isn’t the best answer as there are clear diagnostic signs of Tuberous sclerosis.

Tay-Sachs (E) is an autosomal reccessive deficiency of hexosaminidase A, it presents with seizures, developmental regression, deafness, progressive loss of motor function and increased tone, it is most often found in Ashkenazi Jewish populations.

165
Q

A 3 month old is brought into accident and emergency with a generalized tonic
clonic seizure. She is apyrexial and the seizure stopped after 15 minutes with rectal
diazepam given by the ambulance crew. Her heart rate is 130, respiratory rate of
36 and capillary refill is less than 2 seconds. On examination she is drowsy, has a
port wine stain on her forehead but is otherwise normal on examination. What is
the most likely cause of her seizure?

A. Sturge–Weber syndrome
B. Tuberous sclerosis
C. Neurofibromatosis type I
D. Meningitis
E. Neurofibromatosis type II

A

A. Sturge–Weber syndrome

Sturge-Weber is a manifestation of a port wine stain (Flat haemangioma), any such marks in the hairline or trigeminal area need an MRI to check for cranial involvement as they are at risk of epilepsy.

Tuberous Sclerosis (B) causes spasms in infancy that are brief tonic movements.

Neurofibromatosis (C),(E) rarely causes causes seizures, it is also associated with distinctive ‘cafe au lait’ spots on the skin.

Meningitis (D) that has progressed far enough to cause seizures would leave the child profoundly unwell, with signs of meningsm and systemic shock.